Tải bản đầy đủ (.pdf) (51 trang)

Đề thi thử THPT quốc gia

Bạn đang xem bản rút gọn của tài liệu. Xem và tải ngay bản đầy đủ của tài liệu tại đây (1.93 MB, 51 trang )

<span class='text_page_counter'>(1)</span><div class='page_container' data-page=1>

<b>DAYHOCTOAN.VN </b>


1


<b>DAYHOCTOAN.VN </b>


<b>Bài 1. </b> Xét

 



2 1 2 1


2 1 2 1


2 1 2 1


1 . 1 . 1


<i>n</i> <i>n</i>


<i>n</i> <i>n</i> <i><sub>k</sub></i> <i><sub>k</sub></i> <i><sub>k</sub></i> <i>k</i> <i><sub>k</sub></i>


<i>n</i> <i>n</i>


<i>k o</i> <i>k o</i>


<i>x</i> <i>x</i> <i>C</i> <i>x</i> <i>C</i> <i>x</i>


 
 
 
 
  


  <sub></sub> <sub></sub>  <sub></sub>





Hệ số của số hạng 2<i>n</i> 1


<i>x</i>  là

0

 

2 1

 

2 2

2

 

2 1

2 1

2


2 1 2 1 2 1 ... 1 2 1


<i>n</i> <i><sub>n</sub></i>


<i>n</i> <i>n</i> <i>n</i> <i>n</i>


<i>C</i> <sub></sub>  <i>C</i> <sub></sub>  <i>C</i> <sub></sub>     <i>C</i> <sub></sub>


Ta lại có


<sub>2</sub> <sub>1</sub>

<sub>2</sub> <sub>1</sub>

<sub>2</sub>

2 1


1 <i>n</i> . 1 <i>n</i> 1 <i>n</i>


<i>x</i>  <i>x</i>   <i>x</i>   có hệ số của <i>x</i>2<i>n</i>1 bằng 0 vì đều chứa lũy thừ bậc chẵn của <i>x</i>


Vậy:

<i>C</i><sub>2</sub>0<i><sub>n</sub></i><sub></sub><sub>1</sub>

 

2 <i>C</i>1<sub>2</sub><i><sub>n</sub></i><sub></sub><sub>1</sub>

 

2 <i>C</i><sub>2</sub>2<i><sub>n</sub></i><sub></sub><sub>1</sub>

2  ...

 

1 2<i>n</i>1

<i>C</i><sub>2</sub>2<i><sub>n</sub>n</i><sub></sub><sub>1</sub>1

2 0


<b>Bài 2. </b> Cho <i>n</i> là số tự nhiên, <i>n</i>2. Chứng minh đẳng thức sau:


2

2



2 0 1 2 2 2 2 1 2



1 2 ... 2 <i>n</i> 1 <i>n</i> 1 2<i>n</i>


<i>n</i> <i>n</i> <i>n</i> <i>n</i> <i>n</i>


<i>n C</i>  <i>n</i> <i>C</i>  <i>n</i> <i>C</i>   <i>C</i>   <i>C</i>  <i>n n</i>  .


<b>Lời giải </b>


Ta có với

 



0


0, 1 , 1


<i>n</i>


<i>n</i> <i><sub>k</sub></i> <i><sub>n k</sub></i>


<i>n</i>
<i>k</i>


<i>x</i> <i>x</i> <i>C x</i> 




  



Đạo hàm hai vế của (1) ta được

1 1 <sub>1</sub>



0


1 ( )


<i>n</i>


<i>n</i> <i><sub>k</sub></i> <i><sub>n k</sub></i>


<i>n</i>
<i>k</i>


<i>n x</i> <i>n k C x</i>




 <sub> </sub>




 



Suy ra

 



1
1


0


1 , 2 .



<i>n</i>


<i>n</i> <i><sub>k</sub></i> <i><sub>n k</sub></i>


<i>n</i>
<i>k</i>


<i>nx x</i> <i>n k C x</i>




 




 



Đạo hàm hai vế của (2) ta được

1



2 1

2 <sub>1</sub>

 



0


1 1 1 . , 3 .


<i>n</i>


<i>n</i> <i>n</i> <i><sub>k</sub></i> <i><sub>n k</sub></i>


<i>n</i>
<i>k</i>



<i>n</i> <i>x</i> <i>n</i> <i>x</i> <i>n k</i> <i>C x</i>




  <sub> </sub>




 <sub></sub> <sub> </sub> <sub></sub> <sub></sub> <sub></sub>


 



Thay <i>x</i>1 vào (3) ta được.


<b>Bài 3. </b> Từ các chữ số 0,1, 2, 3, 4, 5có thể lập được bao nhiêu số tự nhiên gồm bốn chữ số khác


nhau, trong đó phải có chữ số 2 và chữ số?


<b>Lời giải </b>


Gọi số tự nhiên cần lập là <i>X</i> <i>a a a a</i>1 2 3 4,

<i>a</i>10



Vì <i>ai</i>

0,1, 2,3, 4,5 ,

 

<i>i</i>1, 2,3, 4

<sub> nên ta có các trường hợp sau : </sub>


TH1 : Trong <i>X</i>có chữ số 0thì có 3 cách xếp chữ số 0 ; 3cách xếp chữ số 2 ; 2cách xếp chữ số


4





1
3
<i>A</i>


cách xếp chữ số 1; 3; 5
Suy ra có 3.3.2.<i>A</i><sub>3</sub>1 = 54 số


TH 2: Trong X khơng có chữ số 0.


Có bốn cách xếp chữ số 2; ba cách xếp chữ số 4 và 2
3


<i>A</i> cách xếp ba chữ số 1; 3; 5. Suy ra có 4.3.


2
3


<i>A</i> = 72 số.


Vậy có tất cả 54 + 72 = 126 số


<b>Bài 4. </b> Tìm hệ số của số hạng chứa <i>x</i>8 trong khai triển nhị thức Niutơn của


2
5
3
1 <i>n</i>
<i>x</i>
<i>x</i>
 <sub></sub> 


 


  , biết


rằng 1 1 2 2 3 3

 



3<i>n</i> 2 3<i>n</i> 3 3<i>n</i> ... <i>n</i> 6144 *


<i>n</i> <i>n</i> <i>n</i> <i>n</i>


<i>C</i>   <i>C</i>   <i>C</i>   <i>nC</i>  .


<b>Lời giải </b>


(<i>n</i> nguyên dương, <i>x</i>0, <i>C<sub>n</sub>k</i> là số tổ hợp chập <i>k</i> của <i>n</i> phần tử). Ta có:


0 1 1 2 2 2

 



</div>
<span class='text_page_counter'>(2)</span><div class='page_container' data-page=2>

<b>DAYHOCTOAN.VN </b>


2


<b>DAYHOCTOAN.VN </b>


Lấy đạo hàm hai vế của (1) ta được:

1 1 1 2 2 3 3 2 1

 



3 <i>n</i> <i><sub>n</sub></i>3<i>n</i> 2 <i><sub>n</sub></i>3<i>n</i> 3 <i><sub>n</sub></i>3<i>n</i> ... <i><sub>n</sub>n</i> <i>n</i> 2


<i>n</i> <i>x</i>  <i>C</i>   <i>C</i>  <i>x</i> <i>C</i>  <i>x</i> <i>nC x</i>  .
Thay <i>x</i>1 vào (*) ta có: <i>C</i>1<i><sub>n</sub></i>3<i>n</i>12<i>C<sub>n</sub></i>23<i>n</i>23<i>C<sub>n</sub></i>33 ...<i>n</i>3 <i>nC<sub>n</sub>n</i><i>n</i>4<i>n</i>1



Khi đó:

 

1

 



* <i>n</i>4<i>n</i> 6144 3 .


Với <i>n</i>

1, 2,3, 4,5

thì (3) khơng thỏa mãn.
Với <i>n</i>6 thì (3) thỏa mãn.


Với <i>n</i>  và <i>n</i>7 thì 4<i>n</i> 1 7.46 6144


<i>n</i>    nên (3) không thỏa mãn.


Suy ra <i>n</i>6là nghiệm duy nhất của (*).
Ta có:


2
5
3


1 <i>n</i>


<i>x</i>
<i>x</i>


 <sub></sub> 


 


 



12
5
3


1


<i>x</i>
<i>x</i>


 


<sub></sub>  <sub></sub>


 

 



12
12


5
12 3


0


1 <i>k</i> <i>k</i>


<i>k</i>
<i>k</i>


<i>C</i> <i>x</i>



<i>x</i>




 


 <sub></sub> <sub></sub>


 


12 11 72


2
12
0


<i>k</i>
<i>k</i>
<i>k</i>


<i>C x</i>






.



Chọn <i>k</i>

<i>k</i>  ,<i>k</i> 0,1, 2,...,12

thỏa11 72 8
2


<i>k</i> <sub></sub>


8


<i>k</i>


  .
Vậy hệ số của 8


<i>x</i> là 8
12 495
<i>C</i>  .


<b>Bài 5. </b> Số tự nhiên có 4 chữ số khác nhau có dạng <i>abcd</i> , <i>abcd</i>2503. Xét hai trường hợp:
Trường hợp 1: <i>a</i>1, số cách chọn <i>b c d</i>, , thỏa mãn yêu cầu bài toán là <i>A</i><sub>6</sub>3


Trường hợp 2: <i>a</i>1,


+ Nếu <i>b</i>5 suy ra có 4 cách chọn <i>b</i>, khi đó có <i>A</i>52 cách chọn <i>c d</i>, .


+ Nếu <i>b</i>5 suy ra <i>c</i>0, suy ra <i>d</i>1 hoặc <i>d</i> 3. Vậy có tất cả <i>A</i>634<i>A</i>52 2 202 số thỏa mãn


điều kiện đề bài.


<b>Bài 6. </b> Cho hai số tự nhiên<i>n k</i>, thỏa : 0 <i>k</i> <i>n</i>. Chứng minh rằng :


   

<sub>2</sub> <sub>2</sub>

 

<sub>2</sub> 2


0 1


2 . 2 ...


<i>n</i> <i>n</i> <i>n</i>


<i>n k</i> <i>n k</i> <i>n</i> <i>n</i> <i>n</i>


<i>C</i> <sub></sub> <i>C</i> <sub></sub> <sub></sub> <i>C</i>  <i>C</i>   <i>C</i> <sub></sub>


 


<b>Lời giải </b>


Ta có:

1<i>x</i>

 

<i>n</i> 1<i>x</i>

 

<i>n</i>  1 <i>x</i>

2<i>n</i> 

   

<i>C<sub>n</sub></i>0 2 <i>C</i>1<i><sub>n</sub></i> 2 ...

 

<i>C<sub>n</sub>n</i> 2 <i>C</i><sub>2</sub><i>n<sub>n</sub></i>.
Mặt khác : với n cố định thì: 2 . 2


<i>n</i> <i>n</i>


<i>k</i> <i>n k</i> <i>n k</i>


<i>u</i> <i>C</i> <sub></sub> <i>C</i> <sub></sub>

 



 



2 <sub>2</sub> 2 <sub>2</sub> 2 <sub>2</sub>


2



1 2 ... 2


!


<i>n</i> <i>k</i> <i>n</i> <i>k</i> <i>n</i> <i>k</i>


<i>n</i>


 <sub></sub> <sub></sub>   <sub></sub>   <sub></sub> <sub></sub> 


     


 là dãy


số giảm theo <i>k</i>, suy ra : <i>u<sub>k</sub></i> <i>u</i><sub>0</sub> 

 

<i>C</i><sub>2</sub><i>n<sub>n</sub></i> 2


Vậy

   

0 2 1 2

 

2 2


2 . 2 ...


<i>n</i> <i>n</i> <i>n</i>


<i>n k</i> <i>n k</i> <i>n</i> <i>n</i> <i>n</i>


<i>C</i> <sub></sub> <i>C</i> <sub></sub> <sub></sub> <i>C</i>  <i>C</i>   <i>C</i> <sub></sub>


  .


Đẳng thức xảy ra khi <i>k</i>0.



<b>Bài 7. </b> Trong thư viện có12 bộ sách gồm 3 bộ sách Tốn giống nhau, 3 bộ sách Vật lý giống
nhau, 3 bộ sách Hóa học giống nhau và 3 bộ sách Sinh học giống nhau được xếp thành một dãy
sao cho khơng có ba bộ nào cùng một mơn đứng kề nhau. Hỏi có bao nhiêu cách xếp như vậy ?


<b>Lời giải </b>


Gọi <i>A</i> là tập hợp các cách xếp 12 bộ thành một dãy tùy ý
Gọi <i>A</i>1 là tập hợp các cách xếp 3 bộ sách Toán đứng kề nhau


Gọi <i>A</i>2 là tập hợp các cách xếp 3 bộ sách Lý đứng kề nhau


</div>
<span class='text_page_counter'>(3)</span><div class='page_container' data-page=3>

<b>DAYHOCTOAN.VN </b>


3


<b>DAYHOCTOAN.VN </b>


Gọi <i>A</i><sub>4</sub> là tập hợp các cách xếp 3 bộ sách Sinh đứng kề nhau
Gọi *


<i>A</i> là tập hợp các cách xếp thỏa yêu cầu đề bài
Ta có


4 4


* *


1 1


\ <i><sub>i</sub></i> <i><sub>i</sub></i>



<i>i</i> <i>i</i>


<i>A</i> <i>A</i> <i>A</i> <i>A</i> <i>A</i> <i>A</i>


 


   


Mà 12!<sub>4</sub> 369600


(3!)


<i>A</i>  


4


1 2 3 4


4 3 4 2 4 1 4 0


1


10! 8! 6! 4!


60936


(3!) (3!) (3!) (3!)


<i>i</i>


<i>i</i>


<i>A</i> <i>C</i> <i>C</i> <i>C</i> <i>C</i>




    


*


369600 60936 308664


<i>A</i>


   


<b>Bài 8. </b> Rút gọn biểu thức:


1 2 2 2 3 3 4 2010 2011 2011 2012


2012 2 2012 3.2 2012 4.2 2012 ... 2011.2 2012 2012.2 2012


<i>A</i><i>C</i>  <i>C</i>  <i>C</i>  <i>C</i>   <i>C</i>  <i>C</i> <sub>. </sub>


<b>Lời giải </b>


2012 <sub>0</sub> <sub>1</sub> <sub>2</sub> <sub>2</sub> <sub>2012</sub> <sub>2012</sub>


2012 2012 2012 2012 2012



1<i>x</i> <i>C</i> <i>xC</i> <i>x C</i>  ... <i>x Ck</i> <i>k</i>  .... <i>x</i> <i>C</i> (1)


Lấy đạo hàm hai vế của (1) ta có :


2011 <sub>1</sub> <sub>2</sub> <sub>1</sub> <sub>2011</sub> <sub>2012</sub>


2012 2012 2012 2012


2012 1<i>x</i> <i>C</i> 2<i>xC</i>  ... <i>kxk</i><i>Ck</i>  .... 2012<i>x</i> <i>C</i> (2)


Chọn <i>x</i> 2 thay vào (2) có :


2011 <sub>1</sub> <sub>2</sub> <sub>1</sub> <sub>2011</sub> <sub>2012</sub>


2012 2012 2012 2012


2012 1 2 2( 2) ... ( 2)<i>k</i> <i>k</i> .... 2012( 2) (3)


<i>C</i> <i>C</i> <i>k</i> <i>C</i> <i>C</i>


         


1 2 2 2 3 3 4 2010 2011 2011 2012


2012 2012 2012 2012 2012 2012


2012 <i>C</i> 2 <i>C</i> 3.2 <i>C</i> 4.2 <i>C</i> ... 2011.2 <i>C</i> 2012.2 <i>C</i>


        



2012


<i>A</i>


  


<b>Bài 9. </b> Tìm hệ số của <i>x</i>8 trong khai triển nhị thức Niutơn của

<i>x</i>22

<i>n</i><b>, biết: </b>


<b> </b><i>A<sub>n</sub></i>38<i>C<sub>n</sub></i>2<i>C<sub>n</sub></i>149<b>, với </b><i>n</i><i>N n</i>, 3


<b>Lời giải </b>


Ta có 3 8 2 1 49

1



2

8

1

49


2


<i>n</i> <i>n</i> <i>n</i>


<i>n n</i>


<i>A</i>  <i>C</i> <i>C</i>  <i>n n</i> <i>n</i>    <i>n</i>


3 2


7 7 49 0 7


<i>n</i> <i>n</i> <i>n</i> <i>n</i>


       Khi đó:

 

 



7


7 <sub>2 7</sub>


2 2


7
0


2 <i>n</i> 2 <i>k</i> <i>k</i> 2<i>k</i>


<i>k</i>


<i>x</i> <i>x</i> <i>C x</i> 




   

.


Số hạng chứa 8


<i>x</i> là 2 7

<i>k</i>

  8 <i>k</i> 3.
Hệ số của 8


<i>x</i> là 3 3
7.2 280
<i>C</i>  .


<b>Bài 10. </b> <i> </i>Chứng minh rằng:



1 2


( 1) 2.4.6...2


1 ...


3 5 2 1 1.3.5...(2 1)


<i>n</i> <i>n</i>


<i>n</i> <i>n</i> <i>n</i>


<i>C</i> <i>C</i> <i>C</i> <i>n</i>


<i>n</i> <i>n</i>




    


  ,  <i>n</i>


<b>Lời giải </b>


Ta có

2

1 2 4

 

2


2


1<i>x</i> <i>n</i>  1 <i>C xn</i> <i>C xn</i>   ... 1 <i>nC xnn</i> <i>n</i> với



*
<i>n</i>
  


 



1 1


2 1 2 4 2


2


0 0


1 <i>x</i> <i>n</i>d<i>x</i> 1 <i>C x<sub>n</sub></i> <i>C xn</i> ... 1 <i>nC x<sub>n</sub>n</i> <i>n</i> d<i>x</i>


<sub></sub>

 

<sub></sub>

     1 1 1 2 ( 1)


1 ...


3 5 2 1


<i>n</i> <i>n</i>


<i>n</i>


<i>n</i> <i>n</i>


<i>C</i>



<i>C</i> <i>C</i>


<i>n</i>




    




</div>
<span class='text_page_counter'>(4)</span><div class='page_container' data-page=4>

<b>DAYHOCTOAN.VN </b>


4


<b>DAYHOCTOAN.VN </b>


Tính



1
2
0


1 <i>n</i>d


<i>n</i>


<i>I</i> 

<i>x</i> <i>x</i>


Đặt cos , 0,



2


<i>x</i> <i>t t</i>  <sub></sub>


 


d<i>x</i> sin d<i>t t</i>


  


2
2 1
0


sin <i>n</i> d


<i>n</i>


<i>I</i> <i>t t</i>




 


Đặt
2
sin


d sin d


<i>n</i>



<i>u</i> <i>t</i>


<i>v</i> <i>t t</i>


 






2 2


2 2 1 2 2 1


1


0 0


2 cos sin <i>n</i> d 2 (1 sin ) sin <i>n</i> d 2 2


<i>n</i> <i>n</i> <i>n</i>


<i>I</i> <i>n</i> <i>t</i> <i>t t</i> <i>n</i> <i>t</i> <i>t t</i> <i>nI</i> <i>nI</i>


 
 

 

  


1 0
2 2.4...2


2 1 3.5... 2 1


<i>n</i> <i>n</i>


<i>n</i> <i>n</i>


<i>I</i> <i>I</i> <i>I</i>


<i>n</i>  <i>n</i>


  


 


Vậy In=




2.4.6...2
1.3.5... 2 1
<i>n</i>


<i>n</i>
<i>I</i>


<i>n</i>





 

2


Từ

 

1 và

 

2 suy ra điều phải chứng minh


<b>Bài 11. </b> Tìm số các số hạng là số nguyên trong khai triển

3

125


7 3


<b>Lời giải</b>


Ta có



125
125


125


3 2 3


125
0


7 3 3 7


<i>k</i>
<i>k</i>
<i>k</i>


<i>k</i>

<i>C</i>





 

. Do vậy, để số hạng trong khai triển là số nguyên thì


,


0 125


0 125 0 41


125


125 125 3


2


2 2


3 2 1,


3


<i>k</i>


<i>k m</i> <i>m</i>


<i>k</i>



<i>k</i> <i>m</i>


<i>k</i>


<i>k</i> <i>m</i>


<i>k</i>


<i>k</i> <i>m</i> <i>m</i> <i>t</i> <i>t</i>



 <sub></sub> <sub></sub> <sub></sub> <sub></sub>
    
 <sub></sub>   <sub></sub>  
 <sub></sub> <sub></sub> <sub></sub>
    <sub></sub>   <sub></sub>
  
  
   
  
  

.


Vậy có 21 giá trị của m thoả mãn nên có 21 số hạng nhận giá trị nguyên.


<b>Bài 12. </b> Từ tập hợp tất cả các số tự nhiên có năm chữ số mà các chữ số đều khác 0, lấy ngẫu
nhiên một số. Tính xác suất để trong số tự nhiên được lấy ra chỉ có mặt ba chữ số khác nhau.


<b>Lời giải</b>



Xét phép thử : T = ‘Chọn ngẫu nhiên một số tự nhiên có năm chữ số mà các chữ số đều khác 0".
Ta có:  95 59049


Gọi <i>A</i> là biến cố cần tìm xác suất, ta có:


Số cách chọn 3 chữ số phân biệt <i>a, b, c </i>từ 9 chữ số thập phân khác 0 là C . Chọn 2 chữ số cịn lại 3<sub>9</sub>
từ 3 chữ số đó, có 2 trường hợp rời nhau sau đây:


</div>
<span class='text_page_counter'>(5)</span><div class='page_container' data-page=5>

<b>DAYHOCTOAN.VN </b>


5


<b>DAYHOCTOAN.VN </b>


mà <i>a</i>, <i>a</i>, <i>a</i> chiếm chỗ thì chỉ tạo ra cùng một số <i>n</i>, nên trong TH1 này có cả thảy 3 5! 60
3!


  số tự


nhiên.


TH2. 1 trong 2 chữ số còn lại bằng 1 trong 3 chữ số <i>a, b, c</i> và chữ số kia bằng 1 chữ số khác trong
3 chữ số đó: có 3 cách; mỗi hốn vị từ 5! hoán vị của 5 chữ số (chẳng hạn) <i>a</i>, <i>a</i>, <i>b</i>, <i>b</i>, <i>c</i> tạo ra một
số tự nhiên <i>n</i>; nhưng cứ 2! hốn vị của các vị trí mà <i>a</i>, <i>a</i> chiếm chỗ và 2! hoán vị của các vị trí mà


<i>b</i>, <i>b</i> chiếm chỗ thì chỉ tạo ra cùng một số <i>n</i>, nên trong TH2 này có cả thảy 3 5! 90
2!2!


  số tự nhiên.



Vậy: 3


9


9!


(60 90)C 150 150 7 4 3 12600
3!6!


<i>A</i>


          .


Kết luận:

 

12600 1400 0,213382106


59049 6561


<i>A</i>


<i>P A</i>     




<b>Bài 13. </b> Tính tổng:


2 3 2014


0 1 2 2 2013 2013



2013 2013 2013 2013


2 1 2 1 2 1


.2. .2 . ... .2 .


2 3 2014


<i>S</i> <i>C</i>   <i>C</i>   <i>C</i>    <i>C</i> .


<b>Lời giải</b>


Xét

1 2 <i>x</i>

2013<i>C</i><sub>2013</sub>0 <i>C</i>1<sub>2013</sub>. 2

 

<i>x</i> <i>C</i><sub>2013</sub>2 . 2

 

<i>x</i> 2 ... <i>C</i><sub>2013</sub>2013. 2

 

<i>x</i> 2013


 

2014


2 2 2014 2014


2013 2013


1 1


2
1 2


1 5 3


1 2 d 1 2 d 1 2


1



2 4028 4028


<i>x</i>


<i>I</i> 

 <i>x</i> <i>x</i>

 <i>x</i>  <i>x</i>    


 

 

 



2


2 2013


0 1 2 2013


2013 2013 2013 2013


1


. 2 . 2 ... . 2 d


<i>C</i> <i>C</i> <i>x</i> <i>C</i> <i>x</i> <i>C</i> <i>x</i> <i>x</i>


 <sub></sub> <sub></sub> <sub> </sub> 


 




2 3 2014



0 1 2 2 2013 2013


2013 2013 2013 2013


2 1 2 1 2 1


.2. .2 . ... .2 .


2 3 2014


<i>C</i>   <i>C</i>   <i>C</i>    <i>C</i> .


Vậy <i>S</i>


2014 2014


5 3


4028




 .


<b>Bài 14. </b> Chứng minh rằng:


0 1 2 2 3 3 2011 2011 2010


2011 2.2. 2011 3.2 . 2011 4.2 . 2011 ... 2012.2 . 2011 4025.3



<i>C</i>  <i>C</i>  <i>C</i>  <i>C</i>   <i>C</i> 


<b>Lời giải</b>


Ta cã



0


1


<i>n</i>


<i>n</i> <i><sub>k</sub></i> <i><sub>k</sub></i>


<i>n</i>
<i>k</i>


<i>x</i> <i>C x</i>




 



1


0


1



<i>n</i>


<i>n</i> <i><sub>k</sub></i> <i><sub>k</sub></i>


<i>n</i>
<i>k</i>


<i>x</i> <i>x</i> <i>C x</i> 




  

 

1


Đạo hàm hai vế của

 

1 ta được.


1

1


0


1 . 1 1


<i>n</i>


<i>n</i> <i>n</i> <i>k</i> <i>k</i>


<i>n</i>
<i>k</i>


<i>x</i> <i>n x</i> <i>x</i>  <i>k</i> <i>C x</i> 



   

 

2


Thay <i>n</i>2001 và <i>x</i>2 vào

 

2 . Đpcm


</div>
<span class='text_page_counter'>(6)</span><div class='page_container' data-page=6>

<b>DAYHOCTOAN.VN </b>


6


<b>DAYHOCTOAN.VN </b>


tam giác tạo bởi các đường thẳng đó mà mỗi tam giác đều khơng có đỉnh là một trong 20 điểm đã
cho


<b>Lời giải</b>


Số đường thẳng đi qua hai trong 20 điểm đã cho là: 2
20 190
<i>C</i>  .


Số tam giác tạo bởi các đường thẳng đó mà mỗi tam giác đều có đỉnh là một trong 20 điểm đã cho
là <i>C</i><sub>190</sub>3 20<i>C</i><sub>19</sub>3.


Số tam giác tạo bởi các đường thẳng đó mà mỗi tam giác đều có ba đỉnh là ba trong 20 điểm đã
cho là 3


20
<i>C</i> .


Số tam giác tạo bởi các đường thẳng đó mà mỗi tam giác đều có đúng hai đỉnh là hai trong 20


điểm đã cho là 2


20.18.17
<i>C</i>


Số tam giác tạo bởi các đường thẳng đó mà mỗi tam giác đều có đúng một đỉnh là một trong 20


điểm đã cho là 2 2


19 17


20.<i>C C</i>. .


Vậy số tam giác tạo bởi các đường thẳng đó mà mỗi tam giác đều khơng có đỉnh là một trong 20


điểm đã cho là 3 3 3 2 2 2


190 20. 19 20 20.18.17 20. 19. 17 581400


<i>C</i>  <i>C</i> <i>C</i> <i>C</i>  <i>C C</i>  .


<b>Bài 16. </b> Cho khai triển

2 14

15 2 210


0 1 2 210


1 <i>x</i> <i>x</i>  ... <i>x</i> <i>a</i> <i>a x a x</i>  ... <i>a</i> <i>x</i> . Chứng minh rằng:


0 1 2 15


15 15 15 14 15 13 ... 15 0 15.


<i>C a</i> <i>C a</i> <i>C a</i>  <i>C a</i>  


<b>Lời giải</b>


Nhân hai vế của

 

1 với

1<i>x</i>

15, ta được



210


15 15


15


0


1 1 <i><sub>k</sub></i> <i>k</i>


<i>k</i>


<i>x</i> <i>x</i> <i>a x</i>




 


   <sub></sub> <sub></sub>


 

2 . Ta có


<sub>15</sub>

15 15

 

<sub>15</sub>



15
0


1 <i>i</i> 1 <i>i</i> <i>i</i>


<i>i</i>


<i>x</i>

<i><sub>C</sub></i>

<i>x</i>




 

 

3 .


Hệ số của 15


<i>x</i> trong khai triển

 

3 là : 

<i><sub>C</sub></i>

1<sub>15</sub> 15

 

4


15 210

0 1 2 2 15 15



2 210



15 15 15 15 0 1 2 210


0


1 <i><sub>k</sub></i> <i>k</i> ... ...


<i>k</i>


<i>x</i> <i>a x</i> <i>C</i> <i>C x C x</i> <i>C x</i> <i>a</i> <i>a x a x</i> <i>a</i> <i>x</i>





 


 <sub></sub> <sub></sub>        


 

5


Thực hiện phép nhân đa thức ta được hệ số của <i>x</i>15 trong khai triển

 

5 là :


0 1 2 15


15 15 15 14 15 13 ... 15 0
<i>C a</i> <i>C a</i> <i>C a</i>  <i>C a</i> .


Từ

 

2 ,

 

4 và

 

6 ta được

<i><sub>C</sub></i>

<sub>15</sub>0<i>a</i><sub>15</sub>

<i><sub>C</sub></i>

<sub>15</sub>1<i>a</i><sub>14</sub>

<i><sub>C</sub></i>

<sub>15</sub>2<i>a</i><sub>13</sub> ...

<i><sub>C</sub></i>

<sub>15</sub>15<i>a</i><sub>0</sub>  15.(đpcm)


<b>Bài 17. </b> Có bao nhiêusố tự nhiên gồm 6 chữ số khác nhau trong đó hai chữ số kề nhau khơng
cùng là số lẻ?


<b>Lời giải </b>


Gọi số đó là <i>A</i><i>a a a a a a</i><sub>1 2 3 4 5 6</sub> . Từ giả thiết suy ra <i>A</i> có 1 hoặc 2 hoặc 3 chữ số lẻ. TH1: <i>A</i> có 1 chữ
số lẻ


+) <i>a</i>1 lẻ: Số các số A là
1


5 5 600
<i>C P</i> 



+) <i>a</i><sub>1</sub> chẵn: Có 4 cách chọn <i>a</i><sub>1</sub>. Số các số <i>A</i> là

1 4


5 4 5


4. <i>C C</i> <i>P</i> 2400


Tổng có: 600 + 2400 = 3000 số các số <i>A</i> trong đó có đúng một chữ số lẻ.
TH2: <i>A</i> có 2 chữ số lẻ


</div>
<span class='text_page_counter'>(7)</span><div class='page_container' data-page=7>

<b>DAYHOCTOAN.VN </b>


7


<b>DAYHOCTOAN.VN </b>


Vậy số các số <i>A</i> là

1 3


4 4 4


5.5. <i>C C</i> <i>P</i> 9600


+) <i>a</i><sub>1</sub> chẵn: Có 4 cách chọn <i>a</i><sub>1</sub>. Có 6 cách chọn hai vị trí khơng kề nhau của hai số lẻ trong <i>a</i><sub>2</sub> <i>a</i><sub>3</sub>
4


<i>a</i> <i>a</i><sub>5</sub><i>a</i><sub>6</sub>. Vậy số các số <i>A</i> là 4.

<i>C</i><sub>5</sub>2.6.<i>P</i><sub>2</sub>

.<i>A</i><sub>4</sub>3 11520
Tổng có: 9600 + 11520 = 21120 số các số <i>A</i>.
TH3: <i>A</i> có 3 chữ số lẻ


+) <i>a</i>1 lẻ: Có 5 cách chọn <i>a</i>1. Có 5 cách chọn <i>a</i>2. Có 3 cách chọn hai vị trí khơng kề nhau của hai


số lẻ trong <i>a</i>3 <i>a</i>4<i>a</i>5 <i>a</i>6.. Vậy số các số <i>A</i> là




2 2


4 2 4


5.5. <i>C</i> .3.<i>P</i> .<i>A</i> 10800


+) <i>a</i><sub>1</sub> chẵn: Có 4 cách chọn <i>a</i><sub>1</sub>. Có 1 cách chọn 3 vị trí khơng kề nhau của 3 số lẻ trong <i>a</i><sub>2</sub><i>a</i><sub>3</sub> <i>a</i><sub>4</sub> <i>a</i><sub>5</sub>
6


<i>a</i> . Vậy số các số <i>A</i> là 4.

<i>C</i><sub>5</sub>3.1.<i>P</i><sub>3</sub>

.<i>A</i><sub>4</sub>2 2880
Tổng có: 10800 + 2880 = 13680 số các số <i>A</i>.


Tóm lại có: 3000 + 21120 + 13680 = 37800 số các số <i>A</i>.


<b>Bài 18. </b> Tìm hệ số của 8


<i>x</i> trong khai triển nhị thức Niutơn của (<i>x</i>22)<i>n</i>, biết:


3 2 1


8 49, ( , 3)


<i>n</i> <i>n</i> <i>n</i>


<i>A</i>  <i>C</i> <i>C</i>  <i>n</i><i>N n</i> .


<b>Lời giải </b>


Ta có: 3 8 2 1 49 ( 1)( 2) 8 ( 1) 49



2


<i>n</i> <i>n</i> <i>n</i>


<i>n n</i>


<i>A</i>  <i>C</i> <i>C</i>  <i>n n</i> <i>n</i>    <i>n</i>


3 2


7 7 49 0 7


<i>n</i> <i>n</i> <i>n</i> <i>n</i>


      


Khi đó: 2 2 7 7 2(7 )


7
0


( 2)<i>n</i> ( 2) <i>k</i> <i>k</i> 2<i>k</i>


<i>k</i>


<i>x</i> <i>x</i> <i>C x</i> 




   




Số hạng chứa 8


<i>x</i> là 2(7<i>k</i>)  8 <i>k</i> 3.
Hệ số của 8


<i>x</i> là 3 3
7.2 280
<i>C</i>  .


<b>Bài 19. </b> Có bao nhiêu số tự nhiên có 6 chữ số đơi một khác nhau mà trong đó chỉ có một chữ
số lẻ ?


<b>Lời giải </b>


Ta kí hiệu số <i>A</i> là <i>a a a a a a</i><sub>1 2 3 4 5 6</sub>


 Có 5 khả năng chọn một chữ số lẻ


 Mỗi cách chọn 1 chữ số lẻ và 5 chữ số chẵn có <i>P</i><sub>6</sub> 6! Cách sắp xếp 6 chữ số đã
cho vào 6 vị trí từ <i>a</i>1 đến <i>a</i>6


 Nh vậy có 5<i>P</i>6 5.6! cách sắp xếp 10 chữ số từ 0 đến 9 vào 6 vị trí từ <i>a</i>1 đến
6


<i>a</i> mà mỗi cách chỉ có một chữ số lẻ.


*Trong tất cả các cách sắp xếp đó thì những cách xếp có chữ số 0 đứng ở vị trí <i>a</i>1


</div>
<span class='text_page_counter'>(8)</span><div class='page_container' data-page=8>

<b>DAYHOCTOAN.VN </b>



8


<b>DAYHOCTOAN.VN </b>


* Do tính bình đẳng của các chữ số đã chọn có


6
1


số cách sắp xếp khơng phải là số có


6 chữ số và bằng

5

.

5

!



6


!


6


.



5

<sub></sub>



Vậy số các số có 6 chữ số mà trong nó chỉ có một số lẻ là




5.6! 5.5! 5! 30 5   25.5! 3000 số


<b>Bài 20. </b> Có bao nhiêu số gồm 5 chữ số sao cho tổng các chữ số của mỗi số là một số lẻ.


<b>Lời giải </b>



Xét số có 5 chữ số là <i>a</i><i>a a a a a</i><sub>1 2 3 4 5</sub> , để <i>a</i> có tổng các chữ số là một số lẻ có 2 khả
năng xẩy ra:


* Nếu <i>a</i><sub>1</sub>  <i>a</i><sub>2</sub> <i>a</i><sub>3</sub> <i>a</i><sub>4</sub> chẵn thì <i>a</i><sub>5</sub>

1,3,5, 7,9


* Nếu <i>a</i>1  <i>a</i>2 <i>a</i>3 <i>a</i>4 lẻ thì <i>a</i>5

0, 2, 4, 6,8



Mặt khác, số các chữ số có 4 chữ số <i>a a a a</i><sub>1 2 3 4</sub> bằng 9.10.10.109.103 và mỗi số đó sinh
5 số có 5 chữ số mà tổng các chữ số là 1 số lẻ. Vậy có tất cả 5.9.103 45.000 số


<b>Bài 21. </b> Chứng minh rằng với mọi số tự nhiên <i>n</i> tuỳ ý, ta có:


 



  

 



2
2
2
2


0


2 !


! !


<i>n</i>


<i>n</i>


<i>n</i>
<i>k</i>


<i>n</i>


<i>C</i>


<i>k</i> <i>n</i> <i>k</i>







 


 




<b>Lời giải </b>


Từ đồng nhất thức:

2

 



1<i>x</i> <i>n</i>  1 <i>x</i> <i>n</i>. 1<i>x</i> <i>n</i>
Sử dụng nhị thức Niutơn ta có:


 



0 1 2 2 0 1 0 1



2 2 ... 2 ... . ...


<i>n</i> <i>n</i> <i>n</i> <i>n</i> <i>n</i> <i>n</i>


<i>n</i> <i>n</i> <i>n</i> <i>n</i> <i>n</i> <i>n</i> <i>n</i> <i>n</i> <i>n</i>


<i>C</i> <i>C x</i> <i>C x</i>  <i>C</i> <i>C x</i> <i>C x</i> <i>C</i> <i>C x</i> <i>C x</i>


So sánh các hệ số của <i>n</i>


<i>x</i> và sử dụng đẳng thức


<i>C<sub>n</sub>k</i> <i>C<sub>n</sub>n k</i> ,<i>k</i> 0,1,..., .<i>n</i>


Ta được:

   

0 2 1 2

 

2


2 ...


<i>n</i> <i>n</i>


<i>n</i> <i>n</i> <i>n</i> <i>n</i>


<i>C</i>  <i>C</i>  <i>C</i>   <i>C</i>


</div>
<span class='text_page_counter'>(9)</span><div class='page_container' data-page=9>

<b>DAYHOCTOAN.VN </b>


9


<b>DAYHOCTOAN.VN </b>


 





 

 



   

   



2


2 2


2 2


0 0


2 2 2 2


0 1


2 2


2 ! 2 ! !


.
! !


( !) ! ( !) !


...



<i>n</i> <i>n</i>


<i>k</i> <i>k</i>


<i>n</i> <i>n</i> <i>n</i>


<i>n</i> <i>n</i> <i>n</i> <i>n</i> <i>n</i>


<i>n</i> <i>n</i> <i>n</i>


<i>n n</i>


<i>k</i> <i>n k</i> <i>k</i> <i>n k</i>


<i>C</i> <i>C</i> <i>C</i> <i>C</i> <i>C</i>


 




 


 


 <sub></sub>    <sub></sub>


 





Đó là điều cần chứng minh.


<b>Bài 22. </b> Tìm hệ số của 7


<i>x</i> trong khai triển thành đa thức của <i>P x</i>( )  (5<i>x</i>3) (<i>n</i> <i>n</i> *) , biết


1 2 3 2 1 20


2 1 2 2 1 3 2 1 ... 2 1 ... (2 1) 2 1 21.2


<i>k</i> <i>n</i>


<i>n</i> <i>n</i> <i>n</i> <i>n</i> <i>n</i>


<i>C</i> <sub></sub>  <i>C</i> <sub></sub>  <i>C</i> <sub></sub>   <i>kC</i> <sub></sub>   <i>n</i> <i>C</i> <sub></sub>  (*)


<b>Lời giải </b>


Xét


2 1 0 1 2 2 2 1 2 1 2 1
2 1 2 1 2 1 2 1 2 1


(1

<i>x</i>

)

<i>n</i>

<i>C</i>

<i><sub>n</sub></i><sub></sub>

<i>C</i>

<i><sub>n</sub></i><sub></sub>

<i>x</i>

<i>C</i>

<i><sub>n</sub></i><sub></sub>

<i>x</i>

 

...

<i>C</i>

<i><sub>n</sub>n</i><sub></sub>

<i>x</i>

<i>k</i>

 

...

<i>C</i>

<i><sub>n</sub>n</i><sub></sub>

<i>x</i>

<i>n</i>

,

<i>x</i>

(1)


đạo hàm 2 vế của (1) ta có


2 1 2 2 1 1 2 1 2


2 1 2 1 2 1 2 1



(2

<i>n</i>

1)(1

<i>x</i>

)

<i>n</i>

<i>C</i>

<i><sub>n</sub></i><sub></sub>

2

<i>C</i>

<i><sub>n</sub></i><sub></sub>

<i>x</i>

 

...

<i>kC</i>

<i><sub>n</sub>n</i><sub></sub>

<i>x</i>

<i>k</i>

 

... (2

<i>n</i>

1)

<i>C</i>

<i><sub>n</sub>n</i><sub></sub>

<i>x</i>

<i>n</i>

(2)


Chọn <i>x</i>1 thay vào (2) ta có


2 1 2 2 1 2 1


2 1 2 1 2 1 2 1


(2

<i>n</i>

1)2

<i>n</i>

<i>C</i>

<i><sub>n</sub></i><sub></sub>

2

<i>C</i>

<i><sub>n</sub></i><sub></sub>

 

...

<i>kC</i>

<i><sub>n</sub>n</i><sub></sub>

 

... (2

<i>n</i>

1)

<i>C</i>

<i><sub>n</sub>n</i><sub></sub> (*)


2 20


(2

<i>n</i>

1)2

<i>n</i>

21.2 (3)





Nếu <i>n</i>10 ta thấy <i>VT</i>

 

3 <i>VP</i>

 

3 nên <i>n</i>10 loại
tương tự 0 <i>n</i> 10 loại;


10


<i>n</i> thỏa mãnvới <i>n</i>10 : <i>P x</i>( )  (5<i>x</i>3)10  ( 3 5 )<i>x</i> 10


Hệ số của <i>k</i>


<i>x</i> trong khai triển thành đa thức của <i>P x</i>

 

<i>C</i>

<sub>10</sub><i>k</i>

( 3)

10<i>k</i>

.5

<i>k</i>
Hệ số của 7


<i>x</i> trong khai triển thành đa thức của <i>P x</i>

 

là<i>C</i><sub>10</sub>7( 3) .5 3 7  253125000
<b>Bài 23. </b> Tính: 20150 20152 20154 20152014


1 1 1


3 5 2015


<i>S</i> <i>C</i>  <i>C</i>  <i>C</i>   <i>C</i>


Lời giải
Chứng minh:


1
1


1 1


<i>k</i> <i>k</i>


<i>n</i> <i>n</i>


<i>C</i> <i>C</i>


<i>k</i> <i>n</i>







 



Khi đó:

1 3 5 2015



2016 2016 2016 2016
1


...
2016


<i>S</i> <i>C</i> <i>C</i> <i>C</i>  <i>C</i>


Ta lại có:

2016 <sub>0</sub> <sub>1</sub> <sub>2</sub> <sub>3</sub> <sub>2015</sub> <sub>2016</sub>


2016 2016 2016 2016 2016 2016


1 1 <i>C</i> <i>C</i> <i>C</i> <i>C</i>  .... <i>C</i> <i>C</i>


 

2016 <sub>0</sub> <sub>1</sub> <sub>2</sub> <sub>3</sub> <sub>2015</sub> <sub>2016</sub>
2016 2016 2016 2016 2016 2016


1 1 <i>C</i> <i>C</i> <i>C</i> <i>C</i>  .... <i>C</i> <i>C</i>




2016 1 3 5 2015


2016 2016 2016 2016


2 2 <i>C</i> <i>C</i> <i>C</i> .... <i>C</i>


</div>
<span class='text_page_counter'>(10)</span><div class='page_container' data-page=10>

<b>DAYHOCTOAN.VN </b>



10


<b>DAYHOCTOAN.VN </b>


1 3 5 2015 2015


2016 2016 2016 .... 2016 2


<i>C</i> <i>C</i> <i>C</i> <i>C</i>


     
Vậy
2015
2
2016
<i>S</i> 


<b>Bài 24. </b> Có bao nhiêu số tự nhiên có năm chữ số đơi một khác nhau và các chữ số này đều lớn
hơn 4. Hãy tính tổng tất cả các số tự nhiên đó.


<b>Lời giải </b>


Có 5! 120 số thoả mãn điều kiện đầu bài.


Trong 120 số có 60 cặp mà mỗi cặp số đó có tổng 155554
Vậy tổng tất cả các số đó là 155554*60 9333240


<b>Bài 25. </b> Chứng minh rằng :





<b>Lời giải </b>


Ta có

 

 











 100
0
100
100
2
100
100
0
100
100
2
100
2
<i>k</i>
<i>k</i>
<i>k</i>


<i>k</i>
<i>k</i>
<i>k</i>
<i>k</i>
<i>x</i>
<i>C</i>
<i>x</i>
<i>x</i>
<i>C</i>
<i>x</i>
<i>x</i>
<i>x</i>
<i>x</i>


Lấy đạo hàm cả hai vế ta được :

 








 100
0
99
100
99
2
100
2
1


100
<i>k</i>
<i>k</i>
<i>k</i>
<i>x</i>
<i>C</i>
<i>k</i>
<i>x</i>
<i>x</i>
<i>x</i>
Thay
2
1



<i>x</i> vào đẳng thức trên ta được :












 100
0
99


100
2
1
100
0
<i>k</i>
<i>k</i>
<i>k</i>
<i>C</i>
<i>k</i>
Hay
(đpcm) .


<b>Bài 26. </b> Có bao nhiêu số tự nhiên gồm năm chữ số mà trong đó có đúng hai chữ số 1 và ba chữ
số còn lại khác nhau.


<b>Lời giải </b>


Số tạo thành có 5 vị trí. Xét hai trờng hợp:
- <i>Trường hợp 1</i>: Số tạo thành có chữ số 0 .
Có bốn cách chọn vị trí cho chữ số 0 .


Số cách chọn hai trong bốn vị trí còn lại cho chữ số 1 là <i>C</i><sub>4</sub>2.
Số cách chọn hai trong tám chữ số còn lại cho hai vị trí cịn lại là 2


8
<i>A</i> .
Theo qui tắc nhân, số cách chọn là 2 2


4 8



4.<i>C A</i>. 1344.


<i> Trường hợp 2</i>: Số tạo thành khơng có chữ số 0 .


Số cách chọn hai trong năm vị trí cho hai chữ số 1 là <i>C</i><sub>5</sub>2.


0
2
1
200
2
1
199
...
2
1
101
2
1
100
199
100
100
198
99
100
100
1
100


99
0


100  






























<i>C</i>
<i>C</i>
<i>C</i>
<i>C</i>
0
2
1
200
2
1
199
...
2
1
101
2
1
100
199
100
100
198
99
100
100
1
100
99

0


100  


</div>
<span class='text_page_counter'>(11)</span><div class='page_container' data-page=11>

<b>DAYHOCTOAN.VN </b>


11


<b>DAYHOCTOAN.VN </b>


Số cách chọn ba trong tám chữ số khác 0, 1 cho ba vị trí cịn lại là<i>A</i>83.


Theo qui tắc nhân, số cách chọn là 2 3


5. 8 3360
<i>C A</i>  .


Theo qui tắc cộng, số các số phải tìm là 1344 3360 4704  .


<b>Bài 27. </b> Cho hai đường thẳng song song <i>a b</i>, . Trên <i>a</i> lấy 17 điểm phân biệt, trên <i>b</i> lấy <i>n</i> điểm
phân biệt. Tìm <i>n</i> để có 5950 tam giác có các đỉnh là ba trong các điểm đã cho.


<b>Lời giải </b>


Vì tam giác có các đỉnh là 3 trong các điểm đã cho nên xảy ra các TH sau


<b>TH1.</b> Tam giác có 1 đỉnh trên <i>a</i> và 2 đỉnh trên <i>b</i>


 có 1 2



17. <i>n</i>


<i>C C</i> tam giác


<b>TH2.</b> Tam giác có 2 đỉnh trên <i>a</i> và 1 đỉnh trên <i>b</i>
 có <i>C C</i><sub>17</sub>2. <i><sub>n</sub></i>1 tam giác


Vậy số các tam giác có các đỉnh là 3 trong các điểm đã cho là


1 2
17. <i>n</i>


<i>C C</i> + <i>C C</i><sub>17</sub>2. 1<i><sub>n</sub></i> Theo bài ra ta có : <i>C C</i><sub>17</sub>1. <i><sub>n</sub></i>2<i>C C</i><sub>17</sub>2. <i><sub>n</sub></i>15950


17! ! 17! !


. . 5950


1!.16! 2!( 2)! 2!.15! ( 1)!


<i>n</i> <i>n</i>


<i>n</i> <i>n</i>


  


 


2
17



( 1) 136 5950 17 255 11900 0


2 <i>n n</i> <i>n</i> <i>n</i> <i>n</i>


       


20


<i>n</i>


  hoặc <i>n</i> 35 (loại)


<b>Bài 28. </b> Chứng minh:


2 3 4


*


2 3 4


2 3


... 1, , 2


( 1) ( 1) ( 1) ( 1)


<i>n</i>
<i>n</i>



<i>C n</i> <i>C n</i> <i>C n</i> <i>nC n</i>


<i>n</i> <i>n</i>


<i>n</i>  <i>n</i>  <i>n</i>   <i>n</i>    


<b>Lời giải </b>


Xét

 

. 1 1
<i>n</i>


<i>f x</i> <i>x</i>
<i>x</i>


 


 <sub></sub>  <sub></sub>
 


 



1


1 1


1 . 1


<i>n</i>


<i>n</i>


<i>f</i> <i>x</i>


<i>x</i> <i>x</i>






   


  <sub></sub> <sub></sub> <sub></sub>  <sub></sub>


    (1)


 

0 1 2
2


1 1 1


. <i><sub>n</sub></i> <i><sub>n</sub></i> <i><sub>n</sub></i> ... <i><sub>n</sub>n</i> <i><sub>n</sub></i>


<i>f x</i> <i>x C</i> <i>C</i> <i>C</i> <i>C</i>


<i>x</i> <i>x</i> <i>x</i>


 


 <sub></sub>     <sub></sub>


 <sub> </sub>



= <i>C x C<sub>n</sub></i>0 <i><sub>n</sub></i>1 <i>C<sub>n</sub></i>2 1 <i>C<sub>n</sub></i>3 1<sub>2</sub> ... <i>C<sub>n</sub>n</i> 1<i><sub>n</sub></i>


<i>x</i> <i>x</i> <i>x</i>


 <sub></sub> <sub></sub> <sub></sub> <sub> </sub> 


 


 


 



2 3 4


0


2 3 4


2 3 ( 1)


...


<i>n</i>


<i>n</i> <i>n</i> <i>n</i> <i>n</i>


<i>n</i> <i>n</i>


<i>C</i> <i>C</i> <i>C</i> <i>n</i> <i>C</i>



<i>f</i> <i>x</i> <i>C</i>


<i>x</i> <i>x</i> <i>x</i> <i>x</i>


  


  <sub></sub>     <sub></sub>


 


</div>
<span class='text_page_counter'>(12)</span><div class='page_container' data-page=12>

<b>DAYHOCTOAN.VN </b>


12


<b>DAYHOCTOAN.VN </b>


Từ (1) và (2) cho (3)


Trong (3) thay <i>x</i> <i>n</i> 1 thì VT 0 và cho kết quả.


<b>Bài 29. </b> Có bao nhiêu số tự nhiên có 7chữ số khác nhau dạng <i>a a a a a a a</i><sub>1 2 3 4 5 6 7</sub> sao cho


1 2 3 4


4 5 6 7


<i>a</i> <i>a</i> <i>a</i> <i>a</i>


<i>a</i> <i>a</i> <i>a</i> <i>a</i>



  




   


<b>Lời giải </b>


Xét các trường hợp sau


TH<sub>1</sub>: Chọn 7 chữ số bất kỳ khơng có chữ số 0 có <i>C</i><sub>9</sub>7cách.
Sau đó xếp 7 chữ số đó vào 7 vị trí <i>a a a a a a a</i><sub>1 2 3 4 5 6 7</sub>


Vị trí <i>a</i><sub>4</sub> có một cách xếp vì <i>a</i><sub>4</sub> lớn nhất .
Có <i>C</i><sub>6</sub>3 cách xếp 3 vị trí <i>a a a</i><sub>1 2 3</sub>


Còn 1 cách xếp 3 chữ số còn lại vào 3 vị trí <i>a a a</i><sub>5</sub> <sub>6</sub> <sub>7</sub>


Vậy có 7 3


9. 6


<i>C C</i> số thoả mãn yêu cầu bài toán TH<sub>1</sub>


TH<sub>2</sub>: Chọn 7 chữ số bất kỳ phải có chữ số 0 có <i>C</i><sub>9</sub>6cách.
Tương tự TH<sub>1</sub>: Có <i>C C</i><sub>9</sub>6. <sub>5</sub>3 số thoả mãn yêu cầu bài tốn.


Vậy có 7 3 6 3



9 6 9 5 1560


<i>C C</i> <i>C C</i>  (số)


<b>Bài 30. </b> Cho số nguyên dương <i>n</i>. Chứng minh rằng: <sub>1</sub> <sub>2</sub> <sub>1</sub>


2009 2010 2009+n


1 1 1 1


+ +...+


C C C<i>n</i> 2007


<b>Lời giải </b>


Ta có : <sub>1</sub>


2009


1 2008!( 1)! 2008!( 1)!


(2009 ( 2))


(2009 )! (2009 )!2007


<i>k</i>
<i>k</i>



<i>k</i> <i>k</i>


<i>k</i> <i>k</i>


<i>C</i>  <sub></sub> <i>k</i> <i>k</i>


 


    


 


2008 2007!( 1)! 2007!( 2)!


2007 (2008 )! (2009 )!


<i>k</i> <i>k</i>


<i>k</i> <i>k</i>


   


 <sub></sub>  <sub></sub>


 


 


1 2



2008 2009


2008 1 1


2007 <i>Ck</i> <sub></sub><i><sub>k</sub></i> <i>Ck</i> <sub></sub><i><sub>k</sub></i>


 


 <sub></sub>  <sub></sub>


 


Lấy tổng : <sub>1</sub> <sub>1</sub> <sub>2</sub> <sub>1</sub>


0 2009 2008 2009 2008


1 2008 1 1 2008 1 1


.


2007 2007 2007


<i>n</i>


<i>k</i> <i>n</i>


<i>k</i> <i>C</i> <i>k</i> <i>C</i> <i>C</i> <i>n</i> <i>C</i>


 



  


 


 <sub></sub>  <sub></sub> 


 


</div>
<span class='text_page_counter'>(13)</span><div class='page_container' data-page=13>

<b>DAYHOCTOAN.VN </b>


13


<b>DAYHOCTOAN.VN </b>


<b>Bài 31. </b> Đội thanh niên tình nguyện quốc tế có 12 bạn gồm 5 bạn quốc tịch Việt Nam, 4 bạn
quốc tịch Pháp và 3 bạn quốc tịch Thái Lan. Hỏi có bao nhiêu cách chọn ra 4 bạn vào một tổ
công tác sao cho trong tổ có khơng q 2 quốc tịch?


<b>Lời giải </b>


Chọn 4 bạn bất kỳ cho tổ cơng tác gồm 12 bạn có :<i>C</i><sub>12</sub>4 495(cách)
Chọn 4 bạn cho tổ cơng tác sao cho có cả 3 quốc tịch có :




1 1 2 1 2 1 2 1 1
5 4 3 5 4 3 5 4 3 270


<i>C C C</i> <i>C C C</i> <i>C C C</i>  (cách)



Vậy số cách chọn 4 học sinh vào một tổ công tác sao cho trong tổ không quá 2 quốc tịch
là :


495 270 225 (cách)


<b>Bài 32. </b> Cho các chữ số 0, 1, 2, 3, 4, 5. Có bao nhiêu số gồm 6 chữ số khác nhau được thành


lập từ các chữ số đã cho, trong đó hai chữ số 0 và 1 không đứng cạnh nhau ?


<b>Lời giải </b>


Gọi <i>a a a a a a</i><sub>1 2</sub> <sub>3</sub> <sub>4</sub> <sub>5</sub> <sub>6</sub> là số gồm 6 chữ số đôi một khác nhau được thiết lập từ tập

0,1, 2,3, 4,5 .



+ Để lập thành một số dạng <i>a a a a a a</i>1 2 3 4 5 6 :
1 0


<i>a</i>  nên có 5 cách chọn <i>a</i>1, sau đó chọn một hốn vị 5 chữ số cịn lại.


Do đó có tất cả 5.<i>P</i><sub>5</sub>   5 5! 600 số dạng <i>a a a a a a</i><sub>1 2</sub> <sub>3</sub> <sub>4</sub> <sub>5</sub> <sub>6</sub> .
+ Ta tìm các chữ số có hai chữ số 0 và 1 đứng cạnh nhau:


Có 5 vị trí trong mỗi số <i>a a a a a a</i>1 2 3 4 5 6 để 2 chữ số 0 và 1 đứng cạnh nhau:
3 4 5 6, 1 4 5 6, 1 2 5 6, 1 2 3 6, 1 2 3 4 ,


<i>xya a a a</i> <i>a xya a a</i> <i>a a xya a a a a xya a a a a xy</i> trong đó vị trí đầu bên trái chỉ


có một khả năng là 10<i>a a a a</i><sub>3</sub> <sub>4</sub> <sub>5</sub> <sub>6</sub>,các vị trí cịn lại là một hốn vị của 0 và 1 .


Sau khi chọn vị trí để hai chữ số 0 và 1 đứng cạnh nhau, ta chọn một hoán vị các chữ số


cịn lại cho các chỗ cịn trống.


Do đó có 9 4! 216  <sub> số dạng </sub><i>a a a a a a</i>1 2 3 4 5 6 , trong đó có chữ số 0 và chữ số 1 đứng cạnh


nhau.


Vậy: Có tất cả 600 216 384  <sub> số gồm 6 chữ số khác nhau, trong đó hai chữ số 0 và </sub>1


không đứng cạnh nhau


</div>
<span class='text_page_counter'>(14)</span><div class='page_container' data-page=14>

<b>DAYHOCTOAN.VN </b>


14


<b>DAYHOCTOAN.VN </b>


Ta có:

 



0 0


1 2 2 2


<i>n</i> <i>n</i>


<i>n</i> <i><sub>k</sub></i> <i>k</i> <i><sub>k</sub></i> <i><sub>k</sub></i> <i><sub>k</sub></i>


<i>n</i> <i>n</i>


<i>k</i> <i>k</i>



<i>x</i> <i>C</i> <i>x</i> <i>C</i> <i>x</i>


 


 



0 1 2 2 2 3 3 3


2 2 2 ... 2<i>n</i> <i>n</i> <i>n</i>


<i>n</i> <i>n</i> <i>n</i> <i>n</i> <i>n</i>


<i>C</i> <i>C x</i> <i>C x</i> <i>C x</i> <i>C x</i>


     


Lấy đạo hàm hai vế, ta có:


1 1 2 2 3 3 2 1 1


2<i>n</i> 1 2 <i>x</i> <i>n</i> 2<i>C<sub>n</sub></i>2 .2<i>C x<sub>n</sub></i> 2 .3<i>C x<sub>n</sub></i>  ... 2 .<i>kkC x<sub>n</sub>k</i> <i>k</i>  ... 2 .<i>nnC x<sub>n</sub>n</i> <i>n</i> .
Với <i>x</i>1, ta có:


1 2 2 3 3 1


2 2 .2 2 .3 ... 2 .<i>k</i> <i>k</i> ... 2 .<i>n</i> <i>n</i> 2 .3<i>n</i>


<i>n</i> <i>n</i> <i>n</i> <i>n</i> <i>n</i>


<i>S</i> <i>C</i>  <i>C</i>  <i>C</i>   <i>kC</i>   <i>nC</i>  <i>n</i> 



<b>Bài 34. </b> Từ các chữ số 0, 1, 2, 3, 4, 5, 6 thành lập được bao nhiêu số tự nhiên, mỗi số có 5
chữ số khác nhau, trong đó ln có mặt chữ số 6


<b>Lời giải </b>


Số cần lập có dạng <i>a a a a a</i><sub>1 2</sub> <sub>2</sub> <sub>4 5</sub>, trong đó ln có mặt chữ số 6 .


Xảy ra các trường hợp:


<i>Trường hợp 1</i>: Nếu <i>a</i><sub>1</sub>6. Khi đó, ta chọn 4 chữ số trong 6 chữ số 0, 1, 2, 3, 4, 5, 6


cho 4 vị trí cịn lại trường hợp này có <i>A</i><sub>6</sub>4 số.


<i>Trường hợp 2</i>: Nếu <i>a</i><sub>1</sub>6, có 4 cách chọn vị trí của chữ số 6 . Khi đó, có 5 cách chọn




1 1; 2;3; 4;5


<i>a</i>  . Sau khi chọn <i>a</i><sub>1</sub>và vị trí cho chữ số 6 , cịn lại 3 vị trí được chọn từ 4


chữ số còn lại, nên số cách chọn là 3
5


<i>A</i>  trường hợp này có 3


5


4.5.<i>A</i> số.



Vậy số các số thoả mãn yêu cầu là 4 3


6 4.5. 5 1560
<i>A</i>  <i>A</i> 


<b>Bài 35. </b> Từ các chữ số 0, 1, 2, 3, 4, 5, 6, 7, 8, 9 ta lập được bao nhiêu số tự nhiên có 6 chữ


số, mà các chữ số đơi một khác nhau và trong đó hai chữ số kề nhau không cùng là số lẻ?


<b>Lời giải </b>


Gọi số đó là <i>A</i><i>a a a a a a</i><sub>1 2 3 4 5 6</sub>. Từ giả thiết suy ra <i>A </i>có 1 hoặc 2 hoặc 3 chữ số lẻ.
TH1: <i>A </i>có 1 chữ số lẻ


+) <i>a</i><sub>1</sub> lẻ: Số các số <i>A</i> là <i>C P</i>1<sub>5 5</sub>600


+) <i>a</i><sub>1</sub> chẵn: Có 4 cách chọn <i>a</i><sub>1</sub>. Số các số <i>A </i> là 4.(<i>C C P</i>1<sub>5</sub> <sub>4</sub>4) <sub>5</sub>2400
Tổng có: 600 2400 3000  số các số <i>A</i> trong đó có đúng một chữ số lẻ.


TH2: <i>A </i>có 2 chữ số lẻ


+) <i>a</i><sub>1</sub> lẻ: Có 5 cách chọn <i>a</i>1. Có 5 cách chọn <i>a</i>2 chẵn.


Vậy số các số <i>A </i>là 5.5.(<i>C C P</i>1<sub>4</sub> <sub>4</sub>3) <sub>4</sub> 9600


+) <i>a</i><sub>1</sub> chẵn: Có 4 cách chọn <i>a</i><sub>1</sub>. Có 6 cách chọn hai vị trí khơng kề nhau của hai số lẻ
trong <i>a</i><sub>2</sub> <i>a</i><sub>3</sub><i>a</i><sub>4</sub><i>a</i><sub>5</sub><i>a</i><sub>6</sub>. Vậy số các số <i>A </i>là 2 3


5 2 4



</div>
<span class='text_page_counter'>(15)</span><div class='page_container' data-page=15>

<b>DAYHOCTOAN.VN </b>


15


<b>DAYHOCTOAN.VN </b>


Tổng có: 9600 11520 21120  số các số <i>A</i>.


TH3: <i>A </i>có 3 chữ số lẻ


+) <i>a</i><sub>1</sub> lẻ: Có 5 cách chọn <i>a</i>1. Có 5 cách chọn <i>a</i>2. Có 3 cách chọn hai vị trí khơng kề


nhau của hai số lẻ trong <i>a</i><sub>3</sub><i>a</i><sub>4</sub><i>a</i><sub>5</sub> <i>a</i><sub>6</sub>.. Vậy số các số <i>A </i>là 5.5.(<i>C</i><sub>4</sub>2.3. ).<i>P A</i><sub>2</sub> <sub>4</sub>2 10800


+) <i>a</i><sub>1</sub> chẵn: Có 4 cách chọn <i>a</i><sub>1</sub>. Có 1 cách chọn 3 vị trí khơng kề nhau của 3 số lẻ
trong


2


<i>a</i> <i>a</i><sub>3</sub> <i>a</i><sub>4</sub><i>a</i><sub>5</sub> <i>a</i><sub>6</sub>. Vậy số các số <i>A </i>là 4.(<i>C</i><sub>5</sub>3.1. ).<i>P A</i><sub>3</sub> <sub>4</sub>22880
Tổng có: 10800 2880 13680  số các số <i>A</i>.


Tóm lại có: 3000 21120 13680 37800   số các số <i>A</i>.


<b>Bài 36. </b> Từ các chữ số 1; 2;3; 4;5; 6 lập được bao nhiêu số tự nhiên gồm 8 chữ số sao cho trong
mỗi số đó có đúng ba chữ số 1, các chữ số còn lại đôi một khác nhau và hai chữ số chẵn không


đứng cạnh nhau?



<b>Lời giải </b>


Số hoán vị 5 chữ số lẻ 1;1;1;3;5 là


Ứng với mỗi hốn vị có .... vị trí đầu, cuối và xen kẽ giữa 2 chữ số lẻ. Do đó có 3
6
<i>A</i>


cách sắp xếp ba chữ số chẵn 2; 4; 6 vào 3 trong 6vị trí đó để được số thỏa đề bài.


Vậy số các số thỏa đề bài là: 3
6
5!


2400.
3!<i>A</i> 


<b>Bài 37. </b> Có bao nhiêu số tự nhiên có ba chữ số <i>abc</i> thỏa mãn điều kiện <i>a b c</i>  .


<b>Lời giải </b>


Ta xét 4 trường hợp sau:


<b>TH1:</b> <i>a b c</i> 


Mỗi số <i>abc</i> là một tổ hợp chập 3 của chín phần tử

1; 2;...;9 suy ra số các số

<i>abc</i> thỏa
mãn <i>a b c</i>  là <i>C</i><sub>9</sub>3.


TH2. <i>a</i> <i>b c</i>



Mỗi số <i>abc</i> là một tổ hợp chập 2 của chín phần tử

1; 2;...;9

suy ra số các số <i>abc</i> thỏa
mãn <i>a</i> <i>b c</i> là 2


9
<i>C</i> .


<b>TH3:</b> <i>a b</i> <i>c</i>


Mỗi số <i>abc</i> là một tổ hợp chập 2 của chín phần tử

1; 2;...;9 suy ra số các số

<i>abc</i> thỏa
mãn <i>a b</i> <i>c</i> là 2


9
<i>C</i> .


<b>TH4:</b> <i>a</i> <i>b</i> <i>c</i>


Số các số <i>abc</i> thỏa mãn <i>a</i> <i>b</i> <i>c</i> là 1
9
<i>C</i> .


Vậy số các số thỏa mãn yêu cầu bài toán là 3 2 2 1


9 9 9 9 165


<i>C</i> <i>C</i> <i>C</i> <i>C</i>  .


<b>Bài 38. </b> Cho hai đường thẳng song song <i>d</i> và <i>d</i>. Trên <i>d</i>có 10 điểm phân biệt, trên <i>d</i>có <i>n</i>


</div>
<span class='text_page_counter'>(16)</span><div class='page_container' data-page=16>

<b>DAYHOCTOAN.VN </b>



16


<b>DAYHOCTOAN.VN </b>


<b>Lời giải </b>


<b>Cách 1: </b>


Ba điểm không thẳng hàng xác định một tam giác. Do đó, có các trường hợp sau:
- Tam giác có một đỉnh trên <i>d</i> và 2 đỉnh trên <i>d</i>, số tam giác loại này là: 2


10.<i>C<sub>n</sub></i>


- Tam giác có một đỉnh trên <i>d</i>và 2 đỉnh trên <i>d</i>, số tam giác loại này là: <i>n C</i>. <sub>10</sub>2


Từ giả thiết, ta có phương trình: 2 2


10


10<i>C<sub>n</sub></i><i>nC</i> 2800 (<i>n</i>2)(*)


(*) . 10 ! 45 2800


2!( 2)!


<i>n</i>


<i>n</i>
<i>n</i>



  


 .5 (<i>n n</i> 1) 45<i>n</i>2800


2 20


8 560 0 20


28


<i>n</i>


<i>n</i> <i>n</i> <i>n</i>


<i>n</i>





    <sub>  </sub>  


 .


<b>Cách 2: </b>


Tổng số điểm trên <i>d</i> và <i>d</i> là <i>n</i>10. Số cách lấy 3 điểm từ <i>n</i>10 điểm trên là: 3
10


<i>n</i>



<i>C</i><sub></sub>


.


Số cách lấy 3 điểm thuộc <i>d</i> là: <i>C</i><sub>10</sub>3 (3 điểm loại này không tạo thành tam giác).
Số cách lấy 3 điểm thuộc <i>d</i> là: 3


<i>n</i>


<i>C</i> (3 điểm loại này không tạo thành tam giác).


Số tam giác tạo thành là: 3 3 3


10 10


<i>n</i> <i>n</i>


<i>C</i><sub></sub> <i>C</i> <i>C</i> .


Theo đề bài tao có: 3 3 3


10 10 2800 ( 3)


<i>n</i> <i>n</i>


<i>C</i><sub></sub> <i>C</i> <i>C</i>  <i>n</i> ….


<b>Bài 39. </b> Có bao nhiêusố tự nhiên gồm 6 chữ số khác nhau trong đó hai chữ số kề nhau khơng
cùng là số lẻ?



<b>Lời giải </b>


Gọi số đó là <i>A</i><i>a a a a a a</i>1 2 3 4 5 6 . Từ giả thiết suy ra <i>A</i> có 1 hoặc 2 hoặc 3 chữ số lẻ.
<b>Trường hợp 1</b>: <i>A</i> có 1 chữ số lẻ.


+) <i>a</i>1 lẻ: Số các số <i>A</i> là: 5.5! 600 .


+) <i>a</i>1 chẵn: Có 4 cách chọn <i>a</i>1, chọn 1 số lẻ đặt vào 5 vị trí có
1
5


5.<i>C</i> cách, chọn 4 số
chẵn và đặt vào 4 vị trí cịn lại có 4! cách. Số các số A là: 4.5.A .4! 24001<sub>5</sub>  .


Tổng có: 600 2400   3000 số các số <i>A</i> trong đó có đúng một chữ số lẻ.


<b>Trường hợp 2</b>: <i>A</i> có 2 chữ số lẻ.


+) <i>a</i>1 lẻ: Có 5 cách chọn <i>a</i>1. Có 5 cách chọn <i>a</i>2 chẵn.


Vậy số các số <i>A</i> là 1 3


4 4 4


5.5.(<i>C C P</i>) 9600.


+) <i>a</i>1 chẵn: Có 4 cách chọn <i>a</i>1, có 6 cách chọn hai vị trí khơng kề nhau của hai số lẻ


trong <i>a</i>2<i>a</i>3 <i>a</i>4 <i>a</i>5 <i>a</i>6. Vậy số các số A là



2 3


5 2 4


4.(<i>C</i> .6. ).<i>P A</i> 11520.
Tổng có: 9600 11520   21120 số các số<i>A</i>.


<b>Trường hợp 3</b>: <i>A</i> có 3 chữ số lẻ.


+) <i>a</i>1 lẻ: Có 5 cách chọn <i>a</i>1, có 5 cách chọn <i>a</i>2, có 3 cách chọn hai vị trí khơng kề


nhau của hai số lẻ trong <i>a</i>3 <i>a</i>4<i>a</i>5 <i>a</i>6.. Vậy số các số <i>A</i> là


2 2


4 2 4


5.5.(<i>C</i> .3. ).<i>P A</i> 10800


+) <i>a</i><sub>1</sub> chẵn: Có 4 cách chọn <i>a</i><sub>1</sub>, có 1 cách chọn 3 vị trí khơng kề nhau của 3 số lẻ trong


2


<i>a</i> <i>a</i><sub>3</sub> <i>a</i><sub>4</sub> <i>a</i><sub>5</sub> <i>a</i><sub>6</sub>. Vậy số các số A là 3 2


5 3 4


</div>
<span class='text_page_counter'>(17)</span><div class='page_container' data-page=17>

<b>DAYHOCTOAN.VN </b>


17



<b>DAYHOCTOAN.VN </b>


Tóm lại có: 3000 21120 13680    37800 số các số<i>A</i>.


<b>Bài 40. </b> Tìm hệ số của 18


<i>x</i> trong khai triển của

<sub>2 – </sub> 2

3<i>n</i>


<i>x</i> biết <i>n</i> * thoả mãn đẳng thức sau:


0 2 4 2


2 2 2 ... 2 512


<i>n</i>


<i>n</i> <i>n</i> <i>n</i> <i>n</i>


<i>C</i> <i>C</i> <i>C</i>  <i>C</i>  .


<b>Lời giải </b>


Ta có:

1 1

2<i>n</i> <i>C</i><sub>2</sub>0<i><sub>n</sub></i><i>C</i><sub>2</sub>1<i><sub>n</sub></i><i>C</i><sub>2</sub>2<i><sub>n</sub></i><i>C</i><sub>2</sub>3<i><sub>n</sub></i> ... <i>C</i><sub>2</sub>2<i><sub>n</sub>n</i>1<i>C</i><sub>2</sub>2<i><sub>n</sub>n</i>

 

1 .


2 0 1 2 3 2 1 2


2 2 2 2 2 2


1 1 <i>n</i> <i>C</i> <i><sub>n</sub></i> <i>C</i> <i><sub>n</sub></i><i>C</i> <i><sub>n</sub></i><i>C</i> <i><sub>n</sub></i> ... <i>C</i> <i><sub>n</sub>n</i> <i>C</i> <i><sub>n</sub>n</i>

 

2 .

Cộng từng vế

 

1 và

 

2 ta được:




2 0 2 4 2 0 2 4 2 2 1


2 2 2 2 2 2 2 2


2 <i>n</i> 2 <i>C</i> <i><sub>n</sub></i> <i>C</i> <i><sub>n</sub></i><i>C</i> <i><sub>n</sub></i> ... <i>C</i> <i><sub>n</sub>n</i> <i>C</i> <i><sub>n</sub></i><i>C</i> <i><sub>n</sub></i><i>C</i> <i><sub>n</sub></i> ... <i>C</i> <i><sub>n</sub>n</i> 2 <i>n</i> .
Theo bài ra ta có: 22<i>n</i>15122<i>n</i>   1 9 <i>n</i> 5


Từ đó,

<sub>2 –</sub> 2

3<i>n</i> <sub> 2 –</sub>

2

15


<i>x</i>  <i>x</i> 


15


15 2


15
0


(2) ( 1)


<i>i</i> <i>i</i> <i>i</i> <i>i</i>


<i>i</i>


<i>C</i>  <i>x</i>







Hệ số của 18


<i>x</i> là số


15
152 ( 1)


<i>i</i> <i>i</i> <i>i</i>


<i>C</i>   sao cho 2<i>i</i> 18  <i>i</i> 9.
Vậy hệ số của 18


<i>x</i> là: -<i>C</i><sub>15</sub>926  320.320.


<b>Bài 41. </b> Đội thanh niên tình nguyện quốc tế có 12 bạn gồm 5 bạn quốc tịch Việt Nam, 4 bạn
quốc tịch Pháp và 3 bạn quốc tịch Thái Lan. Hỏi có bao nhiêu cách chọn ra 4 bạn vào một tổ
công tác sao cho trong tổ có khơng q 2 quốc tịch?


<b>Lời giải </b>


Chọn 4 bạn bất kỳ cho tổ cơng tác gồm 12 bạn có: <i>C</i><sub>12</sub>4 495(cách).


Chọn 4 bạn cho tổ công tác sao cho có cả 3 quốc tịch có:


1 1 2 1 2 1 2 1 1
5 4 3 5 4 3 5 4 3 270



<i>C C C</i> <i>C C C</i> <i>C C C</i>  (cách).


Vậy số cách chọn 4 học sinh vào một tổ công tác sao cho trong tổ không quá 2 quốc
tịch là:


495 270 225 (cách).


<b>Bài 42. </b> Tìm<i>n</i> nguyên dương thỏa mãn: <i>C<sub>n</sub></i>13<i>C<sub>n</sub></i>27<i>C<sub>n</sub></i>3 ...

2<i>n</i>1

<i>C<sub>n</sub>n</i> 32<i>n</i>2<i>n</i>6480


<b>Lời giải </b>


Xét

1<i>x</i>

<i>n</i> <i>C<sub>n</sub></i>0<i>C x C x</i>1<i><sub>n</sub></i>.  <i><sub>n</sub></i>2. 2<i>C x<sub>n</sub></i>3. 3 ... <i>C x<sub>n</sub>n</i>. <i>n</i>


+ Với <i>x</i>2 ta có: 0 1 2 3


3<i>n</i> 2 4 8 ... 2<i>n</i> <i>n</i>


<i>n</i> <i>n</i> <i>n</i> <i>n</i> <i>n</i>


<i>C</i> <i>C</i> <i>C</i> <i>C</i> <i>C</i>


     

 

1


+ Với <i>x</i>1 ta có: 0 1 2 3


2<i>n</i><i>Cn</i><i>Cn</i><i>Cn</i><i>Cn</i> ... <i>Cnn</i>

 

2


+ Lấy

   

1 – 2 ta được: <i>C<sub>n</sub></i>13<i>C<sub>n</sub></i>27<i>C<sub>n</sub></i>3 ...

2<i>n</i>1

<i>C<sub>n</sub>n</i> 3<i>n</i>2<i>n</i>
+ PT <sub>3</sub><i>n</i><sub>2</sub><i>n</i> <sub>3</sub>2<i>n</i> <sub>2</sub><i>n</i><sub>6480</sub> <sub>3</sub>2<i>n</i><sub>3</sub><i>n</i><sub>6480</sub><sub>0</sub><sub>3</sub><i>n</i> <sub>81</sub>


4


<i>n</i>


 


<b>Bài 43. </b> Người ta sơn bề ngoài của một khối lập phương thành màu trắng và cưa thành 64 khối
lập phương nhỏ.Sau đó, từ các khối lập phương nhỏ, người ta xếp để tạo lại khối lập phương cũ,
nhưng lúc ấy các khối lập phương nhỏ có thể thay đổi vị trí và quay đi. Hỏi có bao nhiêu cách sắp
xếp các khối lập phương nhỏ để khối lập phương lớn có bề ngồi được sơn màu trắng.


</div>
<span class='text_page_counter'>(18)</span><div class='page_container' data-page=18>

<b>DAYHOCTOAN.VN </b>


18


<b>DAYHOCTOAN.VN </b>


Trong 64 khối lập phương nhỏ có:
(1) 8 khối được sơn 3 mặt


(2) 24khối được sơn 2 mặt
(3) 24 khối được sơn 1 mặt


(4) 8 khối không được sơn mặt nào.


Rõ ràng là nếu muốn khối lập phương nhận được bên ngồi có sơn thì các khối loại (1)
phải đặt ở đỉnh và mỗi khối có thể đặt theo 3 cách, do đó có 8


3 .8! cách.


Tương tự, đối với loại (2) có 24


2 .24! cách sắp xếp, loại (3) có 4 .24!24 cách sắp xếp và
loại (4) có <sub>24 .8!</sub>8


cách sắp xếp.


Vậy có tất cả là 8 24 24 8

8 48

2


3 .8!.2 .24!. 4 .24!.24 .8! 3 .2 .8!.24!


<b>Bài 44. </b> Có hai hộp đựng bi, mỗi viên bi chỉ mang một màu trắng hoặc đen. Lấy ngẫu nhiên từ
mỗi hộp đúng một viên bi.


<b>a)</b> Biết rằng hộp thứ nhất có 20 viên bi, trong đó có 7 viên bi đen. Hộp thứ hai có 15


viên bi, trong đó có 10 viên bi đen. Tính xác suất để lấy được hai viên bi đen.


<b>b)</b> Biết tổng số bi ở hai hộp là 20 và xác suất để lấy được hai viên bi đen là 55


84 . Tính


xác suất để lấy được hai viên bi trắng.


<b>Lời giải </b>


a) Xét phép thử: Chọn ngẫu nhiên từ mỗi hộp một bi.


Mỗi phần tử của không gian mẫu được chọn nhờ 2 giai đoạn:
1) Chọn 1 bi từ 20 bi hộp 1.



2) Chọn 1 bi từ 15 bi hộp


Suy ra số phần tử không gian mẫu là | | 20.15300


Gọi <i>A</i> là biến cố: Lấy được hai viên bi đen, lập luận tương tự, ta được


|<i>A</i>| 7.10 70 nên xác suất để lấy được hai bi đen là ( ) 7
30
<i>P A</i> 


b) Giả sử hộp 1 có <i>x</i> viên bi, trong đó có <i>a</i> viên bi đen, hộp thứ hai có <i>y</i> viên bi, trong
đó có <i>b</i> viên bi đen. (<i>x y a b</i>, , , là các số nguyên dương,<i>x</i> <i>y</i>, <i>a</i><i>x</i>, <i>b</i><i>y</i>).


Theo lập luận trên và giả thiết, ta có <i>x</i> <i>y</i> 20 và 55
84


<i>ab</i>


<i>xy</i>  nên 55<i>xy</i>84<i>ab</i>. Từ đó


<i>xy</i> chia hết cho 84. Mặt khác 1( )2 100


4


<i>xy</i> <i>x</i><i>y</i>  nên <i>xy</i>84. Ta được <i>x</i>14,<i>y</i>6


Từ đó <i>ab</i>55 nên <i>a</i> là ước của 55.Do <i>a</i>11 và 5


6


55 5
<i>a</i>


<i>b</i>


</div>
<span class='text_page_counter'>(19)</span><div class='page_container' data-page=19>

<b>DAYHOCTOAN.VN </b>


19


<b>DAYHOCTOAN.VN </b>


Vậy xác suất để được 2 bi trắng là (6 5)(14 11) 1


6.14 28


  <sub></sub>


.


<b>Bài 45. </b> Tìm số nghiệm nguyên dương của hệ




1 2 3 2014


1 <i><sub>i</sub></i> 1007 , 1; 2;3


<i>x</i> <i>x</i> <i>x</i>


<i>x</i> <i>i</i>



  





    


 .


<b>Lời giải </b>


Ta có




1 2 3 2014


1 <i><sub>i</sub></i> 1007 , 1; 2;3


<i>x</i> <i>x</i> <i>x</i>


<i>x</i> <i>i</i>


  





    






 




1 2 3 4


4 4


2014 *


1 <i><sub>i</sub></i> 1007 , 1; 2;3 ; 1;


<i>x</i> <i>x</i> <i>x</i> <i>x</i>


<i>x</i> <i>i</i> <i>x</i> <i>x</i>


   



 


     





Xét phương trình nghiệm nguyên dương <i>x</i>1  <i>x</i>2 <i>x</i>3 <i>x</i>4 2014


Số nghiệm của phương trình chính là số cặp

<i>x x x x</i><sub>1</sub>; <sub>2</sub>; <sub>3</sub>; <sub>4</sub>

ta cho tương ứng với dãy


1 2 3 4



11....1011....1011....1011....1


<i>x</i> <i>x</i> <i>x</i> <i>x</i>


, ta phải chọn 3 vị trí đặt số 0 trong 2013 vị trí. Số
nghiệm của phương trình là 3


2013
<i>C</i> .


Xét bài toán ngược: Trong các nghiệm <i>x x x</i>1; 2; 3 có nghiệm lớn hơn1007. Dễ thấy rằng


khơng thể có nhiều hơn một nghiệm lớn hơn 1007 vì tổng bằng 2014. Giả sử <i>x</i>1 1007


, đặt <i>x</i>1 <i>x</i> 1007 thay vào phương trình (*) ta được <i>x</i>  <i>x</i>2 <i>x</i>3 <i>x</i>4 1007. Lý luận


tương tự như trên ta được số nghiệm là 3
1006
<i>C</i> .


Vậy số nghiệm nguyên dương của hệ đã cho là 3 3


2013 3 1006
<i>C</i>  <i>C</i> .


<b>Bài 46. </b> Trong một quyển sách có 800 trang thì có bao nhiêu trang mà số trang có ít nhất một
chữ số 5.


<b>Lời giải </b>



<b> Trường hợp 1:</b> số trang có 1 chữ số: có 1 trang.


<b> Trường hợp 2:</b> số trang có 2 chữ số <i>a a</i><sub>1 2</sub>.


+ Nếu <i>a</i>1  5 <i>a</i>2 có 10 cách chọn  có 10 trang.


+ Nếu <i>a</i>2  5 <i>a</i>1 có 8 cách chọn (vì <i>a</i>10;<i>a</i>15) có 18 trang.
<b>Trường hợp 3:</b> số trang có 3 chữ số <i>a a a</i><sub>1 2</sub> <sub>3</sub>


Do sách có 800 trang <i>a</i>1 chọn từ 17


+ Nếu <i>a</i>1  5 <i>a</i>2 có 10 cách chọn, <i>a</i>3 có 10 cách chọn có 100 trang


</div>
<span class='text_page_counter'>(20)</span><div class='page_container' data-page=20>

<b>DAYHOCTOAN.VN </b>


20


<b>DAYHOCTOAN.VN </b>


+ Nếu <i>a</i><sub>3</sub>  5 <i>a</i><sub>1</sub> có 6 cách chọn, <i>a</i><sub>2</sub> có 9 cách chọn (vì <i>a</i><sub>1</sub>5,<i>a</i><sub>2</sub> 5)  có 54


trang


Vậy số trang thỏa mãn yêu cầu bài toán là: 233 trang.


<b>Bài 47. </b> Tìm các nghiệm nguyên dương của phương trình <i>x</i>  <i>y</i> <i>z</i> 2012. Trong số các nghiệm


này có bao nhiêu nghiệm

<i>x y z</i>0; 0; 0

trong đó <i>x y z</i>0; 0; 0 đôi một khác nhau.



<b>Lời giải </b>


Mỗi bộ ba số nguyên dương

<i>x y z</i>; ;

thỏa mãn <i>x</i>  <i>y</i> <i>z</i> 2012 tương ứng với bộ
(11...1011...1011...1)


<i>x</i> <i>y</i> <i>z</i>


Trong đó có đúng 2012 số 1 và 2 số 0 .


Như vậy bộ ba số cần tìm chính là số cách xếp 2 số 2 và 2012 chữ số 1 vào 2013 vị
trí sao cho 2 số 0 không đứng cạnh nhau và không được đứng đầu và đứng cuối.
Để sắp xếp các số nhứ trên ta thực hiện


+ Sắp xếp 2012 chữ số 1 có 1 cách sắp xếp.


+ Sắp xếp số 0 đầu tiên vào giữa 2012 số 1 ta có 2011 cách xếp ( trừ đi vị trí đầu và
cuối).


+ Sắp xếp chữ số 0 thứ 2 vào giữa 2013 số trên ta được 2010 cách xếp ( không được
sắp đầu cuối và trài phải số 0 ).


+ Vì 2 số 0 có thể đổi chỗ cho nhau nên có 2011.2010


2 các bộ số cần tìm.


Ta có nhận xét 2012 khơng chia hết cho 3 nên phương trình khơng có 3 nghiệm bằng
nhau.


Ta đếm các nghiệm <i>x y z</i>0; 0; 0 trong đó <i>x</i>0  <i>y</i>0. Để có nghiệm loại này ta thấy mỗi cặp



0 0


<i>x</i>  <i>y</i> có duy nhất một số nguyên <i>z</i><sub>0</sub> với 0<i>x</i><sub>0</sub>  <i>y</i><sub>0</sub> 1005 để chọn nghiệm loại này
ta thực hiện:


+ Chọn một số nguyên thuộc[1;1005] vào 2 vị trí <i>x y</i>0; 0có 1005 cách chọn.


+Số cịn lại là <i>z</i>0 là 2012 2 <i>x</i>0 có đúng 1 cách chọn.


Vậy có 1005 bộ ba số

<i>x y z</i>0; 0; 0

trong đó <i>x</i>0 <i>y</i>0.


Vì vai trị của <i>x y z</i>0; 0; 0 đổi chỗ cho nhau nên có 3.1005 các nghiệm có 2 số giống


</div>
<span class='text_page_counter'>(21)</span><div class='page_container' data-page=21>

<b>DAYHOCTOAN.VN </b>


21


<b>DAYHOCTOAN.VN </b>


Vậy có 2011.2010 3.1005 2008.1005


2   các bộ nghiệm

<i>x y z</i>0; 0; 0

trong đó <i>x y z</i>0; 0; 0


đơi một khác nhau.


<b>Bài 48. </b> Từ các số nhiên, lập được bao nhiêu số gồm 6 chữ số khác nhau. Trong đó nhất thiết
phải có mặt hai chữ số 1 và 9 .


<b>Hướng dẫn giải </b>



Gọi số gồm 6 chữ số khác nhau là <i>abcdef</i>


+) 1 và 9 xếp vào 6 vị trí từ <i>a</i> đến <i>f</i> có <i>A</i><sub>6</sub>2 cách chọn
+) <i>a</i>

2;3; 4;5;6;7;8

có 7 các chọn sau khi xếp số 1 và 9
+) Còn lại 7 số sắp xếp vào 3 vị trí có 3


7


<i>A</i> cách chọn.


Suy ra có 2 3


6 7


7.<i>A A</i>. 44100 số


<b>Bài 49. </b> Từ các chữ số 0,1, 2, 3, 4, 5 có thể lập được bao nhiêu số tự nhiên gồm 4 chữ số khác
nhau, trong đó phải có chữ số 2 và 4.


<b>Hướng dẫn giải </b>


Gọi số tự nhiên cần lập là <i>X</i> <i>a a a a</i><sub>1 2</sub> <sub>3</sub> <sub>4</sub>

<i>a</i><sub>1</sub>0 ,

<i>a<sub>i</sub></i>

0,1, 2, 3, 4, 5

 

<i>i</i>1, 2, 3, 4



Trường hợp 1: Trong <i>X</i> có chữ số 0


Có ba cách xếp chữ số 0 , ba cách xếp chữ số 2, hai cách xếp chữ số 4 và <i>A</i><sub>3</sub>1 cách xếp
ba chữ số 1, 3, 5.


Suy ra có 1



3


3.3.2.<i>A</i> 54 số


Trường hợp 2: Trong <i>X</i> khơng có chữ số 0


Có bốn cách xếp chữ số 2, ba cách xếp chữ số 4 và <i>A</i><sub>3</sub>2 cách xếp ba chữ số 1, 3, 5.


Suy ra có 2


3


4.3.<i>A</i> 72 số
Vậy có tất cả 54 72 126  số.


<b>Bài 50. </b> Trong một dạ hội cuối năm ở một cơ quan, ban tổ chức phát ra 100 vé xổ số đánh từ 1


đến 100 cho 100 người. Xổ số có bốn giải: 1 giải nhất, 1 giải nhì, 1 giải ba, 1 giải tư. Kết quả là
việc công bố ai trúng giải nhất, giải nhì, giải ba, giải tư. Hỏi:


a. Có bao nhiêu kết quả có thể, nếu biết rằng nguời giữ vé số 47 được giải nhất?


b. Có bao nhiêu kết quả có thể, nếu biết rằng nguời giữ vé số 47 trúng một trong bốn
giải?


<b>Hướng dẫn giải </b>


a. Nếu giải nhất đã xác định thì 3 giải nhì, ba, tư sẽ rơi vào 99 người còn lại.
Vậy: có 3



99


<i>A</i> 941094 kết quả có thể


b. Người giữ vé số 47 có bốn khả năng trúng một trong bốn giải. Sau khi xác định giải
của người này thì 3 giải cịn lại sẽ rơi vào 99 người không giữ vé số 47 .


Vậy: có 3
99


<i>A</i> khả năng. Theo quy tắc nhân ta có: 3


99


4 2 <i>A</i> 3764376 kết quả có thể


<b>Bài 51. </b> Có bao nhiêu số tự nhiên có 6 chữ số đơi một khác nhau mà trong đó chỉ có một chữ số
lẻ?


<b>Hướng dẫn giải </b>


</div>
<span class='text_page_counter'>(22)</span><div class='page_container' data-page=22>

<b>DAYHOCTOAN.VN </b>


22


<b>DAYHOCTOAN.VN </b>


Có 5 khả năng chọn một chữ số lẻ


Mỗi cách chọn 1 chữ số lẻ và 5 chữ số chẵn có <i>P</i>6 6! Cách sắp xếp 6 chữ số đã cho



vào 6 vị trí từ <i>a</i>1 đến <i>a</i>6


Nh vậy có 5.<i>P</i><sub>6</sub> 5.6! cách sắp xếp 10 chữ số từ 0 đến 9 vào 6 vị trí từ <i>a</i>1 đến <i>a</i>6 mà


mỗi cách chỉ có một chữ số lẻ.


*Trong tất cả các cách sắp xếp đó thì những cách xếp có chữ số 0 đứng ở vị trí <i>a</i><sub>1</sub> khơng
phải là một số có 6 chữ số


* Do tính bình đẳng của các chữ số đã chọn có 1


6 số cách sắp xếp khơng phải là số có 6


chữ số và bằng 5.6! 5.5!
6 


Vậy số các số có 6 chữ số mà trong nó chỉ có một số lẻ là
5.6! 5.5! 5! 30 5 

 

25.5! 3000 số


<b>Bài 52. </b> Có bao nhiêu cách chia hết 200 quyển sách thành bốn phần biết rằng mỗi phần có ít
nhất 20 quyển sách.


<b>Hướng dẫn giải </b>


Gọi , , ,<i>x y z t</i> là số sách mà từng phần có được thì . . .<i>x y z t</i> thoả mãn điệu kiện , , ,<i>x y z t</i>

20;



 



200
<i>x</i>   <i>y</i> <i>z</i> <i>t</i>

 

1


(Đặt<i>a</i> <i>x</i> 19;<i>b</i> <i>y</i> 19;<i>c</i> <i>z</i> 19;<i>d</i>  <i>t</i> 19)
(1) trở thành <i>a b c d</i>   276 (*)


(Đặt<i>a</i> <i>x</i> 19;<i>b</i> <i>y</i> 19;<i>c</i> <i>z</i> 19;<i>d</i>  <i>t</i> 19) thì <i>a b c d</i>, , , dương .
số cách chia quà là số nghiệm nguyên dương của phương trình (*)


+) Đặt 276 dấu chấm theo hàng ngang tạo đơc 275 khoảng trống .Đặt 3 que tính vào ba
khoảng trống bất kỳ. (phần dấu chấm đầu tiên gán cho <i>a</i>. phần dấu chấm thứ hai gán cho
<i>b</i>. phần dấu chấm thứ ba gán cho <i>c</i>. phần dấu chấm thứ <i>t</i> gán cho <i>d</i>) như vậy ta có một
nghiệm nguyên.


... / ... / .../ ...


<i>a</i> <i>b</i> <i>c</i> <i>d</i>


vậy số nghiệm nguyên của pt

 

* là số cách đặt 3 que và ba khoảng trống bất kỳ là số tổ
hợp chập 3 của 275 .


KL: Số cách chia 200 quyển sách thoả mãn đàu bài là là 3


275 3428425


<i>C</i> 


<b>Bài 53. </b> Cho tập hợp <i>X</i> 

1, 2, 3, 4, 5, 6, 7, 8

. Lập một số <i>N</i> gồm 2010 chữ số thỏa mãn <i>N</i>


chia hết 99.999 và các chữ số của <i>N</i> được lấy từ các số thuộc tập <i>X</i> . Tính trung bình cộng tất cả



các số <i>N</i> tìm được.


<b>Hướng dẫn giải </b>


Gọi <i>M</i> là tập hợp các số <i>N</i> thỏa điều kiện đề bài.
Ta xây dựng ánh xạ <i>f</i> như sau :


</div>
<span class='text_page_counter'>(23)</span><div class='page_container' data-page=23>

<b>DAYHOCTOAN.VN </b>


23


<b>DAYHOCTOAN.VN </b>


2010


( ) 99. . . 9


<i>N</i> <i>f N</i>  chia hết cho 99.999


Suy ra <i>f</i> là song ánh từ <i>M</i><i>M</i>


Từ đó ta có


2010


2010
2010


2 ( ) 99 . . . 9 99 . . . 9 10 1



<i>N M</i> <i>N M</i>


<i>N</i> <i>N</i> <i>f N</i> <i>M</i>


 


     




Suy ra trung bình cộng tất cả các số <i>N</i> tìm được là:


2010


10 1


2






<b>Bài 54. </b> Từ các chữ số1, 2, 3, 4, 5, 6 lập được bao nhiêu số tự nhiên gồm 8 chữ số sao cho trong
mỗi số đó có đúng ba chữ số 1, các chữ số cịn lại đơi một khác nhau và hai chữ số chẵn không


đứng cạnh nhau?


<b>Hướng dẫn giải </b>



Số hoán vị 5 chữ số lẻ 1,1,1, 3, 5 là 5!


3!.


Ứng với mỗi hoán vị có 6 vị trí đầu, cuối và xen kẽ giữa 2 chữ số lẻ. Do đó có <i>A</i><sub>6</sub>3 cách
sắp xếp ba chữ số chẵn 2, 4, 6 vào 3 trong 6 vị trí đó để được số thỏa đề bài.


Vậy số các số thỏa đề bài là: 3
6
5!


2400.
3!<i>A</i> 


<b>Bài 55. </b> Trong một buổi tiệc có 10 chàng trai, mỗi chàng trai dẫn theo một cơ gái.


a) Có bao nhiêu cách xếp họ ngồi thành một hàng ngang sao cho các cô gái ngồi cạnh
nhau, các chàng trai ngồi cạnh nhau và có một chàng trai ngồi cạnh cô gái mà anh ta dẫn
theo?


b) Ký hiệu các cô gái là <i>G G</i>1, 2,...,<i>G</i>10. Xếp hết 20 người ngồi thành một hàng ngang sao


cho các điều kiện sau được đồng thời thỏa mãn:


1. Thứ tự ngồi của các cô gái, xét từ trái sang phải là <i>G G</i><sub>1</sub>, <sub>2</sub>,...,<i>G</i><sub>10</sub>.
2. Giữa <i>G</i><sub>1</sub> và <i>G</i><sub>2</sub> có ít nhất 2 chàng trai.


3. Giữa <i>G</i><sub>8</sub> và <i>G</i><sub>9</sub> có ít nhất 1 chàng trai và nhiều nhất 3 chàng trai.
Hỏi có tất cả bao nhiêu cách xếp như vậy ?



<b>Hướng dẫn giải </b>


a) Có 2 10! 9!  cách.


b) Giả sử có 20 chỗ ngồi được đánh số thứ tự từ trái sang phải là 1, 2,..., 20.


Gọi <i>x</i><sub>1</sub> là số chàng trai được xếp bên trái <i>G</i><sub>1</sub>, <i>x</i><sub>2</sub> là số chàng trai được xếp ở giữa <i>G</i><sub>1</sub> và


2


<i>G</i> , <i>x</i><sub>3</sub> là số chàng trai được xếp ở giữa <i>G</i><sub>2</sub> và <i>G</i><sub>3</sub>, ..., <i>x</i><sub>10</sub> là số chàng trai được xếp ở
giữa <i>G</i><sub>9</sub> và <i>G</i><sub>10</sub>, <i>x</i><sub>11</sub> là số chàng trai được xếp bên phải <i>G</i><sub>10</sub>.


Bộ số

<i>x x</i>1, 2,...,<i>x</i>11

hoàn toàn xác định vị trí các cơ gái và:


1) <i>x</i>1<i>x</i>2 ... <i>x</i>11 10


2) <i>x</i><sub>2</sub> 2


3) 1<i>x</i><sub>9</sub> 3


</div>
<span class='text_page_counter'>(24)</span><div class='page_container' data-page=24>

<b>DAYHOCTOAN.VN </b>


24


<b>DAYHOCTOAN.VN </b>


Trong đó các ẩn khơng âm và 1<i>x</i><sub>9</sub> 3


Sử dụng kết quả bài toán chia kẹo Euler ta được số bộ

<i>x x</i>1, 2,...,<i>x</i>11

là:


9 9 9


16 15 14 18447
<i>C</i> <i>C</i> <i>C</i> 


Vậy có 18447.10! cách xếp thỏa đề.


<b>Bài 56. </b> Trên mặt phẳng có 2016 điểm phân biệt là <i>A A</i><sub>1</sub>, <sub>2</sub>,...,<i>A</i><sub>2016</sub>. Từ các điểm trên, bạn An


muốn vẽ các vectơ khác vectơ không, thỏa 2 điều kiện sau:


1. Với mọi <i>i j</i>, 

1; 2;3;...; 2016

, nếu đã vẽ <i>A A<sub>i</sub></i> <i><sub>j</sub></i> thì khơng vẽ <i>A A<sub>j</sub></i> <i><sub>i</sub></i> .


2. Với mọi <i>i j k</i>, , 

1; 2;3;...; 2016

, nếu đã vẽ <i>A Ai</i> <i>j</i> và <i>A Aj</i> <i>k</i> thì khơng vẽ <i>A Ai</i> <i>k</i>.
Hỏi An có thể vẽ nhiều nhất bao nhiêu vectơ?


<b>Hướng dẫn giải </b>


Khơng mất tính tổng qt, giả sử <i>A</i><sub>1</sub> thuộc nhiều vectơ nhất.


Với mỗi điểm <i>A i<sub>i</sub></i>

1; 2;...; 2016

ta chia các điểm còn lại thành 3 loại:
Loại 1: Có nối với <i>A</i><sub>1</sub> và <i>A</i><sub>1</sub> là điểm đầu.


Loại 2: Có nối với <i>A</i>1 và <i>A</i>1 là điểm cuối.


Loại 3: Không nối với <i>A</i><sub>1</sub>.


Giả sử có <i>m</i> điểm loại 1, <i>n</i> điểm loại 2, <i>p</i> điểm loại 3 .
Chú ý rằng:



Giữa các điểm loại 1 không có 2 điểm nào nối lại.
Giữa các điểm loại 2 khơng có 2 điểm nào nối lại.


Giữa <i>A</i>1 và các điểm loại 1, loại 2 có tối đa <i>m n mn</i>  vectơ.


Số vectơ liên quan đến các điểm loại 3 tối đa là <i>p m n</i>

.
Vậy tổng số vectơ tối đa là




<i>m n mn</i>   <i>p m n</i> <i>mn m p</i>

 1

 

<i>n p</i>1



2 <sub>2</sub>


1 2016


3 3


<i>m n</i>  <i>p</i>


  .


Đẳng thức xảy ra khi và chỉ khi <i>m</i>   <i>n</i> <i>p</i> 1 672.
Đưa ra mơ hình.


<b>Bài 57. </b> Cho 5 thư vào 5 phong bì, trên mỗi phong bì đã ghi sẵn địa chỉ của 1 trong 5 thư. Tính
số cách bỏ vào thư nếu :


a. Chỉ có đúng 2 lá thư bỏ đúng địa chỉ.


b. Chỉ có đúng một lá thư bỏ đúng địa chỉ.
c. Khơng có lá thư nào bỏ đúng địa chỉ.


<b>Hướng dẫn giải </b>


Gọi 5 lá thư là <i>T T T T T</i>1, 2, 3, 4, 5. Phong bì là <i>B B B B B</i>1, 2, 3, 4, 5


a. Mỗi cách bỏ thư thực hiện qua 2 bước


Bước 1: Chọn 2 trong 5 thư để bỏ đúng địa chỉ:
có 2


5 10


<i>C</i>  cách chọn (giả sử <i>T T</i><sub>1</sub>, <sub>2</sub> bỏ đúng <i>B B</i><sub>1</sub>, <sub>2</sub>)
Bước 2: Bỏ 3 thư cịn lại đều sai địa chỉ: có 2 cách bỏ


</div>
<span class='text_page_counter'>(25)</span><div class='page_container' data-page=25>

<b>DAYHOCTOAN.VN </b>


25


<b>DAYHOCTOAN.VN </b>


 Bước 1: Chọn 1 trong 5 thư để bỏ đúng địa chỉ có <i>C</i>51 5 cách (giả sử chọn <i>T</i>5 vào
5


<i>B</i> )


 Bước 2: Bỏ <i>T</i><sub>1</sub> vào <i>B B B</i><sub>2</sub>, <sub>3</sub>, <sub>4</sub> có 3 cách



 Bước 3: Bỏ <i>T T T</i>2, 3, 4 vào <i>B B B</i>1, 3, 4 sao cho không thư nào đúng địa chỉ có 3 cách.
<b>Vậy</b>: số cách bỏ 1 thư đúng địa chỉ là: 5 3 3  45 cách


c. Số cách bỏ 5 thư vào 5 bì thư là 5! 120


 Có 45 cách bỏ dúng 1 lá thư vào đúng bì thư


 Có 20 cách bỏ đúng 2 lá thư vào đúng bì thư


 Có <i>C</i><sub>5</sub>2 10 cách bỏ đúng 3 lá thư vào đúng bì thư


 Có 1 cách bỏ đúng 4 lá ( 5 lá ) thư vào đúng bì thư


<b>Vậy</b>: số cách bỏ để khơng lá thư nào đúng địa chỉ là: 120 – 45 20 10 1

   

44 cách.


<b>Bài 58. </b> Trong thư viện có 12 bộ sách gồm 3 bộ sách Toán giống nhau, 3 bộ sách Vật lý giống
nhau, 3 bộ sách Hóa học giống nhau và 3 bộ sách Sinh học giống nhau được xếp thành một dãy
sao cho khơng có ba bộ nào cùng một mơn đứng kề nhau. Hỏi có bao nhiêu cách xếp như vậy ?


<b>Hướng dẫn giải </b>


Gọi <i>A</i> là tập hợp các cách xếp 12 bộ thành một dãy tùy ý
Gọi <i>A</i><sub>1</sub> là tập hợp các cách xếp 3 bộ sách Toán đứng kề nhau
Gọi <i>A</i><sub>2</sub> là tập hợp các cách xếp 3 bộ sách Lý đứng kề nhau
Gọi <i>A</i><sub>3</sub> là tập hợp các cách xếp 3 bộ sách Hóa đứng kề nhau
Gọi <i>A</i>4 là tập hợp các cách xếp 3 bộ sách Sinh đứng kề nhau


Gọi *


<i>A</i> là tập hợp các cách xếp thỏa yêu cầu đề bài


Ta có


4 4


* *


1 1


\ <i><sub>i</sub></i> <i><sub>i</sub></i>


<i>i</i> <i>i</i>


<i>A</i> <i>A</i> <i>A</i> <i>A</i> <i>A</i> <i>A</i>


 


   


Mà 12!<sub>4</sub> 369600


(3!)


<i>A</i>  


4


1 2 3 4


4 3 4 2 4 1 4 0



1


10! 8! 6! 4!


60936


(3!) (3!) (3!) (3!)


<i>i</i>
<i>i</i>


<i>A</i> <i>C</i> <i>C</i> <i>C</i> <i>C</i>




    


*


369600 60936 308664


<i>A</i>


   


<b>Bài 59. </b> Cho tập hợp <i>A</i>

0; 4;5;6;7;8 .

Hỏi có thể lập được bao nhiêu số có 4 chữ số (các chữ
số này chọn từ tập <i>A</i>) sao cho mỗi số tạo thành đều chia hết cho 4?


<b>Hướng dẫn giải </b>



2) Ta có một số chia hết cho 4 khi và chỉ khi hai chữ số cuối của nó lập thành một số
chia hết cho 4.


Gọi số cần tìm là <i>a a a a</i><sub>1 2 3 4</sub> với


4


1, 2, 3, .
<i>a a a a</i> <i>A</i>
Ta có


* <i>a a</i><sub>3</sub> <sub>4</sub> có thể chọn trong các số 00, 04, 08, 40, 44, 48, 56, 60, 64, 68, 76,80,84,88.
Có tất cả 14 cách chọn


</div>
<span class='text_page_counter'>(26)</span><div class='page_container' data-page=26>

<b>DAYHOCTOAN.VN </b>


26


<b>DAYHOCTOAN.VN </b>


* <i>a</i><sub>1</sub> có 5 cách chọn ( vì <i>a</i>10 )


KL: Vậy có 14 6 5 420   số.


Trong một buổi liên hoan có 9 người tham dự. Hỏi có bao nhiêu cách sắp xếp 9 người
này sao cho mỗi người bắt tay với đúng ba người còn lại


<b>Bài 60. </b> Ta sắp một ơ vng có 9 dịng và 9 cột
mỗi dòng và mỗi cột ta tương ứng với người thứ



, 1, 2,.., 9


<i>i i</i> hai người bắt tay với nhau ta đánh số
thứ tự 1, hai người không bắt tay nhau nhau ta đánh


số thứ 0 . Đường chéo của hình vng ta đánh số
0 .


Vì mỗi người bắt ta với đúng ba người nên
mỗi hàng trong hình vng có đúng ba số


1 vì có 9 hàng nên có 27 số 1.


Mặt khác người thứ <i>i</i> bắt tay với người
thứ <i>j</i> thì người thứ <i>j</i> cũng bắt tay với
người thứ <i>i</i> nên các số 1 trên ô vuông đối
xứng nhau qua đường chéo do vậy số các
số 1 phải là số chẵn điều này mâu thuẩn
với 27 số lẻ.


Vậy không thể sắp xếp 9 người sao cho mỗi người bắt tay với đúng ba người còn lại


<b>Bài 61. </b> Tìm số nguyên dương n sao cho:


1 2 2 3 2 2 1


2 1 2.2 2 1 3.2 2 1 ... (2 1).2 2 1 2011


<i>n</i> <i>n</i>



<i>n</i> <i>n</i> <i>n</i> <i>n</i>


<i>C</i> <sub></sub>  <i>C</i> <sub></sub>  <i>C</i> <sub></sub>   <i>n</i> <i>C</i> <sub></sub>  .


<b>Lời giải </b>


Ta có: (1<i>x</i>)2<i>n</i>1<i>C</i><sub>2</sub>0<i><sub>n</sub></i><sub>1</sub><i>C</i><sub>2</sub>1<i><sub>n</sub></i><sub>1</sub>.<i>x C</i> <sub>2</sub>2<i><sub>n</sub></i><sub>1</sub>.<i>x</i>2<i>C</i><sub>2</sub>3<i><sub>n</sub></i><sub>1</sub>.<i>x</i>3 .... <i>C</i><sub>2</sub>2<i><sub>n</sub>n</i><sub>1</sub>1.<i>x</i>2<i>n</i>1 (1)


Lấy đạo hàm cả hai vế của (1) theo x ta được:


2 1 2 3 2 2 1 2


2 1 2 1 2 1 2 1


(2<i>n</i>1)(1<i>x</i>) ( 1)<i>n</i>   <i>C<sub>n</sub></i><sub></sub> 2<i>C<sub>n</sub></i><sub></sub>.<i>x</i>3.<i>C<sub>n</sub></i><sub></sub>.<i>x</i>  .... (2<i>n</i>1).<i>C<sub>n</sub>n</i><sub></sub>.<i>x</i> <i>n</i>


 2 1 2 3 2 2 1 2


2 1 2 1 2 1 2 1


(2<i>n</i>1)(1<i>x</i>) <i>n</i><i>C<sub>n</sub></i><sub></sub> 2<i>C<sub>n</sub></i><sub></sub>.<i>x</i>3.<i>C<sub>n</sub></i><sub></sub>.<i>x</i>  .... (2<i>n</i>1).<i>C<sub>n</sub>n</i><sub></sub>.<i>x</i> <i>n</i> (2)
Cho <i>x</i>2 vào hai vế của (2) ta được:


1 2 1 2


2 1 2 1


2<i>n</i> 1 <i>C<sub>n</sub></i> ... (2 <i>n</i>1).<i>C<sub>n</sub>n</i>.2 <i>n</i>
Khi đó: 2<i>n</i> 1 2011 <i>n</i> 1005.



Vậy <i>n</i>1005.


1 2 3 4 5 6 7 8 9


1 0


2 0 1


3 1 0 1 1


4 1 0


5 0


6 0


7 0


8 1 0


</div>
<span class='text_page_counter'>(27)</span><div class='page_container' data-page=27>

<b>DAYHOCTOAN.VN </b>


27


<b>DAYHOCTOAN.VN </b>


<b>Bài 62. </b> Cho k là số tự nhiên thỏa mãn 5 k 2011. Chứng minh rằng:


0 1 1 5 5



5. 2011 5. 2011 ... 5. 2011 2016


<i>k</i> <i>k</i> <i>k</i> <i>k</i>


<i>C C</i> <i>C C</i>   <i>C C</i>  <i>C</i> .


<b>Lời giải </b>


Dễ thấy

 

5

2011

2016


1<i>x</i> 1<i>x</i>  1 <i>x</i> ; và


5 <sub>0</sub> <sub>1</sub> <sub>1</sub> <sub>2</sub> <sub>2</sub> <sub>3</sub> <sub>3</sub> <sub>4</sub> <sub>4</sub> <sub>5</sub> <sub>5</sub>


5 5 5 5 5 5


M 1 x C C x C x C x C x C x


2011 <sub>0</sub> <sub>1</sub> <sub>1</sub> <sub>k</sub> <sub>k</sub> <sub>2011</sub> <sub>2011</sub>


2011 2011 2011 2011


N 1 x C C x  ... C x  ... C x .


2016 <sub>0</sub> <sub>1</sub> <sub>k</sub> <sub>k</sub> <sub>2016</sub> <sub>2016</sub>


2016 2016 2016 2016


P 1 x C C x ... C  x  ... C x .



Ta có hệ số của k


x trong P là Ck<sub>2016</sub>. Vì <i>P</i><i>M N</i>. , mà số hạng chứa xk trong <i>M N</i>.
là:


0 1 1 1 2 2 2 2 3 3 3 3 4 4 4 4 5 5 5 5


5. 2011 5 2011 5 2011 5 2011 5 2011 5 2011


<i>k</i> <i>k</i> <i>k</i> <i>k</i> <i>k</i> <i>k</i> <i>k</i> <i>k</i> <i>k</i> <i>k</i> <i>k</i> <i>k</i>


<i>C C</i> <i>x</i> <i>C xC</i>  <i>x</i>  <i>C x C</i>  <i>x</i>  <i>C x C</i>  <i>x</i>  <i>C x C</i>  <i>x</i>  <i>C x C</i>  <i>x</i> 


nên


0 1 1 5 5


5. 2011 5. 2011 ... 5. 2011 2016


<i>k</i> <i>k</i> <i>k</i> <i>k</i>


<i>C C</i> <i>C C</i>   <i>C C</i>  <i>C</i> (đfcm).


<b>Bài 63. </b> Cho <i>n</i> là số tự nhiên, <i>n</i>2. Chứng minh đẳng thức sau:


2

2


2 0 1 2 2 2 2 1 2


1 2 ... 2 <i>n</i> 1 <i>n</i> ( 1)2<i>n</i> .



<i>n</i> <i>n</i> <i>n</i> <i>n</i> <i>n</i>


<i>n C</i>  <i>n</i> <i>C</i>  <i>n</i> <i>C</i>   <i>C</i>   <i>C</i>  <i>n n</i> 
<b>Lời giải </b>


Ta có:

 



0


1 , 1


<i>n</i>


<i>n</i> <i><sub>k</sub></i> <i><sub>n k</sub></i>


<i>n</i>
<i>k</i>


<i>x</i> <i>C x</i> 




 



Đạo hàm hai vế

 

1 theo x ta có:



1


1 <sub>1</sub>



0


1 ( )


<i>n</i>


<i>n</i> <i><sub>k</sub></i> <i><sub>n k</sub></i>


<i>n</i>
<i>k</i>


<i>n x</i> <i>n k C x</i>




  




 



Tiếp tục nhân cả hai vế với x ta được:


1 1

 


0


1 , 2


<i>n</i>



<i>n</i> <i>k</i> <i>n k</i>


<i>n</i>
<i>k</i>


<i>nx x</i> <i>n k C x</i>




 <sub></sub>




 



Đạo hàm hai vế

 

2 theo x ta được:


1

 

2 1

2 <sub>1</sub>

 


0


1 1 . . 1 . , 3


<i>n</i>


<i>n</i> <i>n</i> <i><sub>k</sub></i> <i><sub>n k</sub></i>


<i>n</i>
<i>k</i>



<i>n</i> <i>x</i> <i>n</i> <i>x x</i> <i>n k</i> <i>C x</i>




   




 <sub></sub> <sub> </sub> <sub></sub> <sub></sub> <sub></sub>


 

, với<i>x</i>0.


Thay <i>x</i>1vào

 

3 ta được:


2

2


2 0 1 2 2 2 2 1 2


1 2 ... 2 <i>n</i> 1 <i>n</i> ( 1)2<i>n</i> .


<i>n</i> <i>n</i> <i>n</i> <i>n</i> <i>n</i>


<i>n C</i>  <i>n</i> <i>C</i>  <i>n</i> <i>C</i>   <i>C</i>   <i>C</i>  <i>n n</i> 
<b>Bài 64. </b> Tìm số tự nhiên <i>n</i> biết rằng: <i>C</i><sub>2</sub>1<i><sub>n</sub></i><sub></sub><sub>1</sub><i>C</i><sub>2</sub>3<i><sub>n</sub></i><sub></sub><sub>1</sub>...<i>C</i><sub>2</sub>2<i><sub>n</sub>k</i><sub></sub><sub>1</sub>1...<i>C</i><sub>2</sub>2<i><sub>n</sub>n</i><sub></sub><sub>1</sub>165536.


</div>
<span class='text_page_counter'>(28)</span><div class='page_container' data-page=28>

<b>DAYHOCTOAN.VN </b>


28


<b>DAYHOCTOAN.VN </b>



Ta có: (1 1) 2<i>n</i>1 <i>C</i><sub>2</sub>0<i><sub>n</sub></i><sub></sub><sub>1</sub><i>C</i><sub>2</sub>1<i><sub>n</sub></i><sub></sub><sub>1</sub><i>C</i><sub>2</sub>2<i><sub>n</sub></i><sub></sub><sub>1</sub> ... <i>C</i><sub>2</sub>2<i><sub>n</sub>n</i><sub></sub><sub>1</sub><i>C</i><sub>2</sub>2<i><sub>n</sub>n</i><sub></sub><sub>1</sub>1 1

 



 



2 1 0 1 2 2 2 1


2 1 2 1 2 1 2 1 2 1


(1 1) <i>n</i> ... <i>n</i> <i>n</i> 2


<i>n</i> <i>n</i> <i>n</i> <i>n</i> <i>n</i>


<i>C</i> <i>C</i> <i>C</i> <i>C</i> <i>C</i>


 


    


      


Cộng hai vế

 

1 và

 

2 ta được:


1
n
2
1
n
2
3


1
n
2
1
1
n
2
n
2
1
n
2
1
n
2
3
1
n
2
1
1
n
2
1
n
2
C
...
C
C

2
)
C
...
C
C
(
2


2   <sub></sub>  <sub></sub>   <sub></sub>   <sub></sub>  <sub></sub>   <sub></sub> .


Theo bài ra ta có: 22n216n8.


<b>Bài 65. </b> Cho biết ba hệ số đầu của khai triển 1
2
<i>n</i>
<i>x</i>
<i>x</i>
 <sub></sub> 
 


  có các hệ số là ba số hạng liên tiếp


của một cấp số cộng. Tìm tất cả các hạng tử hữu tỷ của khai triển đã cho.


<b>Lời giải </b>


+ Số hạng tổng quát của khai triển: Tk+1=C<i>k<sub>n</sub></i>


3


2
2
<i>n</i> <i>k</i>
<i>k</i>
<i>x</i>



+ Ba hệ số đầu của khai triển: 0 1 1 2 2


; 2 ; 2


<i>n</i> <i>n</i> <i>n</i>


<i>C</i>  <i>C</i>  <i>C</i> lập thành CSC


Suy ra: <i>C<sub>n</sub></i>022<i>C<sub>n</sub></i>22.21<i>C<sub>n</sub></i>1 (1)
Giải (1) được n =1 và n= 8


+ Với n = 1, hạng tử hữu tỷ khi 1 3


2
<i>k</i>


<i>N</i>


 <sub></sub>


k , khơng có hạng tử hữu tỷ.
+ Với n = 8, hạng tử hữu tỷ khi 8 3



2
<i>k</i>


<i>N</i>


 <sub></sub>


k = 0; 2.
Các hạng tử hữu tỷ là: 0 0 4


82


<i>C</i> <i>x</i> ; <i>C</i><sub>8</sub>222<i>x</i>.


<b>Bài 66. </b> Tìm hệ số lớn nhất của khai triển: (1 2 ) <i>x</i> 12.


<b>Lời giải </b>


Ta có:



12
12


12
0


1 2 <i>k</i>(2 ) ;<i>k</i>


<i>k</i>



<i>x</i> <i>C</i> <i>x</i>




 

hệ số của khai triển là: <i>b<sub>k</sub></i> <i>C</i><sub>12</sub><i>k</i>2 .<i>k</i>
Ta có:


1 1


1 12 12


23


2 2


3


<i>k</i> <i>k</i> <i>k</i> <i>k</i>


<i>k</i> <i>k</i>


<i>b</i> <i>b</i><sub></sub> <i>C</i> <i>C</i>    <i>k</i>
1 1


1 12 12


23


2 2



3


<i>k</i> <i>k</i> <i>k</i> <i>k</i>


<i>k</i> <i>k</i>


<i>b</i> <i>b</i><sub></sub> <i>C</i> <i>C</i>    <i>k</i>
1 2 3 ... 7 8 9 ... 12.


<i>b</i> <i>b</i> <i>b</i> <i>b</i> <i>b</i> <i>b</i> <i>b</i>


        


Vậy hệ số lớn nhất là: 8 8


8 122 126720.


<i>b</i> <i>C</i> 


</div>
<span class='text_page_counter'>(29)</span><div class='page_container' data-page=29>

<b>DAYHOCTOAN.VN </b>


29


<b>DAYHOCTOAN.VN </b>


0 2010 1 2009 2010 2010 0


2011. 2011 2011. 2010 ... 2011. 2011 ... 2011. 1 2011.2



<i>k</i> <i>k</i> <i>x</i>


<i>k</i>


<i>C</i> <i>C</i> <i>C</i> <i>C</i>  <i>C</i> <i>C</i> <sub></sub>  <i>C</i> <i>C</i> 
<b>Lời giải </b>


Xét phương trình:


0 2010 1 2009 2010 2010 0


2011. 2011 2011. 2010 ... 2011. 2011 ... 2011. 1 2011.2


<i>k</i> <i>k</i> <i>x</i>


<i>k</i>


<i>C</i> <i>C</i> <i>C</i> <i>C</i>  <i>C</i> <i>C</i>   <i>C</i> <i>C</i>  (***)


Ta có:




2010


2011 2011 2010


2011 !


2011! 2010!



. . 2011. 2011.


!. 2011 ! 2010 ! !. 2010 !


<i>k</i> <i>k</i> <i>k</i>


<i>k</i>


<i>k</i>


<i>C</i> <i>C</i> <i>C</i>


<i>k</i> <i>k</i> <i>k</i> <i>k</i> <i>k</i>







  


  


Thay vào ta được: (***)

0 1 2010



2010 2010 2010


2011. <i>C</i> <i>C</i> ... <i>C</i> 1011.2<i>x</i>



    


2010


2 2<i>x</i> <i>x</i> 2010


    .


<b>Bài 68. </b> Chứng minh rằng 1 3 5

2 1


2 3 2 5 2 ... 2 1 2 .4


2


<i>n</i> <i>n</i>


<i>n</i> <i>n</i> <i>n</i> <i>n</i>


<i>n</i>


<i>C</i>  <i>C</i>  <i>C</i>   <i>n</i> <i>C</i>   (<i>n</i><i>N</i>, <i>n</i>1).


<b>Lời giải </b>


Ta có:


 

2

2 <sub>1</sub> <sub>3</sub> <sub>5</sub> <sub>2</sub> <sub>1</sub>


2 2 2 2



1<i>x</i> <i>n</i> 1 <i>x</i> <i>n</i> 2(<i>C</i> <i><sub>n</sub></i><i>C</i> <i><sub>n</sub></i><i>C</i> <i><sub>n</sub></i> ... <i>C</i> <i><sub>n</sub>n</i>).
Lấy đạo hàm hai vế:


2 1

2 1


1 3 2 5 4 2 1 2 1


2 2 2 2


2<sub></sub><i>C</i> <i><sub>n</sub></i>3<i>C x<sub>n</sub></i> 5<i>C x<sub>n</sub></i>  ... 2<i>n</i>1 <i>C</i> <i><sub>n</sub>n</i>x <i>n</i> <sub></sub>= <i>2n</i> 1<i>x</i> <i>n</i> 2<i>n</i> 1<i>x</i> <i>n</i>


Thay <i>x</i>1 vào hai vế ta được:




1 3 5 2 1 2 1


2 2 2 2


2<sub></sub><i>C</i> <i><sub>n</sub></i>3<i>C</i> <i><sub>n</sub></i>5<i>C</i> <i><sub>n</sub></i> ... 2<i>n</i>1 <i>C</i> <i><sub>n</sub>n</i> <sub></sub> 2 .2<i>n</i> <i>n</i>



1 3 5 2 1


2 3 2 5 2 ... 2 1 2 .4 .


2


<i>n</i> <i>n</i>



<i>n</i> <i>n</i> <i>n</i> <i>n</i>


<i>n</i>


<i>C</i> <i>C</i> <i>C</i> <i>n</i> <i>C</i> 


      


<b>Bài 69. </b> Chứng minh rằng: 0 ≤ k ≤ n , kN, n N* thì:


2


0 3 1 5 2 2 1 2 3


... 1


2 4 2 2


<i>n</i> <i>n</i> <i>n</i>


<i>n</i>


<i>n</i> <i>n</i> <i>n</i> <i>n</i> <i>n</i> <i>n</i>


<i>C</i>  <i>C</i>  <i>C</i>    <i>C</i>   
<b>Lời giải </b>


Từ khai triển

0 1 2 2


1<i>x</i> <i>n</i> <i>Cn</i> <i>C x C xn</i>.  <i>n</i>  ... <i>C xnn</i>. <i>n</i>



Thay 1


2


<i>x</i> ta được


2


0 1 2


3 1 1 1


. ...


2 2 2 2


<i>n</i> <i>n</i>


<i>n</i>


<i>n</i> <i>n</i> <i>n</i> <i>n</i>


<i>C</i> <i>C x</i> <i>C</i> <i>C</i>


  <sub></sub> <sub></sub> <sub></sub>  <sub> </sub> 


     


      (1)



Ta lại có: 0 1 2


</div>
<span class='text_page_counter'>(30)</span><div class='page_container' data-page=30>

<b>DAYHOCTOAN.VN </b>


30


<b>DAYHOCTOAN.VN </b>


<i>n</i> <i>nn</i>


<i>n</i>
<i>n</i>


<i>n</i>
<i>n</i>


<i>n</i>
<i>n</i>


<i>n</i>
<i>n</i>


<i>C</i>
<i>C</i>


<i>C</i>
<i>C</i>


<i>C</i>



2
1
2
...
4


5
2


3
2


3


2 0 0 1 2


2 











Suy ra điều phải chứng minh.



<b>Bài 70. </b> Tìm số tự nhiên n thỏa mãn: 3 2 3


1 12 6


<i>n</i> <i>n</i> <i>n</i>


<i>A</i> <i>A</i><sub></sub>  <i>C</i>  <i>n</i>,


<b>Lời giải </b>


Điều kiện: <i>n</i>3,<i>n</i>


Phương trình đã cho tương đương:




 





3 2 3


1


1 2


12 6 1 2 1 12 6


3!


<i>n</i> <i>n</i> <i>n</i>


<i>n n</i> <i>n</i>



<i>A</i> <i>A</i><sub></sub>  <i>C</i>  <i>n</i><i>n n</i> <i>n</i> <i>n n</i>     <i>n</i>


2 5


4 5 0


1


<i>n</i>


<i>n</i> <i>n</i>


<i>n</i>





  <sub>   </sub>


 


Đối chiếu với điều kiện ta được n = 5.


<b>Bài 71. </b> Tìm hệ số của 6


<i>x</i> trong khai triển của biểu thức sau thành đa thức:


7 2

9



( ) 1 3 1 2


<i>f x</i> <i>x</i>  <i>x</i> <i>x</i>  <i>x</i>
<b>Lời giải </b>


* Xét khai triển <i>E</i><i>x</i>

1 3 <i>x</i>

7


Số hạng tổng quát:

 

1



7 7


. <i>k</i> 3 <i>k</i> 3 .<i>k</i> <i>k</i>. <i>k</i> 0 7


<i>x C</i> <i>x</i>  <i>C x</i>   <i>k</i> .
Số hạng trên chứa 6


<i>x</i> khi và chỉ khi <i>k</i>   1 6 <i>k</i> 5.
Vậy hệ số của 6


<i>x</i> trong khai triển của E là: 3 .5<i>C</i><sub>7</sub>5.


* Xét khai triển 2

9


1 2


<i>G</i><i>x</i>  <i>x</i> .


Số hạng tổng quát: 2

  

2



9 9



. <i>k</i> 2 <i>k</i> 2 .<i>k</i> <i>k</i>. <i>k</i> 0 9


<i>x C</i>  <i>x</i>   <i>C x</i>   <i>k</i> .
Số hạng trên chứa 6


<i>x</i> khi và chỉ khi <i>k</i>   2 6 <i>k</i> 4.
Vậy hệ số của 6


<i>x</i> trong khai triển của G là:

 

2 .4 <i>C</i><sub>9</sub>4 2 .4<i>C</i><sub>9</sub>4.
Vậy hệ số của 6


<i>x</i> trong khai triển <i>f x</i>

 

là: 3 .C5 <sub>7</sub>52 .4<i>C</i><sub>9</sub>4 7119.


<b>Bài 72. </b> Khi khai triển

2

10


1 <i>x</i> <i>x</i> thành đa thức <i>P x</i>

 

<i>a</i>0<i>a x a x</i>1  2 2<i>a x</i>20 20. Hãy tính


tổng <i>a</i>0  <i>a</i>1 <i>a</i>2 <i>a</i>20.


<b>Lời giải </b>


Từ

 

2 20


0 1 2 20


</div>
<span class='text_page_counter'>(31)</span><div class='page_container' data-page=31>

<b>DAYHOCTOAN.VN </b>


31



<b>DAYHOCTOAN.VN </b>


Thay <i>x</i>1 vào ta được <i>P</i>

 

1 <i>a</i><sub>0</sub> <i>a</i><sub>1</sub> <i>a</i><sub>2</sub><i>a</i><sub>20</sub>
Mà <i>P</i>

 

1   

1 1 12

10310 nên <i>a</i><sub>0</sub>  <i>a</i><sub>1</sub> <i>a</i><sub>2</sub> <i>a</i><sub>20</sub>310.


<b>Bài 73. </b> Tìm hệ số của số hạng chứa 15


<i>x</i> trong khai triển

3



2<i>x</i> 3 <i>n</i> thành đa thức, biết <i>n</i> là số


nguyên dương thỏa mãn 3 1 2


8 49


<i>n</i> <i>n</i> <i>n</i>


<i>A</i> <i>C</i>  <i>C</i>  .
(Trong đó <i>k</i>


<i>n</i>


<i>A</i> là số chỉnh hợp chập <i>k</i> của <i>n </i>phần tử, <i>C<sub>n</sub>k</i> là số tổ hợp chập <i>k</i> của <i>n </i>phần tử).
<b>Lời giải </b>


Điều kiện: <i>n</i> , <i>n</i>3


Ta có 3 1 2

2



8 49 7 7 0 7



<i>n</i> <i>n</i> <i>n</i>


<i>A</i> <i>C</i>  <i>C</i>   <i>n</i> <i>n</i>    <i>n</i>


Khi đó:

3

7 7

 

3 7

 

7

 

7 3 7 


7 7


0 0


2 3 <i>k</i> 2 <i>k</i> 3 <i>k</i> <i>k</i> 1 2<i>k</i> <i>k</i>.3<i>k</i> <i>k</i>


<i>k</i> <i>k</i>


<i>x</i> <i>C</i> <i>x</i>  <i>C</i>  <i>x</i> 


 


 

 



Số hạng chứa 15


<i>x</i> trong khai triển ứng với 3 7

<i>k</i>

15 <i>k</i> 2
Hệ số của số hạng cần tìm là <sub>5</sub>

 

2 <sub>7 2</sub> <sub>2</sub>


7 1 2 .3 6048


<i>C</i>    .



<b>Bài 74. </b> Tìm số hạng chứa 8


<i>x</i>

trong khai triển của nhị thức Newton của 1 <i>x</i>2 1 <i>x</i> 8.


<b>Lời giải </b>


Ta có:


8 8


8


2 2 2


8 8


0 0


1 1 <i>i</i> 1 <i>i</i> <i>i</i> <i>i</i> 1 <i>i</i>


<i>i</i> <i>i</i>


<i>x</i> <i>x</i> <i>C x</i> <i>x</i> <i>C x</i> <i>x</i>


8 8


2 2


8 8



0 0 0 0


1 1


<i>i</i> <i>i</i>


<i>k</i> <i>k</i>


<i>i</i> <i>i</i> <i>k</i> <i>k</i> <i>i</i> <i>k</i> <i>i k</i>


<i>i</i> <i>i</i>


<i>i</i> <i>k</i> <i>i</i> <i>k</i>


<i>C x</i> <i>C</i> <i>x</i> <i>C C</i> <i>x</i>


Để xuất hiện số hạng chứa 8


<i>x</i>

thì:


2 8


0 3


0 8 2


,


<i>i</i> <i>k</i>



<i>k</i> <i>i</i> <i>i</i>


<i>i</i> <i>k</i>


<i>i k</i> <i>N</i>


hoặc 4


0


<i>i</i>
<i>k</i>


Vậy số hạng chứa 8


<i>x</i>

là: <i>C C</i><sub>8</sub>3 <sub>3</sub>2 12<i>x</i>8 <i>C C</i><sub>8</sub>4 <sub>4</sub>0 10<i>x</i>8 238<i>x</i>8.


<b>Bài 75. </b> Chứng minh rằng với <i>n</i> , <i>n</i>3. Ta có:


1 2 3


2 3 ...


!


<i>n</i>


<i>n</i> <i>n</i> <i>n</i> <i>n</i>


<i>C</i> <i>C</i> <i>C</i> <i>nC</i>



<i>n</i>
<i>n</i>


    <sub></sub>


<b>Lời giải </b>


</div>
<span class='text_page_counter'>(32)</span><div class='page_container' data-page=32>

<b>DAYHOCTOAN.VN </b>


32


<b>DAYHOCTOAN.VN </b>


Lấy đạo hàm hai vế:

1 <sub>1</sub> <sub>2</sub> <sub>1</sub>


1 <i>n</i> <i><sub>n</sub></i> 2 <i><sub>n</sub></i> ... <i><sub>n</sub>n</i> <i>n</i>


<i>n</i> <i>x</i>  <i>C</i>  <i>C x</i> <i>nC x</i> 


Cho <i>x</i>1, ta có <i>C<sub>n</sub></i>12<i>C<sub>n</sub></i>2 ... <i>nC<sub>n</sub>n</i> <i>n</i> 2<i>n</i>1


1 2 3


1
2 3 ...


2


<i>n</i>


<i>n</i>


<i>n</i> <i>n</i> <i>n</i> <i>n</i>


<i>C</i> <i>C</i> <i>C</i> <i>nC</i>


<i>n</i>




   


  .


Ta chứng minh <sub>2</sub><i>n</i>1<i><sub>n</sub></i><sub>!</sub><sub>, với </sub><i><sub>n</sub></i>


, <i>n</i>3

 

2 bằng phương pháp qui nạp
+ Kiểm tra

 

2 đúng khi <i>n</i>3.


+ Giả sử

 

2 đúng khi <i>n</i> <i>k</i> 3,<i>k</i> , tức là ta có <sub>2</sub><i>k</i>1<i><sub>k</sub></i><sub>!</sub>


Ta chứng minh

 

2 đúng khi <i>n</i> <i>k</i> 1, ta chứng minh:2<i>k</i> 

<i>k</i>1 !



Vì 2 3   <i>k</i> <i>k</i> 1 nên 1



2<i>k</i>  2 2<i>k</i>   2 <i>k</i>! <i>k</i>1 <i>k</i>! <i>k</i>1 !.
Suy ra điều phải chứng minh.


<b>Bài 76. </b> Chứng minh rằng:

<sub>0</sub>

 

2 <sub>1</sub>

 

2 <sub>2</sub>

2

 

2 1

<sub>2</sub> <sub>1</sub>

2



2 1 2 1 2 1 ... 1 2 1 0


<i>n</i> <i><sub>n</sub></i>


<i>n</i> <i>n</i> <i>n</i> <i>n</i>


<i>C</i> <sub></sub>  <i>C</i> <sub></sub>  <i>C</i> <sub></sub>     <i>C</i> <sub></sub>  .


<b>Lời giải </b>


Xét đẳng thức:


2 1

2 1 2 1 2 1

 



2 1 2 1


1 . 1 . 1


<i>n</i> <i>n</i>


<i>n</i> <i>n</i> <i><sub>k</sub></i> <i><sub>k</sub></i> <i><sub>k</sub></i> <i>k</i> <i><sub>k</sub></i>


<i>n</i> <i>n</i>


<i>k o</i> <i>k o</i>


<i>x</i> <i>x</i> <i>C</i> <i>x</i> <i>C</i> <i>x</i>


 



 


 


 


  


  <sub></sub> <sub></sub>  <sub></sub>






Hệ số của <i><sub>x</sub></i>2<i>n</i>1


trong khai triển là:


<sub>0</sub>

 

2 <sub>1</sub>

 

2 <sub>2</sub>

2

 

2 1

<sub>2</sub> <sub>1</sub>

2


2 1 2 1 2 1 ... 1 2 1


<i>n</i> <i><sub>n</sub></i>


<i>n</i> <i>n</i> <i>n</i> <i>n</i>


<i>C</i> <sub></sub>  <i>C</i> <sub></sub>  <i>C</i> <sub></sub>     <i>C</i> <sub></sub>


Ta lại có:


2 1

2 1

<sub>2</sub>

2 1
1 <i>n</i> . 1 <i>n</i> 1 <i>n</i>

<i>x</i>  <i>x</i>   <i>x</i>  


Hệ số của 2<i>n</i> 1


<i>x</i>  bằng 0 vì khai triển chỉ chứa lũy thừa bậc chẵn của x vậy ta có:

<sub>0</sub>

 

2 <sub>1</sub>

 

2 <sub>2</sub>

2

 

2 1

<sub>2</sub> <sub>1</sub>

2


2 1 2 1 2 1 ... 1 2 1 0


<i>n</i> <i><sub>n</sub></i>


<i>n</i> <i>n</i> <i>n</i> <i>n</i>


<i>C</i> <sub></sub>  <i>C</i> <sub></sub>  <i>C</i> <sub></sub>     <i>C</i> <sub></sub>  .


<b>Bài 77. </b> Cho số nguyên dương n. Chứng minh rằng:


1 2 1


2009 2010 2009+n


1 1 1 1


+ +...+ <


C C C<i>n</i> 2007


<b>Lời giải </b>


Ta có: <sub>1</sub>



2009


1 2008!( 1)! 2008!( 1)!


(2009 ( 2))


(2009 )! (2009 )!2007


<i>k</i>
<i>k</i>


<i>k</i> <i>k</i>


<i>k</i> <i>k</i>


<i>C</i>  <sub></sub> <i>k</i> <i>k</i>


 


    


</div>
<span class='text_page_counter'>(33)</span><div class='page_container' data-page=33>

<b>DAYHOCTOAN.VN </b>


33


<b>DAYHOCTOAN.VN </b>


2008 2007!( 1)! 2007!( 2)!



2007 (2008 )! (2009 )!


<i>k</i> <i>k</i>


<i>k</i> <i>k</i>


   


 <sub></sub>  <sub></sub>


 


  1 2


2008 2009


2008 1 1


2007 <i>Ck</i> <sub></sub><i><sub>k</sub></i> <i>Ck</i> <sub></sub><i><sub>k</sub></i>


 


 <sub></sub>  <sub></sub>


 


Lấy tổng các số hạng trên ta được:


1 1 2 1



0 2009 2008 2009 2008


1 2008 1 1 2008 1 1


.


2007 2007 2007


<i>n</i>


<i>k</i> <i>n</i>


<i>k</i> <i>C</i> <i>k</i> <i>C</i> <i>C</i> <i>n</i> <i>C</i>


 
  
 
 <sub></sub>  <sub></sub> 
 

.


<b>Bài 78. </b> Tìm hệ số của số hạng không chứa <i>x</i> trong khai triển nhị thức Niu-tơn của


3 2
3

1

<i>n</i>

<i>x x</i>


<i>x</i>







biết rằng tổng các hệ số của các số hạng trong khai triển này là


0 1 2 ... <i>n</i> 4096


<i>a</i>  <i>a</i> <i>a</i>  <i>a</i>  .


<b>Lời giải </b>


Ta có:


 


5


3 2 <sub>3</sub> <sub>3</sub>


3


0


1


( )



<i>n</i> <i><sub>k</sub></i> <i><sub>n k</sub></i>


<i>n</i>
<i>k</i>
<i>k</i>



<i>f x</i>

<i>x x</i>

<i>a x</i>

<i>x</i>



<i>x</i>






<sub></sub>

<sub></sub>




12


0


(1) 1 1 2 4096 2 12


<i>n</i>


<i>n</i> <i><sub>n</sub></i>


<i>k</i>
<i>k</i>


<i>f</i> <i>a</i> <i>n</i>




       .



Suy ra:


 


12


12 <sub>12</sub> 1 5 <sub>12</sub> 5 12 <sub>12</sub> 6 60


3 2 <sub>3</sub> <sub>3</sub> <sub>3</sub> <sub>3</sub> <sub>3</sub>


12 12 12


3


0 0 0


1

<i>k</i> <i>k</i> <i>k</i> <i>k</i> <i>k</i> <i>k</i> <i>k</i> <i>k</i>


<i>k</i> <i>k</i> <i>k</i>


<i>x x</i>

<i>C</i>

<i>x</i>

<i>x</i>

<i>C x</i>

<i>C x</i>



<i>x</i>


 <sub></sub> <sub></sub> <sub> </sub>
 
  

  


<sub></sub>

<sub></sub>

<sub></sub>

<sub></sub>


  




  



Số hạng không chứa <i>x</i> ứng với  6<i>k</i> 60 0  <i>k</i> 10.
Vậy số hạng khơng chứa <i>x</i> có hệ số là: <i>C</i><sub>12</sub>1066.


<b>Bài 79. </b> Tìm hệ số của số hạng chứa 8


<i>x</i> trong khai triển nhị thức Niutơn của


2
5
3
1 <i>n</i>
<i>x</i>
<i>x</i>
 <sub></sub> 
 


  , biết


rằng 1 1 2 2 3 3


3<i>n</i> 2 3<i>n</i> 3 3<i>n</i> ... <i>n</i> 6144


<i>n</i> <i>n</i> <i>n</i> <i>n</i>


<i>C</i>   <i>C</i>   <i>C</i>   <i>nC</i>  (*)


(n nguyên dương, <i>x</i>0, <i>C<sub>n</sub>k</i> là số tổ hợp chập k của n phần tử)



<b>Lời giải </b>


Ta có:

3<i>x</i>

<i>n</i> <i>C<sub>n</sub></i>03<i>n</i><i>C<sub>n</sub></i>13<i>n</i>1<i>x C</i> <i><sub>n</sub></i>23<i>n</i>2<i>x</i>2<i>C<sub>n</sub></i>33<i>n</i>3<i>x</i>3 ... <i>C x<sub>n</sub>n</i> <i>n</i> (1)
Lấy đạo hàm hai vế của (1) ta được


1 <sub>1</sub> <sub>1</sub> <sub>2</sub> <sub>2</sub> <sub>3</sub> <sub>3</sub> <sub>2</sub> <sub>1</sub>


3 <i>n</i> <i><sub>n</sub></i>3<i>n</i> 2 <i><sub>n</sub></i>3<i>n</i> 3 <i><sub>n</sub></i>3<i>n</i> ... <i><sub>n</sub>n</i> <i>n</i>


<i>n</i> <i>x</i>  <i>C</i>   <i>C</i>  <i>x</i> <i>C</i>  <i>x</i>  <i>nC x</i>  (2)


1


<i>x</i> vào (*) ta có


1 1 2 2 3 3 1


.3<i>n</i> 2 .3<i>n</i> 3 .3<i>n</i> ... <i>n</i> 4<i>n</i>


<i>n</i> <i>n</i> <i>n</i> <i>n</i>


<i>C</i>   <i>C</i>   <i>C</i>   <i>nC</i> <i>n</i> 

 

1


</div>
<span class='text_page_counter'>(34)</span><div class='page_container' data-page=34>

<b>DAYHOCTOAN.VN </b>


34


<b>DAYHOCTOAN.VN </b>



Với <i>n</i>6 thì (3) thỏa


Với <i>n</i><i>N</i> và <i>n</i>7 thì <i>n</i>.4<i>n</i>17.46 6144 nên (3) khơng thỏa
Suy ra <i>n</i>6là nghiệm duy nhất của (*)


 



2 12 12 12 12 11 72


5 5 5 2


12 12


3 3 3


0 0


1 <i>n</i> 1 1 <i>k</i> <i>k</i> <i>k</i>


<i>k</i> <i>k</i>


<i>k</i> <i>k</i>


<i>x</i> <i>x</i> <i>C</i> <i>x</i> <i>C x</i>


<i>x</i> <i>x</i> <i>x</i>


 


 



 <sub></sub>  <sub></sub> <sub></sub>  <sub></sub>   <sub></sub>


     


   

 



<i>k</i><i>Z k</i>, 0,1,...,12

thỏa 11 72 8 8
2


<i>k</i>


<i>k</i>


 <sub>  </sub>


.
Vậy hệ số của 8


<i>x</i> là <i>C</i><sub>12</sub>8 495.


<b>Bài 80. </b> Tính tổng tất cả các hệ số có lũy thừa bậc lẻ của đa thức P(x) trong khai triển:


 

<sub>2</sub>

10


1 4 5


<i>P x</i>   <i>x</i> <i>x</i>


<b>Lời giải </b>



Ta có: <i>P x</i>

 

 

1 4<i>x</i>5<i>x</i>2

10

 

20 19


20 19 ... 1 0


<i>P x</i> <i>a x</i> <i>a x</i>  <i>a x a</i> . Tổng cần tính là: <i>S</i> <i>a</i><sub>19</sub><i>a</i><sub>17</sub> ... <i>a</i><sub>1</sub>


Ta có:

 

10


1 20 19 ... 1 0 1 10


<i>S</i> <i>a</i> <i>a</i>   <i>a</i> <i>a</i> <i>P</i>  (1)


Mặt khác:

 

10


2 20 19 ... 1 0 1 2


<i>S</i> <i>a</i> <i>a</i>   <i>a</i> <i>a</i> <i>P</i>   (2)


Lấy (1) trừ (2) được: 10 10


2<i>S</i> 10 2


Vậy: 1

10 10



10 2
2


<i>S</i>   .



<b>Bài 81. </b> Chứng minh rằng: Nếu <i>n k</i>,  thì:


 

2


2 . 2 2


<i>n</i> <i>n</i> <i>n</i>


<i>n k</i> <i>n k</i> <i>n</i>


<i>C</i> <sub></sub> <i>C</i> <sub></sub>  <i>C</i> <b>. </b>
<b>Lời giải </b>


Cố định <i>n</i>, với 0 <i>k</i> <i>n</i>, xét dãy số

 

<i>uk</i>


Ta có <sub>2</sub> . <sub>2</sub> (2 )! (2. )!


!( )! !( )!


<i>n</i> <i>n</i>


<i>k</i> <i>n k</i> <i>n k</i>


<i>n k</i> <i>n k</i>


<i>u</i>


<i>n n k</i> <i>n n k</i>



<i>c</i>

<i>c</i>



 


 


 


1 2 1 2 1


(2 1)! (2 1)!


. .


!( 1)! !( 1)!


<i>n</i> <i>n</i>


<i>k</i> <i>n k</i> <i>n k</i>


<i>n</i> <i>k</i> <i>n k</i>


<i>u</i>


<i>n n</i> <i>k</i> <i>n n k</i>


<i>c</i>

<i>c</i>



 <sub> </sub> <sub> </sub>



   


 


   










1 2 1


1
1 2


<i>k</i>
<i>k</i>


<i>n k</i> <i>n k</i>
<i>u</i>


<i>u</i> <i>n k</i> <i>n k</i>


   


  


  


2<i>n k</i> 1



<i>n k</i>

 

<i>n k</i> 1 2



<i>n k</i>




       


2<i>nk</i> <i>n</i> 0


   đúng vì 0 <i>k</i> <i>n</i>.


Vậy

 

<i>uk</i> là dãy số giảm.


Suy ra với <i>k</i>0 ta có <i>C</i><sub>2</sub><i>n<sub>n k</sub></i><sub></sub> .<i>C</i><sub>2</sub><i>n<sub>n k</sub></i><sub></sub> 

 

<i>C</i><sub>2</sub><i>n<sub>n</sub></i> 2 (đpcm).
<b>Bài 82. </b> Chứng minh rằng:

 





1 2 <sub>1</sub>


2.4.6...2


1 ...


3 5 2 1 1.3.5... 2 1


<i>n</i> <i>n</i>


<i>n</i>


<i>n</i> <i>n</i> <i>C</i>


<i>C</i> <i>C</i> <i>n</i>



<i>n</i> <i>n</i>




    


  ,  <i>n</i> .


</div>
<span class='text_page_counter'>(35)</span><div class='page_container' data-page=35>

<b>DAYHOCTOAN.VN </b>


35


<b>DAYHOCTOAN.VN </b>


Ta có

1<i>x</i>2

<i>n</i>  1 <i>C x<sub>n</sub></i>1 2<i>C x<sub>n</sub></i>2 4  ...

 

1 <i>nC x<sub>n</sub>n</i> 2<i>n</i>, <i>n</i> .


1 2 2 4

 

2



1 1


2


0 0


1 ...


1 <i>n</i> <i>n</i> 1<i>n</i> <i>n</i> <i>n</i>


<i>n</i>



<i>n</i>


<i>C x</i> <i>C x</i> <i>C x</i>


<i>x</i> <i>dx</i>


 

    


 

<sub> </sub>



1 2 1


1 1


1 ... 1


3 5 2 1


<i>n</i> <i><sub>n</sub></i>
<i>n</i>
<i>n</i> <i>n</i>
<i>C</i>
<i>C</i> <i>C</i>
<i>n</i>

    


 Tính In=


đặt cos , 0;



2


<i>x</i> <i>t t</i>  <sub></sub>


  <i>dx</i> sin<i>tdt</i>


Đặt
2
sin
sin
<i>n</i>
<i>u</i> <i>t</i>
<i>dv</i> <i>tdt</i>
 



2
2 1
0
sin <i>n</i>
<i>n</i>
<i>I</i> <i>tdt</i>



 

2 2 2 1


0



2 cos .sin <i>n</i>


<i>n</i>


<i>I</i> <i>n</i> <i>t</i> <i>tdt</i>






 





2


2 2 1


0


2<i>n</i> 1 sin <i>t</i> .sin <i>n</i> <i>tdt</i>






<sub></sub>

 2<i>nIn</i>12<i>nIn</i> 1

0


2 2.4...2



2 1 3.5... 2 1


<i>n</i> <i>n</i>


<i>n</i> <i>n</i>


<i>I</i> <i>I</i> <i>I</i>


<i>n</i>  <i>n</i>


  


 


Vậy




2.4...2


3.5... 2 1


<i>n</i>


<i>n</i>
<i>I</i>


<i>n</i>





 (2)


Từ (1) và (2) đpcm


<b>Bài 83. </b> Tìm hệ số không chứa <i>x</i> trong khai triển


20
3
1
2 <i>x</i>
<i>x</i>
 <sub></sub> 
 
  .
<b>Lời giải </b>
Ta có
20


20 <sub>1</sub> <sub>1</sub>


3
2
3


1


2 <i>x</i> 2<i>x</i> <i>x</i>



<i>x</i>

 
 <sub></sub>  <sub></sub> <sub></sub>
 
 
   
20 <sub>1</sub>
1
20
3
2
20
0
2
<i>k</i>
<i>k</i>
<i>k</i>
<i>k</i>


<i>C</i> <i>x</i> <i>x</i>


 <sub></sub>

 
 
 <sub></sub> <sub></sub> <sub></sub> <sub></sub>
   


20 60 5


20 <sub>6</sub>
20
0
2
<i>k</i>
<i>k</i> <i>k</i>
<i>k</i>
<i>C</i> <i>x</i>





Với <i>k</i><i>Z</i>, 0 <i>k</i> 20


Hệ số không chứa <i>x</i> thì 60 5 <i>k</i>0 nên <i>k</i>12 ( T/m)
Hệ số cần tìm là. 12 8


20.2 32248320


<i>C</i>  .


<b>Bài 84. </b> Tìm tất cả các số nguyên dơng n thoả mãn: 1 0 1 1 ...

 

1 1


2 3 2 42


<i>n</i>
<i>n</i>



<i>n</i> <i>n</i> <i>n</i>


<i>C</i> <i>C</i> <i>C</i>


<i>n</i>




   


 .


<b>Lời giải </b>


Với mỗi n nguyên dơng, xét tích phân



1


0


1 <i>n</i>


<i>I</i> 

<i>x</i> <i>x</i> <i>dx</i>.


Đặt 1 <i>x</i> <i>t</i> <i>dx</i> <i>dt</i>. Khi <i>x</i>  0 <i>t</i> 1 và khi <i>x</i>  1 <i>t</i> 0.
Vậy


0 1 1 2


1



1 0


1 1 1


(1 ) ( )


0


1 2 1 2


<i>n</i> <i>n</i>


<i>n</i> <i>n</i> <i>n</i> <i>t</i> <i>t</i>


<i>I</i> <i>t t dt</i> <i>t</i> <i>t</i> <i>dt</i>


<i>n</i> <i>n</i> <i>n</i> <i>n</i>


 


  


     <sub></sub>  <sub></sub>  


   


 


(1)


Mặt khác (1<i>x</i> <i>x</i>)<i>n</i> 

0 1 ...

 

1 <i>n</i> <i>n</i> <i>n</i>



<i>n</i> <i>n</i> <i>n</i>


<i>x C</i> <i>C x</i> <i>C x</i>


     0 1 2

 

1


... 1 <i>n</i> <i>n</i> <i>n</i>


<i>n</i> <i>n</i> <i>n</i>


<i>C x C x</i> <i>C x</i> 


    


1 


0
2


)
1


( <i>x</i> <i>ndx</i>


</div>
<span class='text_page_counter'>(36)</span><div class='page_container' data-page=36>

<b>DAYHOCTOAN.VN </b>


36



<b>DAYHOCTOAN.VN </b>


Vậy



1


0 1 2 1 0 2 1 3 2


0


1


1 1 ( 1)


... ( 1) ...


0


2 3 2


<i>n</i>


<i>n</i> <i>n</i> <i>n</i> <i>n</i> <i>n</i>


<i>n</i> <i>n</i> <i>n</i> <i>n</i> <i>n</i> <i>n</i>


<i>I</i> <i>C x C x</i> <i>C x</i> <i>dx</i> <i>C x</i> <i>C x</i> <i>C x</i>


<i>n</i>


    
     <sub></sub>    <sub></sub>

 



= (2)


Từ (1) và (2), suy ra 1 0 1 1

 

1 1 1


...


2 3 2 1 2


<i>n</i>
<i>n</i>


<i>n</i> <i>n</i> <i>n</i>


<i>C</i> <i>C</i> <i>C</i>


<i>n</i> <i>n</i> <i>n</i>




    


   .


Vậy n thoả mãn điều kiện đã cho khi và chỉ khi 1 1 1

 

3


1 2 42
<i>n</i> <i>n</i>  .


Giải phương trình (3) được nghiệm nguyên dương <i>n</i>5. KL: <i>n</i>5.


<b>Bài 85. </b> Tìm<i><b> số hạng </b></i>không<i><b> chứa x trong khai triển</b></i> 3 2
,
<i>n</i>
<i>x</i>
<i>x</i>
 <sub></sub> 
 
 


Điều kiện: <i>n</i>12,<i>n</i><i>N</i> Ta có
Điều kiện: <i>n</i>12,<i>n</i><i>N</i> Ta có


11
1
10
2
9
2
9
2
8
2
8
2


7
2
11
1
10
2
9
2
8
2
7


2 3  3      2  2    


  <i>n</i>  <i>n</i>  <i>n</i>  <i>n</i>  <i>n</i>  <i>n</i>  <i>n</i>  <i>n</i>  <i>n</i>  <i>n</i>  <i>n</i>


<i>n</i> <i>C</i> <i>C</i> <i>C</i> <i>C</i> <i>C</i> <i>C</i> <i>C</i> <i>C</i> <i>C</i> <i>C</i> <i>C</i>


<i>C</i>
 11
1
10
1
9
1
9
1
8
1
11


1
10
1
9
1
8


1 2        


  <i>n</i>  <i>n</i>  <i>n</i>  <i>n</i>  <i>n</i>  <i>n</i>  <i>n</i>  <i>n</i>


<i>n</i> <i>C</i> <i>C</i> <i>C</i> <i>C</i> <i>C</i> <i>C</i> <i>C</i> <i>C</i>


<i>C</i>


9 10 11 10 11


1 1 1 20


<i>n</i> <i>n</i> <i>n</i> <i>n</i> <i>n</i>


<i>C</i> <i>C</i> <i>C</i><sub></sub> <i>C</i><sub></sub> <i>C</i><sub></sub> <i>n</i>


      


Với <i>n</i>20 ta có

 

 



20 <sub>20</sub> <sub>20</sub>


20 40 5



3 <sub>3</sub> 2 <sub>6</sub>


20 20


0 0


2


.2 . .2


<i>k</i>


<i>k</i> <i>k</i>


<i>k</i> <i>k</i> <i>k</i> <i>k</i>


<i>k</i> <i>k</i>


<i>x</i> <i>C</i> <i>x</i> <i>x</i> <i>C</i> <i>x</i>


<i>x</i>

 
 
 <sub></sub>  <sub></sub> <sub></sub>
 
 



Số hạng không hạng không chứa<i> x</i> ứng với số tự nhiên <i>k</i>: 40 5 <i>k</i>  0 <i>k</i> 8


Vậy số hạng không chứa <i>x</i> là 28<i>C</i><sub>20</sub>8


<b>Bài 86. </b> Tìm hệ số của số hạng chứa 13


<i>x</i> trong khai triển Newton của đa thức



3


3
2


1


( ) 2 1


4


<i>n</i>


<i>f x</i> <sub></sub>  <i>x</i> <i>x</i> <sub></sub> <i>x</i>


  với <i>n</i> là số tự nhiên thỏa mãn:


3 2


14
<i>n</i>


<i>n</i> <i>n</i>



<i>A</i> <i>C</i>   <i>n</i>


<b>Lời giải</b>


Từ 3 2


14
<i>n</i>


<i>n</i> <i>n</i>


<i>A</i> <i>C</i>   <i>n</i> suy ra 2<i>n</i>25<i>n</i>250 ta được <i>n</i>5


Ta có



3


3 3 6 21


2


1 1 1


( ) 2 1 2 1 2 1


4 64 64


<i>n</i> <i>n</i>



<i>f x</i> <sub></sub>  <i>x</i> <i>x</i> <sub></sub> <i>x</i>  <i>x</i>   <i>x</i>


 


Vậy hệ số 13 13


13 21


1
2
64


<i>a</i>  <i>C</i> hay <i>a</i><sub>13</sub><i>C</i>13<sub>21</sub>27


<b>Bài 87. </b> Giải phương trình sau: 1 2 3 10


... 1023


<i>x</i> <i>x</i> <i>x</i> <i>x</i>


<i>x</i> <i>x</i> <i>x</i> <i>x</i>


<i>C</i>  <i>C</i>  <i>C</i>   <i>C</i>  


<b>Lời giải</b>


Điều kiện <i>x</i> ,<i>x</i>10


1 2 3 10



... 1023


<i>x</i> <i>x</i> <i>x</i> <i>x</i>


<i>x</i> <i>x</i> <i>x</i> <i>x</i>


<i>C</i>  <i>C</i>  <i>C</i>   <i>C</i>  


1 2 10


.... 1024


<i>x</i> <i>x</i> <i>x</i> <i>x</i>


<i>x</i> <i>x</i> <i>x</i> <i>x</i>


<i>C</i> <i>C</i>  <i>C</i>  <i>C</i> 


     


n


0 1 n


n n n


1 1 ( 1)


C C ... C



2 3 n 2




  


</div>
<span class='text_page_counter'>(37)</span><div class='page_container' data-page=37>

<b>DAYHOCTOAN.VN </b>


37


<b>DAYHOCTOAN.VN </b>


0 1 2 10


10


... 1024


2 2


10


<i>x</i> <i>x</i> <i>x</i> <i>x</i>


<i>x</i>


<i>C</i> <i>C</i> <i>C</i> <i>C</i>


<i>x</i>



     


 


 


<b>Bài 88. </b>


1. Tính <i>S</i><i>C</i><sub>2009</sub>1 2.2008<i>C</i><sub>2009</sub>2 3.20082<i>C</i><sub>2009</sub>3  ... 2009.20082008<i>C</i><sub>2009</sub>2009


2. Chứng tỏ S chia hết cho 2009.


<b>Lời giải</b>


1) Ta có: (1<i>x</i>)2009 <i>C</i><sub>2009</sub>0 <i>C</i><sub>2009</sub>1 <i>x C</i> <sub>2009</sub>2 <i>x</i>2 ... <i>C</i><sub>2009</sub>2009<i>x</i>2009 (1)
Lấy đạo hàm cấp 1 hai vế của (1) ta được:


2008 1 2 2008 2009


2009 2009 2009


2009(1<i>x</i>) <i>C</i> 2<i>xC</i>  ... 2009<i>x</i> <i>C</i> (2)


Thay <i>x</i>2008 vào (2) ta được <i>S</i> 2009.2009200820092009


2) Vì 2009 chia hết 2009 nên <i>S</i> chia hết 2009.


<b>Bài 89. </b> Từ tập hợp tất cả các số tự nhiên có năm chữ số mà các chữ số đều khác 0, lấy ngẫu
nhiên một số. Tính xác suất để trong số tự nhiên được lấy ra chỉ có mặt ba chữ số khác nhau.



<b>Lời giải</b>


Ta có:  95 59.049 Gọi <i>A</i> là biến cố cần tìm xác suất, ta có:


Số cách chọn 3 chữ số phân biệt <i>a b c</i>, , từ 9 chữ số thập phân khác 0 là C . Chọn 2 chữ 3<sub>9</sub>
số còn lại từ 3 chữ số đó, có 2 trường hợp rời nhau sau đây:


<b>TH1.</b> Cả 2 chữ số còn lại cùng bằng 1 trong 3 chữ số <i>a, b, c</i>: có 3 cách; mỗi hốn vị từ
5! hốn vị của 5 chữ số (chẳng hạn) <i>a</i>, <i>a</i>, <i>a</i>, <i>b</i>, <i>c</i> tạo ra một số tự nhiên <i>n</i>; nhưng cứ 3!


hốn vị của các vị trí mà <i>a</i>, <i>a</i>, <i>a</i> chiếm chỗ thì chỉ tạo ra cùng một số <i>n</i>, nên trong TH1
này có cả thảy 3 5! 60


3!


  số tự nhiên.


<b>TH2. </b>1 trong 2 chữ số còn lại bằng 1 trong 3 chữ số <i>a, b, c</i> và chữ số kia bằng 1 chữ số
khác trong 3 chữ số đó: có 3 cách; mỗi hoán vị từ 5! hoán vị của 5 chữ số (chẳng hạn)


<i>a</i>, <i>a</i>, <i>b</i>, <i>b</i>, <i>c</i> tạo ra một số tự nhiên <i>n</i>; nhưng cứ 2! hoán vị của các vị trí mà <i>a</i>, <i>a</i> chiếm
chỗ và 2! hốn vị của các vị trí mà <i>b</i>, <i>b</i> chiếm chỗ thì chỉ tạo ra cùng một số <i>n</i>, nên trong


TH2 này có cả thảy 3 5! 90


2!2!


  số tự nhiên.


Vậy: 3



9


9!


(60 90)C 150 150.7.4.3 12600
3!6!


<i>A</i>


       .


Kết luận:

 

12.600 1.400 0,213382106


59.049 6.561


<i>A</i>


<i>P A</i>     




<b>Bài 90. </b> Gọi <i>A</i> là tập hợp tất cả các số tự nhiên có 5 chữ số. Chọn ngẫu nhiên một số từ tập <i>A</i>,
tính xác suất để chọn được một số chia hết cho 7 và chữ số hàng đơn vị bằng 1.


<b>Lời giải</b>


Số các số tự nhiên có 5 chữ số là 99999 10000 1 90000  


Giả sử số tự nhiên có 5 chữ số chia hết cho 7 và chữ số hàng đơn vị bằng 1 là: <i>abcd</i>1



Ta có <i>abcd</i>1 10. <i>abcd</i> 1 3.<i>abcd</i>7.<i>abcd</i>1 chia hết cho 7 khi và chỉ khi 3.<i>abcd</i>1


</div>
<span class='text_page_counter'>(38)</span><div class='page_container' data-page=38>

<b>DAYHOCTOAN.VN </b>


38


<b>DAYHOCTOAN.VN </b>


Đặt 3. 1 7 2 1


3
<i>h</i>


<i>abcd</i>  <i>h</i><i>abcd</i>  <i>h</i>  là số nguyên khi và chỉ khi <i>h</i> 3<i>t</i> 1
Khi đó ta được: <i>abcd</i>   7<i>t</i> 2 1000  7<i>t</i> 2 9999




998 9997


143, 144,..., 1428


7 <i>t</i> 7 <i>t</i>


    


Suy ra số cách chọn ra <i>t</i> sao cho số <i>abcd</i>1 chia hết cho 7 và chữ số hàng đơn vị bằng 1
là 1286.



Vậy xác suất cần tìm là: 1286 0, 015
90000


<b>Bài 91. </b> Gọi <i>A</i> là tập hợp các số tự nhiên có chín chữ số đơi một khác nhau. Chọn ngẫu nhiên
một số tự nhiên thuộc vào tập <i>A</i><b>. </b>Tính xác suất để chọn được một số thuộc <i>A</i> và số đó chia hết
cho <i>3</i>.


<b>Lời giải</b>


+) Trước hết ta tính <i>n A</i>

 

.


Với số tự nhiên có chín chữ số đơi một khác nhau thì chữ số đầu tiên có 9 cách chọn và
có <i>A</i><sub>9</sub>8 cho 8 vị trí cịn lại. Vậy <i>n A</i>

 

9<i>A</i><sub>9</sub>8


+) Giả sử <i>B</i>

0;1; 2;...;9

ta thấy tổng các phần tử của B bằng 45 3 nên số có chín chữ
số đơi một khác nhau và chia hết cho 3 sẽ được tạo thành từ 9 chữ số của các tập


 

 

 

 



\ 0 ; \ 3 ; \ 6 ; \ 9


<i>B</i> <i>B</i> <i>B</i> <i>B</i> nên số các số loại này là <i>A</i><sub>9</sub>93.8.<i>A</i><sub>8</sub>8.


Vậy xác suất cần tìm là


9 8


9 8


8


9


3.8. 11


9. 27


<i>A</i> <i>A</i>


<i>A</i>






<b>Bài 92. </b> Trong mặt phẳng cho đa giác đều 2<i>n</i> đỉnh <i>A A</i>1 2...<i>A</i>2<i>n</i> (với <i>n</i> là số nguyên lớn hơn 1).
Hỏi có tất cả bao nhiêu hình chữ nhật với các đỉnh là đỉnh của đa giác đều đã cho.


<b>Lời giải</b>


Gọi

 

<i>O</i> là đường tròn ngoại tiếp đa giác đều <i>A A</i><sub>1</sub> <sub>2</sub>...<i>A</i><sub>2</sub><i><sub>n</sub></i>. Dễ thấy đa giác này có đúng
<i>n</i><sub> đường chéo mà là đường kính của </sub>

 

<i>O</i> . Mặt khác, mỗi tứ giác có đỉnh là đỉnh của đa


giác là một hình chữ nhật khi và chỉ khi hai đường chéo của nó là hai đường kính của


 

<i>O</i> .


Bên cạnh đó, hai tứ giác khác nhau thì có hai cặp đường chéo khác nhau. Do vậy số hình
chữ nhật có đỉnh là đỉnh của đa giác bằng với số cặp đừờng chéo của đa giác đều mà là
hai đường kính.



Vậy số các hình chữ nhật cần tìm là 2 ( 1)


2


<i>n</i>


<i>n n</i>
<i>C</i>   .


<b>Bài 93. </b> Từ một tập thể 14 người gồm 6 nam và 8 nữ trong đó có An và Bình, người ta muốn
chọn một tổ công tác gồm 6 người. Tìm số cách chọn trong trường hợp sau: Trong tổ có một tổ
trưởng, năm tổ viên và An, Bình khơng đồng thời có mặt trong tổ”


<b>Lời giải. </b>


Có ba trường hợp xảy ra.


</div>
<span class='text_page_counter'>(39)</span><div class='page_container' data-page=39>

<b>DAYHOCTOAN.VN </b>


39


<b>DAYHOCTOAN.VN </b>


Số cách chọn tổ viên: 5
11
<i>C</i> .


Số cách chọn là 5


11



12.<i>C</i> .


TH2: Trong tổ có An và khơng có Bình.


Nếu An là tổ trưởng thì số cách chọn năm tổ viên trong 12 người còn lại là 5
12
<i>C</i> .
Nếu An là tổ viên thì số cách chọn một tổ trưởng và bốn tổ viên trong 12 người còn lại


là 4


11


12.<i>C</i> .


Số cách chọn trong TH2 là 5 4


12 12. 11
<i>C</i>  <i>C</i> .


TH3: Trong tổ có Bình và khơng có An. Tương tự TH2.


Vậy số cách chọn thỏa mãn yêu cầu đề bài là: 5

5 4



11 12 11


12.<i>C</i> 2 <i>C</i> 12.<i>C</i> 15048 (cách).


<b>Bài 94. </b> Cho tập hợp <i>X</i> 

1, 2....,8

. Có thể thành lập được bao nhiêu số tự nhiên lẻ có 5 chữ

số khác nhau từ các chữ số thuộc tập <i>X</i> trong đó ln có mặt chữ số 1.


<b>Lời giải. </b>


Gọi số cần tìm là <i>a a a a a</i>1 2 3 4 5.


TH1 : <i>a</i><sub>5</sub> 1.


Chọn 4số cịn lại có 4
7
<i>A</i> .


Số các chữ số đước thành lập trong trường hợp này là : 4
7
<i>A</i> .
TH2 : <i>a</i>5 1.


5


<i>a</i> có 3 cách chọn.


Có 4 cách chọn vị trí cho số 1 và <i>A</i><sub>6</sub>3 cách chọn cho các vị trí cịn lại.
Vậy số các chữ số đước thành lập trong TH này là 3


6


3.4.<i>A</i> .


Số các chữ số thỏa mãn yêu cầu bài toán là: 4 3



7 3.4. 6 2280


<i>A</i>  <i>A</i>  .


<b>Bài 95. </b> Có bao nhiêu số tự nhiên gồm 6 chữ số khác nhau trong đó có 3 số chẵn và 3 số lẻ ?


<b>Lời giải. </b>


Gọi số được thành lập là <i>a a a a a a</i>1 2 3 4 5 6


TH1: Trong 3 số chẵn đó có mặt số 0 .
Số các số tìm được là 2 3


4 5


5.C .C .5! 36000 (số).
TH2: Trong 3 số chẵn đó khơng có mặt số 0.
Số các số tìm được là 3 3


4 5


C .C .6! 28800 (số).


Số các số thỏa mãn yêu cầu đề bài là: 36000 28800 64800  số.


<b>Bài 96. </b> Cho 10 thí sinh ngồi quanh một bàn trịn. Ngân hàng đề thi có 10 loại đề khác nhau, mỗi
loại đề có nhiều đề khác nhau. Một cách phát đề gọi là hợp lệ nếu mỗi thí sinh nhận được một loại
đề và hai thí sinh ngồi cạnh nhau khơng nhận được cùng một loại đề. Hỏi có bao nhiêu cách phát
đề hợp lệ ?



<b>Lời giải. </b>


Gọi là số cách phát đề hợp lệ cho thí sinh .


Ta viết nếu và cùng nhận được một loại đề và trong


trường hợp ngược lại.


Xét một cách phát đề hợp lệ cho thí sinh .


<i>n</i>


<i>s</i> <i>n</i> <i>a a</i><sub>1</sub>, <sub>2</sub>,...,<i>a<sub>n</sub></i>


( )


<i>i</i> <i>j</i>


<i>a</i> <i>a i</i> <i>j</i> <i>a<sub>i</sub></i> <i>a<sub>j</sub></i> <i>a<sub>i</sub></i> <i>a<sub>j</sub></i>


</div>
<span class='text_page_counter'>(40)</span><div class='page_container' data-page=40>

<b>DAYHOCTOAN.VN </b>


40


<b>DAYHOCTOAN.VN </b>


- Nếu thì bỏ đi thí sinh ta được một cách phát đề hợp lệ cho thí sinh
. Khi đó có 10-2=8 cách phát đề cho thí sinh (khác với 2 đề của ).


- Nếu thì bỏ đi hai thí sinh ta được một cách phát đề hợp lệ cho



thí sinh . Khi đó có 10-1=9 cách phát đề hợp lệ cho (cụ thể


, còn phát 1 trong 9 đề khác ).
Như vậy ta có hệ thức sau


.


Mặt khác, dễ tính được : .


Do đó tính được .


<b>Bài 97. </b> Nếu một số được chọn ngẫu nhiên từ một tập hợp gồm 5 chữ số trong đó tổng các chữ
số bằng 43. Tính xác xuất để số được chọn chia hết cho 11.


<b>Lời giải. </b>


Gọi số được thành lập là <i>a a a a a</i><sub>1 2 3 4 5</sub>


Trong cơ số 10, chữ số lớn nhất là 9 nên tổng lớn nhất <i>a</i>1<i>a</i>2 <i>a</i>3 <i>a</i>4<i>a</i>5 45 .


Theo giả thiết, tổng của các chữ số trong số được chọn là 43 45 2  nên có thể xảy ra
các trường hợp sau:


TH1: Một chữ số là 7 , tất cả các chữ số còn lại đều bằng 9 là


79999, 97999, 99799, 99979, 99997 có 5 số như vậy.


TH2: Hai chữ số là 8 , tất cả các chữ số còn lại đều bằng 9 , có 5.4 10



2  số như vậy.


Vậy có 15 số có tổng bằng 43.


Để số được chọn chia hết cho 11 thì cần và đủ là: <i>a</i>1   <i>a</i>2 <i>a</i>3 <i>a</i>4 <i>a</i>5 11


Có 3 số thỏa mãn: 97999;99979;98989


Vậy xác suất chọn được số thỏa mãn yêu cầu bài toán là: 3 1


15 5.


<b>Bài 98. </b> Cho các số 1, 2, 3, 4. Hỏi lập đợc bao nhiêu số có 5 chữ số trong đó có hai chữ số 1 và
ba chữ số còn lại khác nhau và khác số 1. Mỗi số có 5 chữ số gồm 2 số 1 và 3 số khác là hoán vị
5 phần tử 1,1,2,3,4


<b>Lời giải. </b>


do 2 số 1 khi hoán vị vẫn đợc 1 số vậy các số cần lập là 5
2


60


<i>P</i>
<i>P</i> 


<b>Bài 99. </b> Một hộp đựng 16 viên bi, trong đó có 5 viên bi màu đỏ đôi một khác nhau, 5 viên bi
màu xanh đôi một khác nhau nhau và 6 viên bi màu vàng đôi một khác nhau. Lấy ngẫu nhiên từ
hộp đã cho 7 viên bi. Tính xác suất để lấy được 7 viên bi lấy ra có đủ ba loại màu?



<b>Lời giải. </b>


Kí hiệu  là khơng gian mẫu, <i>A</i> là biến cố lấy ra được 7 viên bi đủ ba loại màu.
Khi đó <i>n</i>

 

 là số cách lấy ra 7 viên bi bất kì từ hộp gồm 16 viên bi   <i>n</i>

 

<i>C</i>167 .


Số cách lấy ra 7 viên bi từ một trong hai loại đỏ và xanh là 7
10


<i>C</i> , số cách lấy ra 7 viên
1 <i>n</i>


<i>a</i> <i>a</i> <i>an</i>1 <i>n</i>


1, 2,..., <i>n</i>


<i>a a</i> <i>a</i> <i>a<sub>n</sub></i><sub></sub><sub>1</sub> <i>a a</i><sub>1,</sub> <i><sub>n</sub></i>


1 <i>n</i>


<i>a</i> <i>a</i> <i>a a<sub>n</sub></i>, <i><sub>n</sub></i><sub></sub><sub>1</sub> (<i>n</i>1)


1, 2,..., <i>n</i> 1


<i>a a</i> <i>a</i><sub></sub> <i>a a<sub>n</sub></i>, <i><sub>n</sub></i><sub></sub><sub>1</sub>


1


<i>n</i>


<i>a</i> <i>a</i> <i>a<sub>n</sub></i><sub></sub><sub>1</sub> <i>a</i><sub>1</sub>



1 8 9 1, 2


<i>n</i> <i>n</i> <i>n</i>


<i>s</i><sub></sub>  <i>s</i>  <sub></sub>  <i>n</i>


2 10.9 90, 3 10.9.8 720


<i>s</i>   <i>s</i>  


</div>
<span class='text_page_counter'>(41)</span><div class='page_container' data-page=41>

<b>DAYHOCTOAN.VN </b>


41


<b>DAYHOCTOAN.VN </b>


bi từ một trong hai loại xanh và vàng là 7
11


<i>C</i> , số cách lấy ra 7 viên bi từ một trong hai


loại đỏ và vàng là 7
11
<i>C</i>


suy ra <i>n A</i>

 

<i>C</i><sub>10</sub>7 <i>C</i><sub>11</sub>7 <i>C</i><sub>11</sub>7 <i>n A</i>

 

<i>C</i><sub>16</sub>7 

<i>C</i><sub>10</sub>7 <i>C</i><sub>11</sub>7 <i>C</i><sub>11</sub>7

.


Do đó

 

 




 

4441


<i>n A</i>
<i>P A</i>


<i>n</i>


 


 . Vậy xác suất cần tìm là

 



41
44
<i>P A</i>  .


<b>Bài 100. </b> Trên mặt phẳng cho đa giác lồi 10 cạnh Xét các tam giác có 3 đỉnh là 3
đỉnh của đa giác . Hỏi trong số các tam giác đó có bao nhiêu tam giác mà 3 cạnh của nó đều
khơng phải là ba cạnh của đa giác ?


<b>Lời giải. </b>


+) Số tam giác phân biệt có 3 đỉnh là 3 trong các đỉnh của đa giác là


+) ứng với mỗi cạnh của đa giác sẽ có 8 cách chọn các đỉnh còn lại để tạo thành
một tam giác chứa cạnh này. Suy ra số tam giác có ít nhất một cạnh là cạnh của đa
giác T là 80 (tam giác).


+) Trong 80 tam giác trên có 10 tam giác có 2 cạnh là 2 cạnh của đa giác T đợc lặp lại
hai lần.



+) Kết luận: Số các tam giác cần tìm là (120 – 80) + 10 = 50 (tam giác)


<b>Bài 101. </b> Với các chữ số

0,1, 2....,8,9

có thể lập được bao nhiêu số tự nhiên có sáu chữ số đôi
một khác nhau sao cho các chữ số 1, 2,3 đứng kề nhau.


<b>Lời giải. </b>


Từ 10 chữ số đã cho ta lập được 3
7


<i>C</i> bộ gồm 6 chữ số khác nhau, trong đó ln có mặt


các chữ số 1, 2,3.


Từ mỗi bộ như thế lập được 4! 3! số có 6 chữ số khác nhau trong đó các chữ số 1, 2, 3
ln đứng kề nhau (với quy ước tính cả các số mà có chữ số 0 đứng đầu). Vậy có 4! 3!


3
7


<i>C</i> 5040(số).


Trong 5040 số được tạo thành có 3!.3!<i>C</i>62540 (số) gồm 6 chữ số khác nhau mà chữ


số 0 đứng đầu và các chữ sô 1, 2,3 luôn đứng kề nhau.
Vậy có 5040 540 4500  (số cần tìm).


<b>Bài 102. </b> Có hai hộp chứa các quả cầu. Hộp thứ nhất chứa 3 quả cầu đỏ, 7 quả cầu trắng và 10
quả cầu xanh. Hộp thứ hai chứa 5 quả cầu đỏ, 7 quả cầu trắng và 8 quả cầu xanh. Từ mỗi hộp
lấy ngẫu nhiên một quả cầu. Tính xác suất để hai quả cầu lấy ra có cùng một màu.



<b>Lời giải. </b>


Gọi <i>A</i> là biến cố quả cầu lấy ra từ hộp thứ nhất là màu đỏ

 

3


20
<i>P A</i>  .
Gọi <i>B</i> là biến cố quả cầu lấy ra từ hộp thứ hai là màu đỏ

 

5


20
<i>P B</i>  .


<i>AB</i> là biến cố lấy ra hai quả cầu màu đỏ từ hai hộp; ,<i>A B</i>là hai biến cố đôc lập, áp


dụng công thức nhân xác suất ta có

 

   

. 15


400
<i>P AB</i> <i>P A P B</i>  .


.
... 10
2
1<i>A</i> <i>A</i>
<i>A</i>
<i>T</i> 
<i>T</i>


<i>T</i>


<i>T</i> <i>C</i><sub>10</sub>3 120



</div>
<span class='text_page_counter'>(42)</span><div class='page_container' data-page=42>

<b>DAYHOCTOAN.VN </b>


42


<b>DAYHOCTOAN.VN </b>


Tương tự ta tính được xác suất để lấy được hai quả cầu màu trắng là


400
49


, xác suất để
lấy được hai quả cầu màu xanh là


400
80


.


Vậy xác suất để lấy được hai quả cầu cùng màu thỏa mãn bài toán là


15 49 80 9
400400400 25 .


<b>Bài 103. </b> Với các chữ số

1, 2...., 7

xét tập hợp <i>E</i> gồm các số có 7 chữ số khác nhau viết từ các
số đã cho. Chứng minh rằng tổng <i>S</i> tất cả các số của tập <i>E</i> chia hết cho 9 .


<b>Lời giải. </b>



Từ tập <i>E</i> ta có thể lập được 7! số gồm 7 chữ số khác nhau.


Nhận xét rằng các số 1, 2....,7 xuất hiện ở các hàng đơn vị, hàng chục, hàng trăm, hàng
ngàn … 720 lần, từ đó




6 0


10 .720 1 2 3 4 5 6 7 ... 10 .720 1 2 3 4 5 6 7


<i>S</i>              


6 0


720.28.(10 ... 10 ) 9


  


Vì 7209.


<b>Bài 104. </b> Trong một lớp học có 8 bóng đèn, mỗi bóng có xác suất bị cháy là 1


4 . Lớp học có đủ
ánh sáng nếu có ít nhất 4 bóng sáng. Tính xác suất để lớp học đủ ánh sáng.


<b>Lời giải </b>


Gọi <i>A</i>, <i>B</i>, <i>C</i>, <i>D</i>, <i>E</i> tương ứng là các biến cố “Lớp có 8 bóng đèn sáng”, “Lớp có
7 bóng đèn sáng”, “Lớp có 6 bóng đèn sáng”, “Lớp có 5 bóng đèn sáng”, “Lớp có 4



bóng đèn sáng”. Mỗi bóng đèn có xác sáng là 3


4 . Theo quy tắc nhân và quy tắc cộng,
ta có


8


3
( )


4


<i>P A</i>    


 


7


3 1


( ) .


4 4


<i>P B</i>    


 


6 2



3 1


( ) .


4 4


<i>P C</i>        


   


5 3


3 1


( )


4 4


<i>P D</i>        


   


4 4


3 1


( )


4 4



<i>P E</i>        


   


Biến cố <i>A</i>   <i>B</i> <i>C</i> <i>D</i> <i>E</i> chính là biến cố “Lớp học có đủ ánh sáng”. Vì <i>A</i>, <i>B</i>, <i>C</i>,


<i>D</i>, <i>E</i> rời nhau đôi một nên ta có


( ) ( ) ( ) ( ) ( ) ( )


</div>
<span class='text_page_counter'>(43)</span><div class='page_container' data-page=43>

<b>DAYHOCTOAN.VN </b>


43


<b>DAYHOCTOAN.VN </b>


<b>Bài 105. </b> Có bao nhiêu số tự nhiên gồm 9 chữ số biết rằng chữ số 2 có mặt đúng hai lần, chữ số 3
có mặt đúng ba lần, các chữ số cịn lại có mặt khơng q một lần.


<b>Lời giải </b>


+) Nếu số cần tìm chứa 0, thì có 8 cách chọn vị trí cho 0, có 2
8


<i>C</i> cách chọn vị trí cho
hai chữ số 2, có 3


6



<i>C</i> cách chọn vị trí cho ba chữ số 3, và có <i>A</i><sub>7</sub>3 cách chọn ba chữ số
cho ba vị trí cịn lại. Do đó trờng hợp này có 2 3 3


8 6 7


8.<i>C C A</i>. .  940800số. +) Nếu số cần
tìm khơng chứa 0, thì có 2


9


<i>C</i> cách chọn vị trí cho hai chữ số 2, có <i>C</i><sub>7</sub>3 cách chọn vị trí
cho ba chữ số 3, và có 4


7


<i>A</i> cách chọn bốn chữ số cho bốn vị trí cịn lại. Do đó trờng


hợp này có 2 3 4


9. 7 7


<i>C C A</i> = 1058400 số Vậy có 940800 + 1058400 = 1.999.200 số thỏa
mãn yêu cầu


<b>Bài 106. </b> Đội thanh niên xung kích của trường phổ thơng có 12 học sinh, gồm 5 học sinh lớp <i>A</i>,
4 học sinh lớp <i>B</i>, 3 học sinh lớp <i>C</i>. Cần chọn 4 học sinh đi làm nhiệm vụ, sao cho 4 học sinh này


thuộc không quá 2 lớp


Ta dung cách đếm bù, loại đi số cách chọn đủ 3 lớp:


-2 hs lớp A,1 lớp B, 1 lớp C


-2 hs lớp B,1 lớp A, 1 lớp C
-2 hs lớp C,1 lớp B, 1 lớp A
Đs 225


<b>Bài 107. </b> Từ các chữ số 1, 3 , 4, 8 lập các số tự nhiên có sáu chữ số, trong đó chữ số 3 có mặt
đúng ba lần, các chữ số cịn lại có mặt đúng một lần. Trong các số được tạo thành nói trên, chọn
ngẫu nhiên một số. Tính xác suất để số được chọn chia hết cho 4?


<b>Lời giải </b>


Gọi số cần tìm là <i>abcdef</i> với <i>a b c d e f</i>, , , , , 

1,3, 4,8


Sắp xếp chữ số 3 vào 3 trong 6 vị trí, có 3


6


<i>C</i> cách. Sắp xếp 3 chữ số 1; 4; 8 vào 3
vị trí cịn lại có 3! Cách. Vậy có tất cả 3


6.3! 120


<i>C</i>  số. Một số chia hết cho 4 khi và
chỉ khi hai chữ số tận cùng tạo thành 1số chia hết cho 4.


Trong các số trên, số lấy chia hết cho 4 có tận cùng là 48 , 84 . Trong mỗi trường hợp
có <i>C</i><sub>4</sub>34cách sắp xếp chữ số 3 và 1 vào 4 vị trí cịn lại, suy ra có 8 số chia hết cho


4. Gọi <i>A</i> là biến cố: “ Số lấy ra chia hết cho 4”



Vậy số các kết quả thuận lợi cho <i>A</i> là  <i><sub>A</sub></i> 8 Số phần tử của không gian mẫu là
120


  Xác suất của biến cố <i>A</i> là 8 1


120 15


<i>A</i>
<i>A</i>


<i>P</i>    




<b>Bài 108. </b> Người ta sử dụng ba loại sách gồm: 8 cuốn sách về Toán học, 6 cuốn sách về Vật lý
và 5 cuốn sách về Hoá học. Mỗi loại đều gồm các cuốn sách đơi một khác loại nhau. Có bao nhiêu
cách chọn 7 cuốn sách trong số sách trên để làm giải thưởng sao cho mỗi loại có ít nhất một cuốn?


</div>
<span class='text_page_counter'>(44)</span><div class='page_container' data-page=44>

<b>DAYHOCTOAN.VN </b>


44


<b>DAYHOCTOAN.VN </b>


Sử dụng cách tính gián tiếp: Số cách chọn 7 trong số 19 cuốn sách một cách bất kỳ là


7
19
<i>C</i>



Số cách chọn không đủ cả ba loại sách là:


. Số cách chọn 7 trong số 11 cuốn sách Lý và Hoá là <i>C</i><sub>11</sub>7 (khơng có sách Tốn). Số
cách chọn 7 trong số 13 cuốn sách Hoá và Toán là 7


13


<i>C</i> (khơng có sách Lý). Số cách
chọn 7 trong số 14 cuốn sách Toán và Lý là <i>C</i><sub>14</sub>7 (khơng có sách Hố). Số cách chọn


7 trong số 8 cuốn sách Toán là <i>C</i><sub>8</sub>7 (khơng có sách Lý và Hố). Vì mỗi cách chọn
khơng có sách Lý và Hố thuộc cả hai phép chọn: Khơng có sách Lý và khơng có sách


Hố. Nên số cách phải tìm là: 7 7 7 7 7


19 11 13 14 8 44918


<i>C</i> <i>C</i> <i>C</i> <i>C</i> <i>C</i>  cách


<b>Bài 109. </b> Từ 8 chữ số 0 , 1, 2, 3 , 4, 5 , 6 , 7 có thể lập được bao nhiêu số gồm 6 chữ số khác
nhau trong đó nhất thiết phải có mặt chữ số 4.


<b>Lời giải </b>


Xét 2 trường hợp


TH1: Chữ số đầu là số 4. Khi đó 5 chữ số đằng sau có <i>A</i><sub>7</sub>5 cách chọn


TH2: chữ số 4 khơng đứng ở vị trí đầu. Khi đó có 5 vị trí cho số 4. Chữ số đầu có 6



cách chọn và 4 chữ số cịn lại có <i>A</i><sub>6</sub>4 cách chọn. Vậy TH2 có 5.6.<i>A</i><sub>6</sub>4cách chọn.


Vật số các số tự nhiên TMYC đầu bài là: 5 4


7 5.6. 6 13320


<i>A</i>  <i>A</i> 


<b>Bài 110. </b> Một tổ học sinh có 6 nam và 5 nữ. Tìm xác suất để chọn ra 4 học sinh đi lao động sao
cho trong đó có khơng q 2 nữ.


<b>Lời giải </b>


Phép thử: ‘‘lấy ngẫu nhiên 4 sinh viên trong tổ” <i>n</i>

 

 <i>C</i>114 330


Gọi A: ‘‘Lấy 4 HS trong đó có khơng q 2 nữ”
Có 3 trường hợp xảy ra


TH1: Lấy cả 4 HS nam có <i>C</i><sub>6</sub>4 cách chọn


TH2: Lấy 4 HS trong đó có 1 nữ và 3 nam có <i>C C</i><sub>5</sub>1. <sub>6</sub>3 cách
TH3: Lấy 4 HS trong đó có 2nữ và 2 nam có <i>C C</i><sub>5</sub>2. <sub>6</sub>2 cách


 

4 1 3 2 2
6 5. 6 5. 6 265
<i>n A</i> <i>C</i> <i>C C</i> <i>C C</i> 


Vậy xác suất xuất hiện biến cố A là:

 

 



 

330265 5366


<i>n A</i>
<i>P A</i>


<i>n</i>


  




<b>Bài 111. </b> Từ các số 1, 2, 3 , 4, 5 có thể lập được bao nhiêu số tự nhiên có năm chữ số, trong đó
chữ số 3 có mặt đúng ba lần, các chữ số cịn lại có mặt không quá một lần. Trong các số tự nhiên
nói trên, chọn ngẫu nhiên một số, tìm xác suất để số được chọn chia hết cho 3 .


</div>
<span class='text_page_counter'>(45)</span><div class='page_container' data-page=45>

<b>DAYHOCTOAN.VN </b>


45


<b>DAYHOCTOAN.VN </b>


<b>Lời giải </b>


*) Gọi <i>a a a a a</i>1 2 3 4 5 là số tự nhiên có năm chữ số, trong đó chữ số 3 có mặt đúng ba lần,


các chữ số cịn lại có mặt khơng q một lần với <i>a a a a a</i>1, 2, 3, 4, 5

1;2;3;4;5



Sắp chữ số 3 vào 3 vị trí, có 3
5 10


<i>C</i>  cách



Cịn lại 2 vị trí, 4 chữ số. Chọn 2 chữ số xếp vào 2 vị trí đó, có <i>C</i><sub>4</sub>2 12 cách
Vậy khơng gian mẫu có 10.12 120 phần tử


*) Có

1 5 3; 2 4 3

 



Gọi biến cố <i>A</i>: “số được chọn chia hết cho 3” có 2 phương án
2 chữ số cịn lại là 1, 5 có <i>C</i><sub>5</sub>3.2! 20 số


2 chữ số cịn lại là 2, 4 có <i>C</i><sub>5</sub>3.2! 20 số
Vậy biến cố <i>A</i> có 40 phần tử.


*) Xác suất của biến cố <i>A</i> là 40 1


120 3


<i>P</i> 


<b>Bài 112. </b> Số 16200 có bao nhiêu ước nguyên dương.
Ta có: 162002 .3 .53 4 2


Ước của 16200 co dạng: 2 .3 .5<i>m</i> <i>n</i> <i>p</i>

<i>m n p</i>, ,  ;0 <i>m</i> 3, 0 <i>n</i> 4, 0 <i>p</i> 2


+ Với mỗi bộ số

<i>m n p</i>, ,

ta có 1 ước số tự nhiên của 16200.


+ Chọn <i>m</i>: có 4 cách.


<i>n</i>: có 5 cách. Suy ra: có 4.5.360 (bộ số(<i>m, n, p</i>))


<i>p</i>: có 3 cách.



Vậy có 60 ớc số cần tìm.


<b>Bài 113. </b> Một hộp đựng 11 tấm thẻ ghi số từ 1 đến 11. Rút ngẫu nhiên trong hộp ra 6 tấm thẻ.
Tính xác suất để tổng số ghi trên 6 tấm thẻ rút ra là 1 số lẻ


Ta có : 6


11
<i>C</i>


 


Để tổng số ghi trên 6 tấm thẻ rút ra là 1 số lẻ thì số tấm thẻ ghi số lẻ rút ra phải là 1 số
lẻ


TH1 : 1 thẻ ghi số lẻ 5 thẻ ghi số chẵn : có <i>C C</i><sub>6</sub>1. <sub>5</sub>5 cách
TH2 : 3 thẻ ghi số lẻ 3 thẻ ghi số chẵn : có 3 3


6. 5


<i>C C</i> cách


TH3 : 5 thẻ ghi số lẻ 1 thẻ ghi số chẵn : có 5 1
6. 5
<i>C C</i> cách


 Có 1 5


6. 5



<i>C C</i> + 3 3
6. 5


<i>C C</i> + 5 1
6. 5
<i>C C</i> cách


Gọi A là biến cố ‘tổng số ghi trên 6 tấm thẻ rút ra là 1 số lẻ’


 

16 55 63 53 65 51
6


11


.<i>C C</i> <i>C C</i>. <i>C C</i>.


<i>P A</i>


<i>C</i>


 


</div>
<span class='text_page_counter'>(46)</span><div class='page_container' data-page=46>

<b>DAYHOCTOAN.VN </b>


46


<b>DAYHOCTOAN.VN </b>


<b>Bài 114. </b> Có bao nhiêu số tự nhiên gồm 9 chữ số biết rằng chữ số 2 có mặt đúng hai lần, chữ số 3
có mặt đúng ba lần, các chữ số cịn lại có mặt khơng q một lần.



+) Nếu số cần tìm chứa 0, thì có 8 cách chọn vị trí cho 0, có 2
8


<i>C</i> cách chọn vị trí cho
hai chữ số 2, có 3


6


<i>C</i> cách chọn vị trí cho ba chữ số 3, và có 3
7


<i>A</i> cách chọn ba chữ số cho
ba vị trí cịn lại. Do đó trờng hợp này có 2 3 3


8 6 7


8.<i>C C A</i>. . 940800 số.
+) Nếu số cần tìm khơng chứa 0, thì có 2


9


<i>C</i> cách chọn vị trí cho hai chữ số 2, có <i>C</i><sub>7</sub>3
cách chọn vị trí cho ba chữ số 3, và có 4


7


<i>A</i> cách chọn bốn chữ số cho bốn vị trí cịn lại.
Do đó trường hợp này có 2 3 4



9. 7 7 1058400
<i>C C A</i>  số.


Vậy có 940800 + 1058400 = 1.999.200 số thỏa mãn yêu cầu


<b>Bài 115. </b> Xếp 6 học sinh nam và 4 học sinh nữ vào một bàn trịn 10 ghế.Tính xác suất để khơng
có hai học sinh nữ ngồi cạnh nhau. Cố định một vị trí cho một học sinh nữ,đánh số các ghế còn lại
từ 1 đến 9.Số phần tử của khơng gian mẫu là: 9!


A là biến cố “Khơng có hai học sinh nữ ngồi cạnh nhau “.Xét các trường hợp:


*) 1 hs nữ ngồi ghế 2,1hs nữ ngồi ghế 4, hs nữ còn lại ngồi ghế 6 hoặc 7 hoặc 8,có 3.3!
cách.


*) 1 hs nữ ngồi ghế 2,1hs nữ ngồi ghế 5, hs nữ còn lại ngồi ghế 7 hoặc 8,có 2.3! cách.
*) 1 hs nữ ngồi ghế 2,1hs nữ ngồi ghế 6, hs nữ cịn lại ngồi ghế 8,có 3! cách.


*) 1 hs nữ ngồi ghế 3,1hs nữ ngồi ghế 5, hs nữ cịn lại ngồi ghế 7 hoặc 8,có 2.3! cách.
*) 1 hs nữ ngồi ghế 3,1hs nữ ngồi ghế 6, hs nữ cịn lại ngồi ghế 8,có 3! cách.


*) 1 hs nữ ngồi ghế 4,1hs nữ ngồi ghế 6, hs nữ cịn lại ngồi ở ghế 8,có 3! cách. (1,5
điểm)


Hoán vị 6 học sinh nam.




6! 3.3! 2.3! 3! 2.3! 3! 3! 10.3!.6!
<i>A</i>



        


Vậy

 

10.3!.6! 5


9! 42


<i>P A</i>   .


<b>Bài 116. </b> Gọi <i>M</i> là tập tất cả các số tự nhiên có sáu chữ số đơi một khác nhau và có dạng


1 2 3 4 5 6


<i>a a a a a a</i> . Chọn ngẫu nhiên một số từ tập <i>M</i>. Tính xác suất để số được chọn là một số chẵn,


đồng thời thỏa mãn <i>a</i><sub>1</sub><i>a</i><sub>2</sub> <i>a</i><sub>3</sub> <i>a</i><sub>4</sub> <i>a</i><sub>5</sub> <i>a</i><sub>6</sub>.


Gọi <i>A</i> là biến cố “chọn ra được một số tự nhiên chẵn từ tập <i>M</i> đồng thời thỏa mãn


1 2 3 4 5 6


<i>a</i> <i>a</i> <i>a</i> <i>a</i> <i>a</i> <i>a</i> ”. Khi đó: <i>n M</i>

 

9.<i>A</i><sub>9</sub>5 (số có sáu chữ số đơi một khác nhau
thì <i>a</i><sub>1</sub> có chín cách chọn, <i>a a a a a</i><sub>2</sub> <sub>3</sub> <sub>4</sub> <sub>5</sub> <sub>6</sub> là chỉnh hợp chập 5 của 9 phần tử nên có 5


9
<i>A</i> ).
TH1: <i>a</i>6 0thì <i>a a a a a</i>1 2 3 4 5 có


5
9



<i>C</i> cách chọn.
TH2: <i>a</i>6 2thì <i>a a a a a</i>1 2 3 4 5 có


5
7


<i>C</i> cách chọn.
TH3: <i>a</i>6 4thì <i>a a a a a</i>1 2 3 4 5 có


5
5


<i>C</i> cách chọn.


 

5 5 5


9 7 5 148


<i>n A</i> <i>C</i> <i>C</i> <i>C</i>  Do đó

 

 



 

5
9


148 37


9. 34020


<i>n A</i>
<i>P A</i>



<i>n</i> <i>A</i>


  


</div>
<span class='text_page_counter'>(47)</span><div class='page_container' data-page=47>

<b>DAYHOCTOAN.VN </b>


47


<b>DAYHOCTOAN.VN </b>


<b>Bài 117. </b> Một hộp đựng 10 viên bi đỏ có bán kính khác nhau, 5 viên bi xanh có bán kính khác
nhau và 3 viên bi vàng có bán kính khác nhau. Hỏi có bao nhiêu cách chọn ra 9 viên bi có đủ ba
màu. Số cách chọn 9 viên bi tùy ý là: 9


18
<i>C</i>


Những trường hợp không có đủ ba viên bi khác màu là:


+ Khơng có bi đỏ: Khả năng này khơng xảy ra vì tổng các viên bi xanh và vàng chỉ là
8.


+ Khơng có bi xanh: có <i>C</i><sub>13</sub>9 cách.
+ Khơng có bi vàng: có <i>C</i><sub>15</sub>9 cách.


Mặt khác trong các cách chọn khơng có bi xanh, khơng có bi vàng thì có 9
10


<i>C</i> cách chọn
9 viên bi đỏ được tính hai lần.



Vậy số cách chọn 9 viên bi có đủ cả ba màu là: 9 9 9 9


10 18 13 15 42910


<i>C</i> <i>C</i> <i>C</i> <i>C</i>  cách.


<b>Bài 118. </b> Hỏi số 16200 có bao nhiêu ước số tự nhiên?
Ta có: 162002 .3 .53 4 2


Ước của 16200 cú dạng:




2 .3 .5<i>m</i> <i>n</i> <i>p</i> <i>m n p</i>, ,  ;0 <i>m</i> 3, 0 <i>n</i> 4, 0 <i>p</i> 2


+ Với mỗi bộ số (<i>m, n, p</i>) ta có 1 ước số tự nhiên của 16200.
+ Chọn <i>m</i>: có 4 cách.<i> n</i>: có 5 cách. <i>p</i>: có 3 cách.


Suy ra: có 4.5.360 (bộ số(<i>m, n, p</i>)
Vậy có 60 ớc số cần tìm.


<b>Bài 119. </b> Cho tập hợp <i>A</i>

0,1, 2,3, 4,5, 6

.Có thể lập được bao nhiêu số gồm 5 chữ số khác nhau
được lấy ra từ tập A sao cho 3 số lẻ không đứng liền nhau.


Giả sử <i>a a a a a</i><sub>1 2 3 4 5</sub> là số cần tìm.Ta tính tất cả các số gồm 5 chữ số sao cho ln có mặt
3 chữ số lẻ,sau đó trừ đi trường hợp mà 3 số lẻ đứng liền nhau.


*) Tất cả có 3 số lẻ, xếp 3 số lẻ vào 3 trong 5 vị trí, tacó: 3
5 60



<i>A</i>  cách.


Khi đó cịn lại 2 vị trí có thể chọn tuỳ ý trong 4 số chẵn,ta có 2
4 12


<i>A</i>  cách.


Vậy có: 60*12720 (số).


Trong các số trên trừ trường hợp <i>a</i>10:


Nếu <i>a</i>10 thì xếp 3 số lẻ vào 3 trong 4 vị trí, cịn lại 1 vị trí chọn trong 3số chẵn {2,4,6}


ta có <i>A A</i><sub>4</sub>3. <sub>3</sub>172 (số).


Vậy tất cả có: 720 72 648 (số) gồm 5 chữ số sao cho ln có mặt 3 chữ số lẻ<b>. ( 0,5)</b>


*) Tính các số có 5 chữ số sao cho có 3 số lẻ đứng liền nhau.
- Nếu <i>a a a</i><sub>1 2</sub> <sub>3</sub> là 3 số lẻ ta có 3


3 6


<i>A</i>  (cách xếp). Khi đó 2 vị trí cịn lại <i>a a</i><sub>4</sub> <sub>5</sub>có thể chọn


tuỳ ý trong 4 số chẵn, ta có: 2
4 12


<i>A</i>  . Vậy có: 6.1272 (số).
-Nếu <i>a a a</i>2 3 4 là 3 số lẻ ta có



3
3 6


<i>A</i>  (cách xếp). Khi đó:<i>a</i>1có 3 cách chọn(<i>a</i>1 0); <i>a</i>5có


</div>
<span class='text_page_counter'>(48)</span><div class='page_container' data-page=48>

<b>DAYHOCTOAN.VN </b>


48


<b>DAYHOCTOAN.VN </b>


Vậy tất cả có: 72 2.54 180  số có 3 số lẻ đứng liền nhau.


<b>Bài 120. </b> Có thể lập bao nhiêu số tự nhiên có 5 chữ số mà trong mỗi số đó chữ số liền sau ln
lớn hơn chữ số liền trước. Ta biết rằng với mỗi bộ gồm 5 chữ số khác nhau thì chỉ cố một cách
duy nhất tạo thầnh một số có 5 chữ số mà trong mỗi số đó chữ số liền sau luôn lớn hơn chữ số liền
trước.Chứ số tự nhiên cần tìm khơng thể có mặt số 0 vì số 0 khơng thể đứng đầu


Do đó số các số cần tìm là số cách chọn 5 chữ số khác nhau trong 9 só khác 0 đã cho,
đó là số tổ hợp chập 5 của 9


Vậy số các số là: 5
9 126
<i>C</i> 


<b>Bài 121. </b> Trong mặt phẳng cho *


2<i>n</i>1 (<i>n</i> ) đường thẳng phân biệt sao cho khơng có hai đường
nào song song hoặc vng góc và khơng có ba đường nào đồng quy. Chúng cắt nhau tạo thành các


tam giác. Chứng minh rằng số các tam giác nhọn tạo thành không vượt quá

1 2



1



6


<i>n n</i> <i>n</i>


.


Gọi số tam giác tạo thành là <i>f n</i>

 

. Ta phải chứng minh


 

1 2



1

  

*


1 ,
6


<i>n n</i> <i>n</i>


<i>f n</i>     <i>n</i>


Với ba đường thẳng bất kỳ trong số các đường thẳng đã cho luôn cắt nhau tạo thành một
tam giác hoặc nhọn hoặc tù.


Gọi <i>g n</i>

 

là số các tam giác tù. Ta gọi một tam giác tạo bởi ba đường thẳng , ,<i>a b c</i> nào
đó là: "giả nhọn cạnh <i>a</i>" nếu các góc chung cạnh <i>a</i> của tam giác đó là các góc nhọn.
Chọn một đường thẳng <i>d</i> nào đó và coi nó là trục hồnh, các đường thẳng còn lại được
chia làm hai tập: Tập <i>T</i> là các đường thẳng với hệ số góc dương, Tập <i>T</i> là tập các
đường thẳng với hệ số góc âm. Hai đường thẳng tạo với d một tam giác "giả nhọn" nếu
một đường thẳng thuộc tập <i>T</i> và một đường thẳng thuộc tập <i>T</i>.


Gọi <i>p</i> là số đường thẳng thuộc <i>T</i> và <i>q</i> là số các đường thẳng thuộc tập <i>T</i>. Khi đó



2


<i>p</i> <i>q</i> <i>n</i> và số tam giác "giả nhọn cạnh <i>d</i>" là <i>pq</i>. Ta có 2
2


<i>p</i> <i>q</i>
<i>pq</i><sub></sub>  <sub></sub><i>n</i>


 


Nhưng do <i>d</i> có thể là đường thẳng bất kỳ trong số 2<i>n</i>1 đường thẳng đã cho nên ta có
số cặp (đường thẳng <i>d</i>; tam giác "giả nhọn cạnh d") sẽ nhỏ hơn hoặc bằng <i>n</i>2

2<i>n</i>1

.
Trong cách tính trên mỗi tam giác nhọn được tính 3 lần (theo 3 cạnh) cịn mỗi tam giác
tù được tính 1 lần nên


   

2



3<i>f n</i> <i>g n</i> <i>n</i> 2<i>n</i>1 (1)
Thế nhưng tổng số các tam giác là:


 

  

 


3


2 1


2 1 2 2 1


(2)
6



<i>n</i>


<i>n</i> <i>n</i> <i>n</i>


<i>C</i> <sub></sub>  <i>f n</i> <i>g n</i>   


Từ (1) và (2) suy ra


 

2

 

 

2 (2 1)2

2 1



2 2 1 (2 1)


6


<i>n</i> <i>n</i> <i>n</i>


</div>
<span class='text_page_counter'>(49)</span><div class='page_container' data-page=49>

<b>DAYHOCTOAN.VN </b>


49


<b>DAYHOCTOAN.VN </b>
( 1)(2 1)


3
<i>n n</i> <i>n</i>


hay

 

1 2



1




6


<i>n n</i> <i>n</i>


<i>f n</i>   


<b>Bài 122. </b> Từ 6 chữ số 0; 1; 2; 3; 4; 5 lập được bao nhiêu số tự nhiên gồm 6 chữ số sao cho số 0
xuất hiện đúng 3 lần, các số cịn lại xuất hiện khơng q một lần Gọi số có 5 chữ số là <i>a a a a a a</i>1 2 3 4 5 6


Có <i>C</i><sub>5</sub>3 cách xếp sếp 3 số 0


Có <i>A</i><sub>5</sub>3 cách sắp xếp các vị trí cịn lại Vậy có <i>C A</i><sub>5</sub>3. <sub>5</sub>3600cách


<b>Bài 123. </b> Trong một hộp bi có 3 viên bi đỏ, 4 viên bi vàng, 5 viên bi xanh; lấy ngẫu nhiên
4 viên bi trong hộp. Tính xác suất để trong 4 viên bi được lấy số bi đỏ lớn hơn số
bi xanh. Tổng số viên bi trong hộp là: 3 + 4 +5 = 12 viên bi


Lấy ngẫu nhiên 4 viên bi trong hộp ta có số cách lấy là: 4
12 495


<i>C</i>  cách lấy Ta tìm số


cách lấy 4 viên bi mà số bi đỏ lớn hơn số bi xanh, xảy ra các trường hợp sau:
TH1. Chọn 1 bi đỏ, 3 bi vàng  có 1 3


3.C4 12


<i>C</i>  cách chọn


TH2. Chọn 2 bi đỏ, 2 bi vàng  có 2 2


3.C4 18


<i>C</i>  cách chọn


TH3. Chọn 2 bi đỏ, 1 bi xanh, 1 bi vàng  có <i>C</i><sub>3</sub>2.C .C1<sub>4</sub> 1<sub>5</sub>60 cách chọn TH4. Chọn 3
bi đỏ, 1 bi vàng  có <i>C</i><sub>3</sub>3.C1<sub>4</sub> 4 cách chọn


TH5. Chọn 3 bi đỏ, 1 bi xanh  có <i>C</i><sub>3</sub>3.C1<sub>5</sub> 5 cách chọn Vậy xác suất để trong 4
viên bi được lấy số bi đỏ lớn hơn số bi xanh là:


<i>P</i>(<i>C</i><sub>3</sub>1.C3<sub>4</sub>+<i>C</i><sub>3</sub>2.C2<sub>4</sub>+<i>C</i><sub>3</sub>2.C .C1<sub>4</sub> 1<sub>5</sub>+<i>C</i><sub>3</sub>3.C1<sub>4</sub>+<i>C</i><sub>3</sub>3.C1<sub>5</sub>):<i>C</i><sub>12</sub>4 =

12 18 60 4 5 : 495

1
5


    


<b>Bài 124. </b> Chọn ngẫu nhiên một số có 4 chữ số đơi một khác nhau. Tính xác suất để số được chọn
khơng nhỏ hơn 2013.


Ta có <i>n</i>

 

 số cách chọn một số có bốn chữ số đơi một khác nhau9.9.8.7


<i>A</i> là biến cố chọn ra được một số có bốn chữ số đơi một khác nhau <i>abcd</i> và khơng nhỏ
hơn 2013. Ta sẽ tính số các số có bốn chữ số đơi một khác nhau <i>abcd</i> và các số này chỉ
có thể xảy ra với <i>a</i>1, <i>b</i>

0,1,...,9 \ 1

  

, <i>c</i>

0;1;...;9 \ 1;

  

<i>b</i> và


0;1;...;9 \ 1; ;

 



<i>d</i> <i>b c</i> có 7 cách chọn suy ra trong trường hợp này có 9.8.7 số thỏa


mãn. Từ hai trường hợp trên ta được <i>n A</i>

 

7.8.9.9 7.8.9 7.8.9.8. Do đó xác suất



cần tìm là:

 

 



 

7.8.9.89.9.8.7 89


<i>n A</i>
<i>P A</i>


<i>n</i>


  




</div>
<span class='text_page_counter'>(50)</span><div class='page_container' data-page=50>

<b>DAYHOCTOAN.VN </b>


50


<b>DAYHOCTOAN.VN </b>


Số cần tìm có dạng <i>abc</i>


Chọn <i>a</i><i>E a</i>, 0 có 5 cách.


Chọn 2 trong 5 số còn lại của <i>E</i>\

 

<i>a</i> xếp vào hai vị trí b, c có 2
5


<i>A</i> cách.


Vậy có 2



5


5.<i>A</i> 100(số) +) Tính số lập được chia hết cho 3.
Số cần tìm có dạng <i>abc</i>, <i>a b c</i>  3


Xét các tập con gồm 3 phần tử của tập <i>E</i>

0,1, 2,3, 4,5

, ta thấy chỉ có các tập sau thoả
mãn điều kiện tổng các chữ số chia hết cho 3 là:






1 2 3 4


5 6 7 8


0,1, 2 , 0,1,5 , 0, 2, 4 , 0, 4,5


1, 2,3 , 1,3,5 , 2,3, 4 , 3, 4,5


<i>A</i> <i>A</i> <i>A</i> <i>A</i>


<i>A</i> <i>A</i> <i>A</i> <i>A</i>


   


   


Khi <i>a b c</i>, , <i>A A A A</i><sub>1</sub>, <sub>2</sub>, <sub>3</sub>, <sub>4</sub> mỗi trường hợp lập được 4 số thoả mãn yêu cầu.
Khi <i>a b c</i>, , <i>A A A A</i>5; 6; 7; 8 mỗi trường hợp lập được 6 số thoả mãn yêu cầu.



Vậy có 4.4 4.6 40(số)


Suy ra số không chia hết cho 3 là100 40 60 (số)


Xác suất cần tính là 60 0, 60


100


<i>P</i> 


<b>Bài 126. </b> Từ tập hợp tất cả các số tự nhiên có năm chữ số mà các chữ số đều khác 0, lấy ngẫu
nhiên một số. Tính xác suất để trong số tự nhiên được lấy ra chỉ có mặt ba chữ số khác nhau. Ta
có:  95 59.049 Gọi <i>A</i> là biến cố cần tìm xác suất, ta có:


Số cách chọn 3 chữ số phân biệt <i>a, b, c </i>từ 9 chữ số thập phân khác 0 là 3
9


C . Chọn 2 chữ
số cịn lại từ 3 chữ số đó, có 2 trường hợp rời nhau sau đây:


TH1. Cả 2 chữ số còn lại cùng bằng 1 trong 3 chữ số <i>a, b, c</i>: có 3 cách; mỗi hốn vị từ
5! hoán vị của 5 chữ số (chẳng hạn) <i>a</i>, <i>a</i>, <i>a</i>, <i>b</i>, <i>c</i> tạo ra một số tự nhiên <i>n</i>; nhưng cứ 3!


hoán vị của các vị trí mà <i>a</i>, <i>a</i>, <i>a</i> chiếm chỗ thì chỉ tạo ra cùng một số <i>n</i>, nên trong TH1
này có cả thảy 3 5! 60


3!


  số tự nhiên.



TH2. 1 trong 2 chữ số còn lại bằng 1 trong 3 chữ số <i>a, b, c</i> và chữ số kia bằng 1 chữ số
khác trong 3 chữ số đó: có 3 cách; mỗi hốn vị từ 5! hoán vị của 5 chữ số (chẳng hạn)


<i>a</i>, <i>a</i>, <i>b</i>, <i>b</i>, <i>c</i> tạo ra một số tự nhiên <i>n</i>; nhưng cứ 2! hốn vị của các vị trí mà <i>a</i>, <i>a</i> chiếm
chỗ và 2! hoán vị của các vị trí mà <i>b</i>, <i>b</i> chiếm chỗ thì chỉ tạo ra cùng một số <i>n</i>, nên trong


TH2 này có cả thảy 3 5! 90


2!2!


  số tự nhiên. Vậy:


3
9


9!


(60 90)C 150 150 7 4 3 12600
3!6!


<i>A</i>


          . Kết luận:


 

12.600 1.400


0,213382106


59.049 6.561



<i>A</i>


<i>P A</i>     




</div>
<span class='text_page_counter'>(51)</span><div class='page_container' data-page=51>

<b>DAYHOCTOAN.VN </b>


51


<b>DAYHOCTOAN.VN </b>


a) tìm số tam giác có ít nhất 1 cạnh là cạnh của thập giác đó.
b) số tam giác khơng có cạnh nào là cạnh của thập giác đó.


TH1: số tam giác có đúng 1 cạnh là cạnh của thập giác là 6 10<i>x</i> 60 tam giác


TH2 số tam giác có 2 cạnh là cạnh của thập giác là 10 tam giác vi với mối đỉnh cùng hai
cạnh chứa đỉnh đó tạo thành 1 tam giác


Vậy số tam giác ít nhất 1 cạnh là cạnh của thập giác là: 60 10 70tam giác
b) Tổng số tam giác tạo thành từ 10 đỉnh là: 3


10 120
<i>C</i> 


Vậy số tam giác không co cạnh nào là cạnh của thập giác là:120 70 50 tam giác


<b>Bài 128. </b> Trên mặt phẳng cho đa giác lồi 10 cạnh <i>T</i> <i>A A</i>1 2...<i>An</i>. Xét các tam giác có 3 đỉnh là 3


đỉnh của đa giác <i>T</i>. Hỏi trong số các tam giác đó có bao nhiêu tam giác mà 3 cạnh của nó đều


khơng phải là ba cạnh của đa giác <i>T</i> ?


+) Số tam giác phân biệt có 3 đỉnh là 3 trong các đỉnh của đa giác <i>T</i> là <i>C</i><sub>10</sub>3 120
+) ứng với mỗi cạnh của đa giác <i>T</i> sẽ có 8 cách chọn các đỉnh còn lại để tạo thành một
tam giác chứa cạnh này. Suy ra số tam giác có ít nhất một cạnh là cạnh của đa giác <i>T</i>
là 80 (tam giác).


</div>

<!--links-->

Tài liệu bạn tìm kiếm đã sẵn sàng tải về

Tải bản đầy đủ ngay
×